Exam 1: Cardiac Practice Questions

Pataasin ang iyong marka sa homework at exams ngayon gamit ang Quizwiz!

Which of the following nursing diagnoses would be appropriate for a pt with heart failure? Select all that apply. 1. Ineffective tissue perfusion R/T decreased peripheral blood flow secondary to decreased CI 2. Activity intolerance R/T increased cardiac output. 3. Decreased cardiac output R/T structural & functional changes. 4. Impaired gas exchange R/T decreased sympathetic nervous system activity.

1 & 3. HF is a result of structural & functional abnormalities of the heart tissue muscle. The heart muscle becomes weak & does not adequately pump the blood out of the chambers. As a result, blood pools in the left ventricle & backs up into the left atrium, & eventually into the lungs. Therefore, greater amounts of blood remain in the ventricle after contraction thereby decreasing cardiac output. In addition, this pooling leads to thrombus formation & ineffective tissue perfusion because of the decrease in blood flow to the other organs & tissues of the body. Typically, these pts have an ejection fraction of less than 50% & poorly tolerate activity. Activity intolerance is related to a decrease, not increase, in cardiac output. Gas exchange is impaired. However, the decrease in cardiac output triggers compensatory mechanisms, such as an increase in sympathetic nervous system activity.

A nurse in a medical unit is caring for a client with heart failure. The client suddenly develops extreme dyspnea, tachycardia, and lung crackles and the nurse suspects pulmonary edema. The nurse immediately asks another nurse to contact the physician and prepares to implement which priority interventions? Select all that apply. 1. Administering oxygen 2. Inserting a Foley catheter 3. Administering furosemide (Lasix) 4. Administering morphine sulfate intravenously 5. Transporting the client to the coronary care unit 6. Placing the client in a low Fowler's side-lying position

1, 2, 3, 4 Rationale: Pulmonary edema is a life-threatening event that can result from severe heart failure. In pulmonary edema, the left ventricle fails to eject sufficient blood, and pressure increases in the lungs because of the accumulated blood. Oxygen is always prescribed, and the client is placed in a high Fowler's position to ease the work of breathing. Furosemide, a rapid-acting diuretic, will eliminate accumulated fluid. A Foley catheter is inserted to measure output accurately. Intravenously administered morphine sulfate reduces venous return (preload), decreases anxiety, and also reduces the work of breathing. Transporting the client to the coronary care unit is not a priority intervention. In fact, this may not be necessary at all if the client's response to treatment is successful.

A client is receiving morphine to relieve chest pain. The order is for 4 mg IV now. The pharmacy supplies morphine sulfate at 5 mg per mL. How many mL will the nurse give the client?

0.8 Explanation: (4 mg/5 mg) X 1 mL = 0.8 mL.

A nurse is monitoring the digoxin level for a client who has been taking a daily dose of digoxin for 1 month. the digoxin level is 0.25 ng/mL. The nurse should notify the provider and anticipate which of the following: 1. An increase in the client's digoxin dose. 2. A decrease in the client's digoxin dose. 3. No change in the client's digoxin dose. 4. Dicontinuation of the client's digoxin dose.

1

A pt's electrocardiogram strip shows atrial & ventricular rates of 80 complexes per minute. The PR interval is 0.14 second, & the QRS complex measures 0.08 second. The nurse interprets this rhythm is: 1. Normal sinus rhythm 2. Sinus bradycardia 3. Sinus tachycardia 4. Sinus dysrhythmia

1

The client comes into the emergency department saying, "I am having a heart attack" Which question is most pertinent when assessing the client? 1. "Can you describe the chest pain" 2. "What were you doing when the pain started" 3. "Did you have a high-fat meal today" 4. "Does the pain get worse when you lie down"

1

The client diagnosed with a myocardial infarction asks the nurse, "why do I have to rest and take it easy? My chest doesn't hurt anymore." Which statement would be the nurse's best response? 1. "Your heart is damaged and needs about 4 to 6 weeks to heal" 2. "There is necrotic myocardial tissue that puts you at risk for dysrhythmias" 3. "Your doctor has ordered bedrest. Therefore, you must stay in bed." 4. "Just because your chest doesn't hurt anymore doesn't mean you are out of danger"

1

The intensive care department nurse is assessing the client who is 12 hours post-myocardial infarction. The nurse assesses an S3 heart sound. Which intervention should the nurse implement? 1. Notify the health-care provider immediately 2. Elevate the head of the client's bed 3. Document this as a normal and expected finding 4. Administer morphine intravenously

1

The nurse is transcribing the doctor's orders for a client with congestive heart failure. The order reads 2.5 mg of Lanoxin daily. Which action should the nurse implement? 1. Discuss the order with the health-care provider 2. Take the client's apical pulse rate before administering 3. Check the client's potassium level before giving the medication 4. Determine if a digoxin level has been drawn

1

Which medical client problem should the nurse include in the plan of care for a client diagnosed with cardiomyopathy? 1. Heart Failure 2. Activity intolerance 3. Powerlessness 4. Anticipatory grieving

1

Which population is at a higher risk for dying from a myocardial infarction? 1. Caucasian Males 2. Hispanic Females 3. Asian Males 4. African American Females

1

A client with rapid rate atrial fibrillation asks a nurse why the physician is going to perform carotid sinus massage. Which of the following would be reflective of a correct explanation provided by the nurse? 1. The vagus nerve slows the heart rate. 2. The diaphragmatic nerve slows the heart rate. 3. The diaphragmatic nerve overdrives the rhythm. 4. The vagus nerve increases the heart rate, overdriving the rhythm.

1 Rationale: Carotid sinus massage is one maneuver used for vagal stimulation to decrease a rapid heart rate and possibly terminate a tachydysrhythmia. The others include inducing the gag reflex and asking the client to strain or bear down. Medication therapy often is needed as an adjunct to keep the rate down or maintain the normal rhythm. Options 2, 3, and 4 are incorrect descriptions of this procedure

A nurse is caring for a client who had a resection of an abdominal aortic aneurysm yesterday. The client has an intravenous infusion with a rate of 150 mL/hr, unchanged for the last 10 hours. The client's urine output for the last 3 hours was 90, 50, and 28 mL (28 mL most recent). The client's blood urea nitrogen level is 35 mg/dL and serum creatinine level is 1.8 mg/dL, measured this morning. Which of the following actions should the nurse take next? 1. Call the physician. 2. Check the urine specific gravity. 3. Check to see if the client had a sample for serum albumin level drawn. 4. Put the intravenous line on a pump so that the infusion rate is sure to stay stable.

1 Rationale: Following abdominal aortic aneurysm resection or repair, the nurse monitors the client for signs of renal failure. Renal failure can occur because often much blood is lost during the surgery and, depending on the aneurysm location, the renal arteries may be hypoperfused for a short period during surgery. The nurse monitors hourly intake and output and notes the results of daily blood urea nitrogen and creatinine levels. Urine output lower than 30 to 50 mL/hr is reported to the physician.

A nurse assesses the sternotomy incision of a client on the third day after cardiac surgery. The incision shows some slight "puffiness" along the edges and is nonreddened, with no apparent drainage. Temperature is 99° F orally. The white blood cell count is 7500 cells/mm3. How should the nurse interpret these findings? 1. Incision is slightly edematous but shows no active signs of infection. 2. Incision shows early signs of infection, although the temperature is nearly normal. 3. Incision shows early signs of infection, supported by an elevated white blood cell count. 4. Incision shows no sign of infection, although the white blood cell count is elevated.

1 Rationale: Sternotomy incision sites are assessed for signs and symptoms of infection, such as redness, swelling, induration, and drainage. Elevated temperature and white blood cell count after 3 to 4 days postoperatively usually indicate infection.

Intravenous heparin therapy is prescribed for a client. While implementing this prescription, a nurse ensures that which of the following medications is available on the nursing unit? 1. Protamine sulfate 2. Potassium chloride 3. Aminocaproic acid (Amicar) 4. Vitamin K (AquaMEPHYTON)

1 Rationale: The antidote to heparin is protamine sulfate; it should be readily available for use if excessive bleeding or hemorrhage should occur. Vitamin K is an antidote for warfarin sodium. Aminocaproic acid is the antidote for thrombolytic therapy. Potassium chloride is administered for a potassium deficit

A client has frequent bursts of ventricular tachycardia on the cardiac monitor. What should the nurse be most concerned about with this dysrhythmia? 1. It can develop into ventricular fibrillation at any time. 2. It is almost impossible to convert to a normal rhythm. 3. It is uncomfortable for the client, giving a sense of impending doom. 4. It produces a high cardiac output that quickly leads to cerebral and myocardial ischemia.

1 Rationale: Ventricular tachycardia is a life-threatening dysrhythmia that results from an irritable ectopic focus that takes over as the pacemaker for the heart. The low cardiac output that results can lead quickly to cerebral and myocardial ischemia. Clients frequently experience a feeling of impending doom. Ventricular tachycardia is treated with antidysrhythmic medications, cardioversion (client awake), or defibrillation (loss of consciousness). Ventricular tachycardia can deteriorate into ventricular fibrillation at any time.

A client has frequent bursts of ventricular tachycardia on the cardiac monitor. Why should the nurse be most concerned about with this dysrhythmia? 1. It can develop into ventricular fibrillation at any time. 2. It is almost impossible to convert to a normal rhythm. 3. It is uncomfortable for the client, giving a sense of impending doom. 4. It produces a high cardiac output that quickly leads to cerebral and myocardial ischemia.

1 Rationale: Ventricular tachycardia is a life-threatening dysrhythmia that results from an irritable ectopic focus that takes over as the pacemaker for the heart. The low cardiac output that results can lead quickly to cerebral and myocardial ischemia. Clients frequently experience a feeling of impending doom. Ventricular tachycardia is treated with antidysrhythmic medications, cardioversion (client awake), or defibrillation (loss of consciousness). Ventricular tachycardia can deteriorate into ventricular fibrillation at any time.

Furosemide is administered intravenously to a pt with HF. How soon after administration should the nurse begin to see evidence of the drugs desired effect? 1. 5 to 10 min 2. 30 to 60 min 3. 2 to 4 hours 4. 6 to 8 hours

1. After IV injection of furosemide, diuresis normally begins in about 5 minutes & reaches its peak w/in about 30 minutes. Med effects last 2 - 4 hours.

Which of the following terms describes the force against which the ventricle must expel blood? 1. Afterload 2. Cardiac output 3. Overload 4. Preload

1. Afterload refers to the resistance normally maintained by the aortic & pulmonic valves, the condition & tone of the aorta, & the resistance offered by the systemic & pulmonary arterioles. Cardiac output is the amount of blood expelled by the heart per minute. Overload refers to an abundance of circulating volume. Preload is the volume of blood in the ventricle at the end of diastole.

Which of the following types of cardiomyopathy can be associated with childbirth? 1. Dilated 2. Hypertrophic 3. Myocarditis 4. Restrictive

1. Although the cause isn't entirely known, cardiac dilation & heart failure may develop during the last month of pregnancy or the first few months after birth. The condition may result from a preexisting cardiomyopathy not apparent prior to pregnancy. Hypertrophic cardiomyopathy is an abnormal symmetry of the ventricles that has an unknown etiology but a strong familial tendency. Myocarditis isn't specifically associated w/ childbirth. Restrictive cardiomyopathy indicates constrictive pericarditis; the underlying cause is usually myocardial.

Which of the following recurring conditions most commonly occurs in pts with cardiomyopathy? 1. Heart failure 2. Diabetes 3. MI 4. Pericardial effusion

1. Because the structure & function of the heart muscle is affected, heart failure most commonly occurs in pts w/ cardiomyopathy. MI results from prolonged myocardial ischemia due to reduced blood flow through one of the coronary arteries. Pericardial effusion is most predominant in pts w/ pericarditis.

Which of the following classes of meds protects the ischemic myocardium by blocking catecholamines & sympathetic nerve stimulation? 1. Beta-adrenergic blockers 2. Calcium channel blockers 3. Narcotics 4. Nitrates

1. Beta-adrenergic blockers work by blocking beta receptors in the myocardium, reducing the response to catecholamines & sympathetic nerve stimulation. They protect the myocardium, helping to reduce the risk of another infarction by decreasing myocardial oxygen demand. Calcium channel blockers reduce the workload of the heart by decreasing the heart rate. Narcotics reduce myocardial oxygen demand promote vasodilation, & decrease anxiety. Nitrates reduce myocardial oxygen consumption by decreasing left ventricular end-diastolic pressure (preload) & systemic vascular resistance (afterload).

Which of the following heart muscle diseases is unrelated to other cardiovascular disease? 1. Cardiomyopathy 2. Coronary artery disease 3. Myocardial infarction 4. Pericardial effusion

1. Cardiomyopathy isn't usually related to an underlying heart disease such as atherosclerosis. The etiology in most cases is unknown. CAD & MI are directly related to atherosclerosis. Pericardial effusion is the escape of fluid into the pericardial sac, a condition associated w/ Pericarditis & advanced heart failure.

Which of the following symptoms is most commonly associated with left-sided heart failure? 1. Crackles 2. Arrhythmias 3. Hepatic engorgement 4. Hypotension

1. Crackles in the lungs are a classic sign of left-sided heart failure. These sounds are caused by fluid backing up into the pulmonary system. Arrhythmias can be associated w/ both right- & left-sided heart failure. Left-sided heart failure causes hypertension secondary to an increased workload on the system.

An 18-year-old pt who recently had an URI is admitted with suspected rheumatic fever. Which assessment findings confirm this diagnosis? 1. Erythema marginatum, subcutaneous nodules, & fever 2. Tachycardia, finger clubbing, & a load S3 3. Dyspnea, cough, & palpitations 4. Dyspnea, fatigue, & synocope

1. Diagnosis of rheumatic fever requires that the pt have either two major Jones criteria or one minor criterion plus evidence of a previous streptococcal infection. Major criteria include carditis, polyarthritis, Sydenham's chorea, subcutaneous nodules, & erythema maginatum (transient, nonprurtic macules on the trunk or inner aspects of the upper arms or thighs). Minor criteria include fever, arthralgia, elevated levels of acute phase reactants, & a prolonged PR-interval on ECG.

A nurse is providing teaching to a client who has a new prescription for digoxin (Lanoxin) Which of the following may indicate dig toxicity & should be reported to the provider? 1. Fatigue 2. constipation 3. Anorexia 4. Rash 5. Diplopia

1. Fatigue Not constipation but -- nausea, vomiting & diarrhea 3. Anorexia b/c GI disturbances 5. Diplopia -- visual changes , halo, yellow-tinged vision.

The physician orders continuous intravenous nitroglycerin infusion for the pt with MI. Essential nursing actions include which of the following? 1. Obtaining an infusion pump for the med 2. Monitoring BP q4h 3. Monitoring urine output hourly 4. Obtaining serum potassium levels daily

1. IV nitro infusion requires an infusion pump for precise control of the med. BP monitoring would be done w/ a continuous system, & more frequently than every 4 hours. Hourly urine outputs are not always required. Obtaining serum potassium levels is not associated w/ nitroglycerin infusion.

A pt admitted with angina compains of severe chest pain & suddenly becomes unresponsive. After establishing unresponsiveness, which of the following actions should the nurse take first? 1. Activate the resuscitation team 2. Open the pt's airway 3. Check for breathing 4. Check for signs of circulation

1. Immediately after establishing unresponsiveness, the nurse should activate the resuscitation team. The next step is to open the airway using the head-tilt, chin-lift maneuver & check for breathing (looking, listening, & feeling for no more than 10-seconds). If the pt isn't breathing, give two slow breaths using a bag mask or pocket mask. Next, check for signs of circulation by palpating the carotid pulse.

An older, sedentary adult may not respond to emotional or physical stress as well as a younger individual because of: 1. Left ventricular atrophy 2. Irregular heartbeats 3. peripheral vascular occlusion 4. Pacemaker placement

1. In older adults who are less active & do not exercise the heart muscle, atrophy can result. Disuse or deconditioning can lead to abnormal changes in the myocardium of the older adult. As a result, under sudden emotional or physical stress, the left ventricle is less able to respond to the increased dem&s on the myocardial muscle.

Toxicity from which of the following meds may cause a pt to see a green-yellow halo around lights? 1. Digoxin 2. Furosemide (Lasix) 3. Metoprolol (Lopressor) 4. Enalapril (Vasotec)

1. One of the most common signs of digoxin toxicity is the visual disturbance known as the "green-yellow halo sign." The other meds aren't associated w/ such an effect.

Which of the following actions is the appropriate initial response to a pt coughing up pink, frothy sputum? 1. Call for help 2. Call the physician 3. Start an I.V. line 4. Suction the pt

1. Production of pink, frothy sputum is a classic sign of acute pulmonary edema. Because the pt is at high risk for decompensation, the nurse should call for help but not leave the room. The other three interventions would immediately follow.

Which of the following complications is indicated by a third heart sound (S3)? 1. Ventricular dilation 2. Systemic hypertension 3. Aortic valve malfunction 4. Increased atrial contractions

1. Rapid filling of the ventricle causes vasodilation that is auscultated as S3. Increased atrial contraction or systemic hypertension can result in a fourth heart sound. Aortic valve malfunction is heard as a murmur.

Which of the following results is the primary treatment goal for angina? 1. Reversal of ischemia 2. Reversal of infarction 3. Reduction of stress & anxiety 4. Reduction of associated risk factors

1. Reversal of the ischemia is the primary goal, achieved by reducing oxygen consumption & increasing oxygen supply. An infarction is permanent & can't be reversed.

A pt comes into the E.R. with acute shortness of breath & a cough that produces pink, frothy sputum. Admission assessment reveals crackles & wheezes, a BP of 85/46, a HR of 122 BPM, & a respiratory rate of 38 breaths/minute. The pt's medical history included DM, HTN, & heart failure. Which of the following disorders should the nurse suspect? 1. Pulmonary edema 2. Pneumothorax 3. Cardiac tamponade 4. Pulmonary embolus

1. SOB, tachypnea, low BP, tachycardia, crackles, & a cough producing pink, frothy sputum are late signs of pulmonary edema.

Which of the following interventions should be the first priority when treating a pt experiencing chest pain while walking? 1. Sit the pt down 2. Get the pt back to bed 3. Obtain an ECG 4. Administer sublingual nitroglycerin

1. The initial priority is to decrease the oxygen consumption; this would be achieved by sitting the pt down. An ECG can be obtained after the pt is sitting down. After the ECGm sublingual nitro would be administered. When the pt's condition is stabilized, he can be returned to bed.

After an anterior wall myocardial infarction, which of the following problems is indicated by auscultation of crackles in the lungs? 1. Left-sided heart failure 2. Pulmonic valve malfunction 3. Right-sided heart failure 4. Tricupsid valve malfunction

1. The left ventricle is responsible for most of the cardiac output. An anterior wall MI may result in a decrease in left ventricular function. When the left ventricle doesn't function properly, resulting in left-sided heart failure, fluid accumulates in the interstitial & alveolar spaces in the lungs & causes crackles. Pulmonic & tricuspid valve malfunction causes right sided heart failure.

Which of the following is the most common symptom of myocardial infarction (MI)? 1. Chest pain 2. Dyspnea 3. Edema 4. Palpitations

1. The most common symptom of an MI is chest pain, resulting from deprivation of oxygen to the heart. Dyspnea is the second most common symptom, related to an increase in the metabolic needs of the body during an MI. Edema is a later sign of heart failure, often seen after an MI. Palpitations may result from reduced cardiac output, producing arrhythmias.

When developing a teaching plan for a pt with endocarditis, which of the following points is most essential for the nurse to include? 1. "Report fever, anorexia, & night sweats to the physician." 2. "Take prophylactic antibiotics after dental work & invasive procedures." 3. "Include potassium rich foods in your diet." 4. "Monitor your pulse regularly."

1. The most essential teaching point is to report signs of relapse, such as fever, anorexia, & night sweats, to the physician. To prevent further endocarditis episodes, prophylactic antibiotics are taken before & sometimes after dental work, childbirth, or GU, GI, or gynecologic procedures. A potassium-rich diet & daily pulse monitoring aren't necessary for a pt w/ endocarditis.

Along with persistent, crushing chest pain, which signs/symptoms would make the nurse suspect that the client is experiencing a myocardial infarction? 1. Midepigastric pain and pyrosis 2. Diaphoresis and cool clammy skin 3. Intermittent claudication and paloor 4. Jugular vein distention and dependent edema

2

The client is one day postoperative coronary artery bypass surgery. The client complains of chest pain. Which intervention should the nurse implement first? 1. Medicate the client with intravenous morphine 2. Assess the client's chest dressing and vital signs 3. Encourage the client to turn from side to side 4. Check the client's telemetry monitor

2

Which intervention should the nurse implement with the client diagnosed with dilated cardiomyopathy? 1. Keep the client in the supine position with legs elevated 2. Discuss a heart transplant, which is a definitive treatment 3. Prepare the client for coronary artery bypass graft 4. Teach the client to take a calcium channel blocker in the morning

2

A client with myocardial infarction is developing cardiogenic shock. Because of the risk of myocardial ischemia, for which of the following should the nurse carefully assess the client? 1. Bradycardia 2. Ventricular dysrhythmias 3. Rising diastolic blood pressure 4. Falling central venous pressure

2 Rationale: Classic signs of cardiogenic shock as they relate to this question include low blood pressure and tachycardia. The central venous pressure would rise as the backward effects of the severe left ventricular failure became apparent. Dysrhythmias commonly occur as a result of decreased oxygenation and severe damage to greater than 40% of the myocardium.

A client with atrial fibrillation is receiving a continuous heparin infusion at 1000 units/hr. The nurse would determine that the client is receiving the therapeutic effect based on which of the following results? 1. Prothrombin time of 12.5 seconds 2. Activated partial thromboplastin time of 60 seconds 3. Activated partial thromboplastin time of 28 seconds 4. Activated partial thromboplastin time longer than 120 seconds

2 Rationale: Common laboratory ranges for activated partial thromboplastin time are 20 to 36 seconds. Because the activated partial thromboplastin time should be 1.5 to 2.5 times the normal value, the client's activated partial thromboplastin time would be considered therapeutic if it was 60 seconds.

nurse is evaluating a client's response to cardioversion. Which of the following observations would be of highest priority to the nurse? 1. Blood pressure 2. Status of airway 3. Oxygen flow rate 4. Level of consciousness

2 Rationale: Nursing responsibilities after cardioversion include maintenance first of a patent airway, and then oxygen administration, assessment of vital signs and level of consciousness, and dysrhythmia detection.

A client with myocardial infarction suddenly becomes tachycardic, shows signs of air hunger, and begins coughing frothy, pink-tinged sputum. Which of the following would the nurse anticipate when auscultating the client's breath sounds? 1. Stridor 2. Crackles 3. Scattered rhonchi 4. Diminished breath sounds

2 Rationale: Pulmonary edema is characterized by extreme breathlessness, dyspnea, air hunger, and the production of frothy, pink-tinged sputum. Auscultation of the lungs reveals crackles. Rhonchi and diminished breath sounds are not associated with pulmonary edema. Stridor is a crowing sound associated with laryngospasm or edema of the upper airway.

A client who had cardiac surgery 24 hours ago has a urine output averaging 20 mL/hr for 2 hours. The client received a single bolus of 500 mL of intravenous fluid. Urine output for the subsequent hour was 25 mL. Daily laboratory results indicate that the blood urea nitrogen level is 45 mg/dL and the serum creatinine level is 2.2 mg/dL. Based on these findings, the nurse would anticipate that the client is at risk for which of the following? 1. Hypovolemia 2. Acute renal failure 3. Glomerulonephritis 4. Urinary tract infection

2 Rationale: The client who undergoes cardiac surgery is at risk for renal injury from poor perfusion, hemolysis, low cardiac output, or vasopressor medication therapy. Renal insult is signaled by decreased urine output and increased blood urea nitrogen and creatinine levels. The client may need medications to increase renal perfusion and possibly could need peritoneal dialysis or hemodialysis. No data in the question indicate the presence of hypovolemia, urinary tract infection, or glomerulonephritis.

A client with angina complains that the anginal pain is prolonged and severe and occurs at the same time each day, most often at rest in the absence of precipitating factors. How would the nurse best describe this type of anginal pain? 1. Stable angina 2. Variant angina 3. Unstable angina 4. Nonanginal pain

2 Rationale: Variant angina, or Prinzmetal's angina, is prolonged and severe and occurs at the same time each day, most often at rest. Stable angina is induced by exercise and relieved by rest or nitroglycerin tablets. Unstable angina occurs at lower and lower levels of activity or at rest, is less predictable, and is often a precursor of myocardial infarction.

The nurse finds the apical pulse below the 5th intercostal space. The nurse suspects: 1. Left atrial enlargement 2. Left ventricular enlargement 3. Right atrial enlargement 4. Right ventricular enlargement

2. A normal apical impulse is found under over the apex of the heart & is typically located & auscultated in the left fifth intercostal space in the midclavicular line. An apical impulse located or auscultated below the fifth intercostal space or lateral to the midclavicular line may indicate left ventricular enlargement.

What is the first intervention for a pt experiencing MI? 1. Administer morphine 2. Administer oxygen 3. Administer sublingual nitroglycerin 4. Obtain an ECG

2. Administering supplemental oxygen to the pt is the first priority of care. The myocardium is deprived of oxygen during an infarction, so additional oxygen is administered to assist in oxygenation & prevent further damage. Morphine & nitro are also used to treat MI, but they're more commonly administered after the oxygen. An ECG is the most common diagnostic tool used to evaluate MI.

Aspirin is administered to the pt experiencing an MI because of its: 1. Antipyrectic action 2. Antithrombotic action 3. Antiplatelet action 4. Analgesic action

2. Aspirin does have antipyretic, antiplatelet, & analgesic actions, but the primary reason ASA is administered to the pt experiencing an MI is its antithrombotic action.

Which of the following classes of drugs is most widely used in the treatment of cardiomyopathy? 1. Antihypertensives 2. Beta-adrenergic blockers 3. Calcium channel blockers 4. Nitrates

2. By decreasing the heart rate & contractility, beta-blockers improve myocardial filling & cardiac output, which are primary goals in the treatment of cardiomyopathy. Antihypertensives aren't usually indicated because they would decrease cardiac output in pts who are already hypotensive. Calcium channel blockers are sometimes used for the same reasons as beta-blockers; however, they aren't as effective as beta-blockers & cause increased hypotension. Nitrates aren't used because of their dilating effects, which would further compromise the myocardium.

Which of the following foods should the nurse teach a pt with heart failure to avoid or limit when following a 2-gram sodium diet? 1. Apples 2. Tomato juice 3. Whole wheat bread 4. Beef tenderloin

2. Canned foods & juices, such as tomato juice, are typically high in sodium & should be avoided in a sodium-restricted diet. BRING ON THE STEAK!

In which of the following types of cardiomyopathy does cardiac output remain normal? 1. Dilated 2. Hypertrophic 3. Obliterative 4. Restrictive

2. Cardiac output isn't affected by hypertrophic cardiomyopathy because the size of the ventricle remains relatively unchanged. All of the rest decrease cardiac output.

A nurse is preparing for the admission of a pt with heart failure who is being sent directly to the hospital from the physician's office. The nurse would plan on having which of the following meds readily available for use? 1. Diltiazem (Cardizem) 2. Digoxin (Lanoxin) 3. Propranolol (Inderal) 4. Metoprolol (Lopressor)

2. Digoxin exerts a positive inotropic effect on the heart while slowing the overall rate through a variety of mechanisms. Digoxin is the med of choice to treat heart failure. Diltiazem (calcium channel blocker) & propranolol & metoprolol (beta blockers) have a negative inotropic effect & would worsen the failing heart.

With which of the following disorders is jugular vein distention most prominent? 1. Abdominal aortic aneurysm 2. Heart failure 3. MI 4. Pneumothorax

2. Elevated venous pressure, exhibited as jugular vein distention, indicates a failure of the heart to pump. JVD isn't a symptom of abdominal aortic aneurysm or pneumothorax. An MI, if severe enough, can progress to heart failure, however, in & of itself, an MI doesn't cause JVD.

Which of the following actions is the first priority of care for a pt exhibiting signs & symptoms of coronary artery disease? 1. Decrease anxiety 2. Enhance myocardial oxygenation 3. Administer sublingual nitroglycerin 4. Educate the pt about his symptoms

2. Enhancing myocardial oxygenation is always the first priority when a pt exhibits signs or symptoms of cardiac compromise. W/out adequate oxygenation, the myocardium suffers damage. Sublingual nitroglycerin is administered to treat acute angina, but administration isn't the first priority. Although educating the pt & decreasing anxiety are important in care delivery, neither are priorities when a pt is compromised.

A nurse is administering a dopamine infusion at a moderate dose to a client who has severe HF. Which of the following is an expected effect? 1. Lowered heart rate 2. Increased myocardial contractility 3. Decreased conduction through the AV node 4. Vasoconstriction of the renal blood vessels

2. Increased myocardial contractility -- thus increasing CO

A 55-year-old pt is admitted with an acute inferior-wall myocardial infarction. During the admission interview, he says he stopped taking his metoprolol (Lopressor) 5 days ago because he was feeling better. Which of the following nursing diagnoses takes priority for this pt? 1. Anxiety 2. Ineffective tissue perfusion; cardiopulmonary 3. Acute pain 4. Ineffective therapeutic regimen management

2. MI results from prolonged myocardial ischemia caused by reduced blood flow through the coronary arteries. Therefore, the priority nursing diagnosis for this pt is Ineffective tissue perfusion (cardiopulmonary). Anxiety, acute pain, & ineffective therapeutic regimen management are appropriate but don't take priority.

Which of the following reflects the principle on which a pt's diet will most likely be based during the acute phase of MI? 1. Liquids as ordered 2. Small, easily digested meals 3. Three regular meals per day 4. NPO

2. Recommended dietary principles in the acute phase of MI include avoiding large meals because small, easily digested foods are better digested foods are better tolerated. Fluids are given according to the pt's needs, & sodium restrictions may be prescribed, especially for pts w/ manifestations of heart failure. Cholesterol restrictions may be ordered as well. Pts are not prescribed a diet of liquids only or NPO unless their condition is very unstable.

Stimulation of the sympathetic nervous system produces which of the following responses? 1. Bradycardia 2. Tachycardia 3. Hypotension 4. Decreased myocardial contractility

2. Stimulation of the sympathetic nervous system causes tachycardia & increased contractility. The other symptoms listed are related to the parasympathetic nervous system, which is responsible for slowing the heart rate.

A home care nurse is making a routine visit to a pt receiving digoxin (Lanoxin) in the treatment of heart failure. The nurse would particularly assess the pt for: 1. Thrombocytopenia & weight gain 2. Anorexia, nausea, & visual disturbances 3. Diarrhea & hypotension 4. Fatigue & muscle twitching

2. The first signs & symptoms of digoxin toxicity in adults include abdominal pain, N/V, visual disturbances (blurred, yellow, or green vision, halos around lights), bradycardia, & other dysrhythmias.

A nurse is preparing to ambulate a pt on the 3rd day after cardiac surgery. The nurse would plan to do which of the following to enable the pt to best tolerate the ambulation? 1. Encourage the pt to cough & deep breathe 2. Premedicate the pt with an analgesic 3. Provide the pt with a walker 4. Remove telemetry equipment because it weighs down the hospital gown.

2. The nurse should encourage regular use of pain med for the first 48 to 72 hours after cardiac surgery because analgesia will promote rest, decrease myocardial oxygen consumption resulting from pain, & allow better participation in activities such as coughing, deep breathing, & ambulation. Options 1 & 3 will not help in tolerating ambulation. Removal of telemetry equipment is contraindicated unless prescribed.

Which of the following conditions is associated with a predictable level of pain that occurs as a result of physical or emotional stress? 1. Anxiety 2. Stable angina 3. Unstable angina 4. Variant angina

2. The pain of stable angina is predictable in nature, builds gradually, & quickly reaches maximum intensity. Unstable angina doesn't always need a trigger, is more intense, & lasts longer than stable angina. Variant angina usually occurs at rest—not as a result of exercise or stress.

The nurse is caring for a client diagnosed with a myocardial infarction who is experiencing chest pain. Which interventions should the nurse implement first? Select All that Apply 1. Administer morphine sulfate Intramuscularly 2. Administer an aspirin orally 3. Apply oxygen via nasal cannula 4. Place the client in a supine position 5. Administer nitroglycerin subcutaneously

23

The client is admitted to the emergency department, and the nurse suspects a cardiac problem. Which assessment interventions should the nurse implement? Select All that Apply 1. Obtain a midstream urine specimen 2. Attach telemetry monitor to the client 3. Start a saline lock in the right arm 4. Draw a baseline metabolic panel (BMP) 5. Request an order for a STAT 12-lead ECG

235

The nurse is monitoring a client who is taking digoxin (Lanoxin) for adverse effects. Which findings are characteristic of digoxin toxicity. Select all that apply. 1. Tremors 2. Diarrhea 3. Irritability 4. Blurred vision 5. Nausea and vomiting

245 Rationale: Digoxin (Lanoxin) is a cardiac glycoside. The risk of toxicity can occur with the use of this medication. Toxicity can lead to life-threatening events and the nurse needs to monitor the client closely for signs of toxicity. Early signs of toxicity include gastrointestinal manifestations such as anorexia, nausea, vomiting, and diarrhea. Subsequent manifestations include headache, visual disturbances such as diplopia, blurred vision, yellow-green halos, photophobia, drowsiness, fatigue, and weakness. Cardiac rhythm abnormalities can also occur. The nurse also monitors the digoxin level. Therapeutic levels for digoxin range from 0.5 to 2 ng/mL.

A client with HF has an order for lisnopril (Prinivil, Zestril) Which of the following conditions in the client's history would lead a nurse to confirm the order with the provider? 1. A history of HT previously treated with diuretics. 2. A history of seasonal allergies currently treated with antihistamines. 3. A history of angioedema after taking enalapril (Vasotec) 4. A history of alcoholism, currently abstaining.

3

The client diagnosed with a myocardial infarction is on bedrest. The unlicensed assistive personnel is encouraging the client to move the legs. Which action should the nurse implement? 1. Instruct the UAP to stop encouraging leg movements 2. Report this behavior to the charge nurse as soon as possible 3. Praise the UAP for encouraging the client to move legs 4. Take no action concerning the UAP's behavior

3

The client diagnosed with a myocardial infarction is six hours post-right femoral percutanous transluminal coronary angioplasty (PTCA), also known as balloon surgery. Which assessment data would require immediate intervention by the nurse? 1. The client is keeping the affected extremity straight 2. The pressure dressing to the right femoral area is intact 3. The client is complaining of numbness in the right foot 4. The client's right pedal pulse is +3 and bounding

3

The client diagnosed with rule-out myocardial infarction is experiencing chest pain while walking to the bathroom. Which action should the nurse implement first? 1. Administer sublingual nitroglycerin 2. Obtain a stat 12 Lead ECG 3. Have the client sit down immediately 4. Assess the client's vital signs

3

The client has just returned from a cardiac catherization. Which assessment data would warrant immediate intervention from the nurse? 1. The client's BP is 110/70 and pulse is 90 2. The client's groin dressing is dry and intact 3. The client refuses to keep the leg straight 4. The client denies any numbness and tingling

3

The client who has had a myocardial infarction is admitted to the telementry unit from intensive care. Which referral would be most appropriate for the client? 1. Social worker 2. Physical therapy 3. Cardiac rehabilitation 4. Occupation therapy

3

Which client would most likely be misdiagnosed for having a myocardial infarction? 1. A 55 year old Caucasian male with crushing chest pain and diaphoresis 2. A 60 year old Native American male with an elevated troponin level 3. A 40 year old Hispanic female with a normal ECG 4. An 80 year old Peruvian female with normal CK-MB at 12 hours

3

Which preprocedure information should be taught to the female client having an exercise stress test in the morning? 1. Wear open-toed shoes to the stress test 2. Inform the client not to wear a bra 3. Do not eat anything for 4 hours 4. Take the beta blocker one hour before the test

3

A 66-year-old client complaining of not feeling well is seen in a clinic. The client is taking several medications for the control of heart disease and hypertension. These medications include atenolol (Tenormin), digoxin (Lanoxin), and chlorothiazide (Diuril). A tentative diagnosis of digoxin toxicity is made. Which of the following assessment data would support this diagnosis? 1. Dyspnea, edema, and palpitations 2. Chest pain, hypotension, and paresthesia 3. Double vision, loss of appetite, and nausea 4. Constipation, dry mouth, and sleep disorder

3 Rationale: Double vision, loss of appetite, and nausea are early signs of digoxin toxicity. Additional signs of digoxin toxicity include bradycardia, difficulty reading, other visual alterations such as green and yellow vision or seeing spots or halos, confusion, vomiting, diarrhea, decreased libido, and impotence.

A client is wearing a continuous cardiac monitor, which begins to sound its alarm. A nurse sees no electrocardiographic complexes on the screen. Which of the following should be the priority action of the nurse? 1. Call a code blue. 2. Call the physician. 3. Check the client status and lead placement. 4. Press the recorder button on the electrocardiogram console.

3 Rationale: Sudden loss of electrocardiographic complexes indicates ventricular asystole or possibly electrode displacement. Accurate assessment of the client and equipment is necessary to determine the cause and identify the appropriate intervention. Options 1, 2, and 4 are unnecessary

A home health care nurse is visiting an older client at home. Furosemide (Lasix) is prescribed for the client and the nurse teaches the client about the medication. Which of the following statements, if made by the client, indicates the need for further teaching? 1. "I will sit up slowly before standing each morning." 2. "I will take my medication every morning with breakfast." 3. "I need to drink lots of coffee and tea to keep myself healthy." 4. "I will call my doctor if my ankles swell or my rings get tight."

3 Rationale: Tea and coffee are stimulants and mild diuretics. These are a poor choice for hydration. Taking the medication at the same time each day improves compliance. Because furosemide is a diuretic, the morning is the best time to take the medication so as not to interrupt sleep. Notification of the health care provider is appropriate if edema is noticed in the hands, feet, or face or if the client is short of breath. Sitting up slowly prevents postural hypotension.

A nurse is preparing to ambulate a client on the third day after cardiac surgery. The nurse would plan to do which of the following to enable the client to best tolerate the ambulation? 1. Remove telemetry equipment. 2. Provide the client with a walker. 3. Premedicate the client with an analgesic. 4. Encourage the client to cough and deep breathe.

3 Rationale: The nurse should encourage regular use of pain medication for the first 48 to 72 hours after cardiac surgery because analgesia will promote rest, decrease myocardial oxygen consumption resulting from pain, and allow better participation in activities such as coughing, deep breathing, and ambulation. Options 2 and 4 will not help in tolerating ambulation. Removal of telemetry equipment is contraindicated unless prescribed.

A nurse is planning to administer hydrochlorothiazide (HydroDIURIL) to a client. The nurse understands that which of the following are concerns related to the administration of this medication? 1. Hypouricemia, hyperkalemia 2. Increased risk of osteoporosis 3. Hypokalemia, hyperglycemia, sulfa allergy 4. Hyperkalemia, hypoglycemia, penicillin allergy

3 Rationale: Thiazide diuretics such as hydrochlorothiazide are sulfa-based medications, and a client with a sulfa allergy is at risk for an allergic reaction. Also, clients are at risk for hypokalemia, hyperglycemia, hypercalcemia, hyperlipidemia, and hyperuricemia.

A client is admitted with pulmonary embolism and is to be treated with streptokinase (Streptase). A nurse would report which of the following assessments to the physician before initiating this therapy? 1. Adventitious breath sounds 2. Temperature of 99.4° F orally 3. Blood pressure of 198/110 mm Hg 4. Respiratory rate of 28 breaths/min

3 Rationale: Thrombolytic therapy is contraindicated in a number of preexisting conditions in which there is a risk of uncontrolled bleeding, similar to the case in anticoagulant therapy. Thrombolytic therapy also is contraindicated in severe uncontrolled hypertension because of the risk of cerebral hemorrhage. Therefore the nurse would report the results of the blood pressure to the physician before initiating therapy.

A client is diagnosed with an acute myocardial infarction and is receiving tissue plasminogen activator, alteplase (Activase, tPA). Which of the following is a priority nursing intervention? 1. Monitor for renal failure. 2. Monitor psychosocial status. 3. Monitor for signs of bleeding. 4. Have heparin sodium available

3 Rationale: Tissue plasminogen activator is a thrombolytic. Hemorrhage is a complication of any type of thrombolytic medication. The client is monitored for bleeding. Monitoring for renal failure and monitoring the client's psychosocial status are important but are not the most critical interventions. Heparin is given after thrombolytic therapy, but the question is not asking about follow-up medications.

When teaching a patient with peripheral arterial disease, the nurse determines that further teaching is needed when the patient says, 1. "I should not use heating pads to warm my feet." 2. "I will examine my feet every day for any sores or red areas." 3. "I should cut back on my walks if they cause pain in my legs." 4. "I think I can quit smoking with the use of short-term nicotine replacement and support groups."

3. "I should cut back on my walks if they cause pain in my legs." (Rationale: Patients should be taught to exercise to the point of discomfort, stop and rest, and then resume walking until the discomfort recurs. Smoking cessation and proper foot care are also important interventions for patients with peripheral arterial disease.)

Which of the following positions would best aid breathing for a pt with acute pulmonary edema? 1. Lying flat in bed 2. Left side-lying 3. In high Fowler's position 4. In semi-Fowler's position

3. A high Fowler's position promotes ventilation & facilitates breathing by reducing venous return. Lying flat & side-lying positions worsen the breathing & increase workload of the heart. Semi-Fowler's position won't reduce the workload of the heart as well as the Fowler's position will.

Captopril may be administered to a pt with HF because it acts as a: 1. Vasopressor 2. Volume expander 3. Vasodilator 4. Potassium-sparing diuretic

3. ACE inhibitors have become the vasodilators of choice in the pt w/ mild to severe HF. Vasodilator drugs are the only class of drugs clearly shown to improve survival in overt heart failure.

Which of the following would be a priority nursing diagnosis for the pt with heart failure & pulmonary edema? 1. Risk for infection related to stasis of alveolar secretions 2. Impaired skin integrity related to pressure 3. Activity intolerance related to pump failure 4. Constipation related to immobility

3. Activity intolerance is a primary problem for pts w/ heart failure & pulmonary edema. The decreased cardiac output associated w/ heart failure leads to reduced oxygen & fatigue. Pts frequently complain of dyspnea & fatigue. The pt could be at risk for infection related to stasis of secretions or impaired skin integrity related to pressure. However, these are not the priority nursing diagnoses for the pt w/ HF & pulmonary edema, nor is constipation related to immobility.

What is the most common complication of an MI? 1. Cardiogenic shock 2. Heart failure 3. arrhythmias 4. Pericarditis

3. Arrhythmias, caused by oxygen deprivation to the myocardium, are the most common complication of an MI. Cardiogenic shock, another complication of an MI, is defined as the end stage of left ventricular dysfunction. This condition occurs in approximately 15% of pts w/ MI. Because the pumping function of the heart is compromised by an MI, heart failure is the second most common complication. Pericarditis most commonly results from a bacterial or viral infection but may occur after the MI.

Which of the following conditions is most commonly responsible for myocardial infarction? 1. Aneurysm 2. Heart failure 3. Coronary artery thrombosis 4. Renal failure

3. Coronary artery thrombosis causes an inclusion of the artery, leading to myocardial death. An aneurysm is an outpouching of a vessel & doesn't cause an MI. Renal failure can be associated w/ MI but isn't a direct cause. Heart failure is usually a result from an MI.

A nurse is conducting a health history with a pt with a primary diagnosis of heart failure. Which of the following disorders reported by the pt is unlikely to play a role in exacerbating the heart failure? 1. Recent URI 2. Nutritional anemia 3. Peptic ulcer disease 4. A-Fib

3. Heart failure is precipitated or exacerbated by physical or emotional stress, dysrhythmias, infections, anemia, thyroid disorders, pregnancy, Paget's disease, nutritional deficiencies (thiamine, alcoholism), pulmonary disease, & hypervolemia.

Septal involvement occurs in which type of cardiomyopathy? 1. Congestive 2. Dilated 3. Hypertrophic 4. Restrictive

3. In hypertrophic cardiomyopathy, hypertrophy of the ventricular septum—not the ventricle chambers—is apparent. This abnormality isn't seen in other types of cardiomyopathy.

Which of the following symptoms might a pt with right-sided heart failure exhibit? 1. Adequate urine output 2. Polyuria 3. Oliguria 4. Polydipsia

3. Inadequate deactivation of aldosterone by the liver after right-sided heart failure leads to fluid retention, which causes oliguria.

Myocardial oxygen consumption increases as which of the following parameters increase? 1. Preload, afterload, & cerebral blood flow 2. Preload, afterload, & renal blood flow 3. Preload, afterload, contractility, & heart rate. 4. Preload, afterload, cerebral blood flow, & heart rate.

3. Myocardial oxygen consumption increases as preload, afterload, renal contractility, & heart rate increase. Cerebral blood flow doesn't directly affect myocardial oxygen consumption.

Medical treatment of coronary artery disease includes which of the following procedures? 1. Cardiac catherization 2. Coronary artery bypass surgery 3. Oral med therapy 4. Percutaneous transluminal coronary angioplasty

3. Oral med administration is a noninvasive, medical treatment for coronary artery disease. Cardiac catherization isn't a treatment, but a diagnostic tool. Coronary artery bypass surgery & percutaneous transluminal coronary angioplasty are invasive, surgical treatments.

A pt with angina complains that the angina pain is prolonged & severe & occurs at the same time each day, most often in the morning, On further assessment a nurse notes that the pain occurs in the absence of precipitating factors. This type of anginal pain is best described as: 1. Stable angina 2. Unstable angina 3. Variant angina 4. Nonanginal pain

3. Stable angina is induced by exercise & is relieved by rest or nitroglycerin tablets. Unstable angina occurs at lower & lower levels of activity & rest, is less predictable, & is often a precursor of myocardial infarction. Variant angina, or Prinzmetal's angina, is prolonged & severe & occurs at the same time each day, most often in the morning.

Acute pulmonary edema caused by heart failure is usually a result of damage to which of the following areas of the heart? 1. Left atrium 2. Right atrium 3. Left ventricle 4. Right ventricle

3. The left ventricle is responsible for the majority of force for the cardiac output. If the left ventricle is damaged, the output decreases & fluid accumulates in the interstitial & alveolar spaces, causing pulmonary edema. Damage to the left atrium would contribute to heart failure but wouldn't affect cardiac output or, therefore, the onset of pulmonary edema. If the right atrium & right ventricle were damaged, right-sided heart failure would result.

If medical treatments fail, which of the following invasive procedures is necessary for treating cariomyopathy? 1. Cardiac catherization 2. Coronary artery bypass graft (CABG) 3. Heart transplantation 4. Intra-aortic balloon pump (IABP)

3. The only definitive treatment for cardiomyopathy that can't be controlled medically is a heart transplant because the damage to the heart muscle is irreversible.

A pt with pulmonary edema has been on diuretic therapy. The pt has an order for additional furosemide (Lasix) in the amount of 40 mg IV push. Knowing that the pt also will be started on Digoxin (Lanoxin), a nurse checks the pt's most recent: 1. Digoxin level 2. Sodium level 3. Potassium level 4. Creatinine level

3. The serum potassium level is measured in the pt receiving digoxin & furosemide. Heightened digitalis effect leading to digoxin toxicity can occur in the pt w/ hypokalemia. Hypokalemia also predisposes the pt to ventricular dysrhythmias.

Which of the following blood tests is most indicative of cardiac damage? 1. Lactate dehydrogenase 2. Complete blood count (CBC) 3. Troponin I 4. Creatine kinase (CK)

3. Troponin I levels rise rapidly & are detectable w/in 1 hour of myocardial injury. Troponin I levels aren't detectable in people w/out cardiac injury. Lactate dehydrogenase (LDH) is present in almost all body tissues & not specific to heart muscle. LDH isoenzymes are useful in diagnosing cardiac injury. CBC is obtained to review blood counts, & a complete chemistry is obtained to review electrolytes. Because CK levels may rise w/ skeletal muscle injury, CK isoenzymes are required to detect cardiac injury

The client is 3 hours post myocardial infarction. Which data would warrant immediate intervention by the nurse? 1. Bilateral peripheral pulses 2+ 2. The pulse oximeter reading is 96% 3. The urine output is 240 mL in the last 4 hours 4. Cool, clammy, diaphoretic skin

4

The nurse is administering a calcium channel blocker to the client diagnosed with a myocardial infarction. Which assessment data would cause the nurse to question administering this medication? 1. The client's apical pulse is 64 2. The client's calcium level is elevated 3. The client's telemetry shows occasional PVCs 4. The client's blood pressure is 90/62

4

A nurse provides discharge instructions to a postoperative client who is taking warfarin sodium (Coumadin). Which statement, if made by the client, reflects the need for further teaching? 1. "I will take my pills every day at the same time." 2. "I will avoid alcohol consumption." 3. "I have already called my family to pick up a Medic-Alert bracelet." 4. "I will take Ecotrin (enteric-coated aspirin) for my headaches because it is coated."

4 Rationale: Ecotrin is an aspirin-containing product and should be avoided. Alcohol consumption should be avoided by a client taking warfarin sodium. Taking prescribed medication at the same time each day increases client compliance. The Medic-Alert bracelet provides health care personnel emergency information.

A client is on nicotinic acid (niacin) for hyperlipidemia and the nurse provides instructions to the client about the medication. Which statement by the client would indicate an understanding of the instructions? 1. "It is not necessary to avoid the use of alcohol." 2. "The medication should be taken with meals to decrease flushing. 3. "Clay-colored stools are a common side effect and should not be of concern." 4. "Ibuprofen (Motrin) taken 30 minutes before the nicotinic acid should decrease the flushing."

4 Rationale: Flushing is a side effect of this medication. Aspirin or a nonsteroidal anti-inflammatory drug can be taken 30 minutes prior to taking the medication to decrease flushing. Alcohol consumption needs to be avoided because it will enhance this side effect. The medication should be taken with meals but this will decrease gastrointestinal upset; taking the medication with meals has no effect on the flushing. Clay-colored stools are a sign of hepatic dysfunction and should be immediately reported to the physician.

A client is in sinus bradycardia with a heart rate of 45 beats/min, complains of dizziness, and has a blood pressure of 82/60 mm Hg. Which of the following should the nurse anticipate will be prescribed? 1. Defibrillate the client. 2. Administer digoxin (Lanoxin). 3. Continue to monitor the client. 4. Prepare for transcutaneous pacing

4 Rationale: Hypotension and dizziness are signs of decreased cardiac output. Transcutaneous pacing provides a temporary measure to increase the heart rate and thus perfusion in the symptomatic client. Digoxin will further decrease the client's heart rate. Defibrillation is used for treatment of pulseless ventricular tachycardia and ventricular fibrillation. Continuing to monitor the client delays necessary intervention

A client admitted to the hospital with chest pain and history of type 2 diabetes mellitus is scheduled for cardiac catheterization. Which of the following medications would need to be withheld for 48 hours before and after the procedure? 1. Regular insulin 2. Glipizide (Glucotrol) 3. Repaglinide (Prandin) 4. Metformin (Glucophage)

4 Rationale: Metformin (Glucophage) needs to be withheld 48 hours before and after cardiac catheterization because of the injection of contrast medium during the procedure. If the contrast medium affects kidney function, with metformin in the system, the client would be at increased risk for lactic acidosis. The medications in options 1, 2, and 3 do not need to be withheld 48 hours before or after cardiac catheterization.

A home health care nurse is visiting a client with elevated triglyceride levels and a serum cholesterol level of 398 mg/dL. The client is taking cholestyramine (Questran). Which of the following statements, if made by the client, indicates the need for further education? 1. "Constipation and bloating might be a problem." 2. "I'll continue to watch my diet and reduce my fats." 3. "Walking a mile each day will help the whole process." 4. "I'll continue my nicotinic acid from the health food store."

4 Rationale: Nicotinic acid, even an over-the-counter form, should be avoided because it may lead to liver abnormalities. All lipid-lowering medications also can cause liver abnormalities, so a combination of nicotinic acid and cholestyramine resin is to be avoided. Constipation and bloating are the two most common side effects. Walking and the reduction of fats in the diet are therapeutic measures to reduce cholesterol and triglyceride levels.

A client is having frequent premature ventricular contractions. A nurse would place priority on assessment of which of the following? 1. Sensation of palpitations 2. Causative factors, such as caffeine 3. Precipitating factors, such as infection 4. Blood pressure and oxygen saturation

4 Rationale: Premature ventricular contractions can cause hemodynamic compromise. The shortened ventricular filling time with the ectopic beat leads to decreased stroke volume and, if frequent enough, to decreased cardiac output. The client may be asymptomatic or may feel palpitations. Premature ventricular contractions can be caused by cardiac disorders, states of hypoxemia or by any number of physiological stressors, such as infection, illness, surgery, or trauma, and by intake of caffeine, nicotine, or alcohol

When assessing an ECG, the nurse knows that the P-R interval represents the time it takes for the: a. Impulse to begin atrial contraction b. Impulse to transverse the atria to the AV node c. SA node to discharge the impulse to begin atrial depolarization d. Impulse to travel to the ventricles

4. The P-R interval is measured on the ECG strip from the beginning of the P wave to the beginning of the QRS complex. It is the time it takes for the impulse to travel to the ventricle.

Which of the following cardiac conditions does a fourth heart sound (S4) indicate? 1. Dilated aorta 2. Normally functioning heart 3. Decreased myocardial contractility 4. Failure of the ventricle to eject all of the blood during systole

4. An S4 occurs as a result of increased resistance to ventricular filling after atrial contraction. The increased resistance is related to decreased compliance of the ventricle. A dilated aorta doesn't cause an extra heart sound, though it does cause a murmur. Decreased myocardial contractility is heard as a third heart sound. An S4 isn't heard in a normally functioning heart.

Which of the following is an expected outcome for a pt on the second day of hospitalization after an MI? 1. Has severe chest pain 2. Can identify risks factors for MI 3. Agrees to participate in a cardiac rehabilitation walking program 4. Can perform personal self-care activities without pain

4. By day 2 of hospitalization after an MI, pts are expected to be able to perform personal care w/out chest pain. Day 2 hospitalization may be too soon for pts to be able to identify risk factors for MI or begin a walking program; however, the pt may be sitting up in a chair as part of the cardiac rehabilitation program. Severe chest pain should not be present.

Which of the following blood gas abnormalities is initially most suggestive of pulmonary edema? 1. Anoxia 2. Hypercapnia 3. Hyperoxygenation 4. Hypocapnia

4. In an attempt to compensate for increased work of breathing due to hyperventilation, carbon dioxide decreases, causing hypocapnea. If the condition persists, CO2 retention occurs & hypercapnia results.

Which of the following conditions is the predominant cause of angina? 1. Increased preload 2. Decreased afterload 3. Coronary artery spasm 4. Inadequate oxygen supply to the myocardium

4. Inadequate oxygen supply to the myocardium is responsible for the pain accompanying angina. Increased preload would be responsible for right-sided heart failure. Decreased afterload causes increased cardiac output. Coronary artery spasm is responsible for variant angina.

Which of the following classes of meds maximizes cardiac performance in pts with heart failure by increasing ventricular contractibility? 1. Beta-adrenergic blockers 2. Calcium channel blockers 3. Diuretics 4. Inotropic agents

4. Inotropic agents are administered to increase the force of the heart's contractions, thereby increasing ventricular contractility & ultimately increasing cardiac output.

What is the primary reason for administering morphine to a pt with an MI? 1. To sedate the pt 2. To decrease the pt's pain 3. To decrease the pt's anxiety 4. To decrease oxygen demand on the pt's heart

4. Morphine is administered because it decreases myocardial oxygen demand. Morphine will also decrease pain & anxiety while causing sedation, but it isn't primarily given for those reasons.

A nurse caring for a pt in one room is told by another nurse that a second pt has developed severe pulmonary edema. On entering the 2nd pt's room, the nurse would expect the pt to be: 1. Slightly anxious 2. Mildly anxious 3. Moderately anxious 4. Extremely anxious

4. Pulmonary edema causes the pt to be extremely agitated & anxious. The pt may complain of a sense of drowning, suffocation, or smothering.

Which of the following symptoms is the most likely origin of pain the pt described as knifelike chest pain that increases in intensity with inspiration? 1. Cardiac 2. Gastrointestinal 3. Musculoskeletal 4. Pulmonary

4. Pulmonary pain is generally described by these symptoms. Musculoskeletal pain only increases w/ movement. Cardiac & GI pains don't change w/ respiration.

Which of the following tests is used most often to diagnose angina? 1. Chest x-ray 2. Echocardiogram 3. Cardiac catherization 4. 12-lead electrocardiogram (ECG)

4. The 12-lead ECG will indicate ischemia, showing T-wave inversion. In addition, w/ variant angina, the ECG shows ST-segment elevation. A chest x-ray will show heart enlargement or signs of heart failure, but isn't used to diagnose angina.

Which of the following is a compensatory response to decreased cardiac output? 1. Decreased BP 2. Alteration in LOC 3. Decreased BP & diuresis 4. Increased BP & fluid retention

4. The body compensates for a decrease in cardiac output w/ a rise in BP, due to the stimulation of the sympathetic NS & an increase in blood volume as the kidneys retain sodium & water. Blood pressure doesn't initially drop in response to the compensatory mechanism of the body. Alteration in LOC will occur only if the decreased cardiac output persists.

In which of the following disorders would the nurse expect to assess sacral edema in a bedridden pt? 1. Diabetes 2. Pulmonary emboli 3. Renal failure 4. Right-sided heart failure

4. The most accurate area on the body to assess dependent edema in a bed-ridden pt is the sacral area. Sacral, or dependent, edema is secondary to right-sided heart failure.

The nurse coming on duty receives the report from the nurse going off duty. Which of the following pts should the on-duty nurse assess first? 1. The 58-year-old pt who was admitted 2 days ago with heart failure, BP of 126/76, & a RR 21 bpm 2. The 88-year-old pt with end-stage right-sided heart failure, BP of 78/50, & a DNR order. 3. The 62-year-old pt who was admitted one day ago with thrombophlebitis & receiving IV heparin. 4. A 76-year-old pt who was admitted 1 hour ago with new-onset atrial fibrillation & is receiving IV diltiazem (Cardizem).

4. The pt w/ A-fib has the greatest potential to become unstable & is on IV med that requires close monitoring. After assessing this pt, the nurse should assess the pt w/ thrombophlebitis who is receiving a heparin infusion, & then go to the 58-year-old pt admitted 2-days ago w/ heart failure (her s/s are resolving & don't require immediate attention). The lowest priority is the 89-year-old w/ end stage right-sided heart failure, who requires time consuming supportive measures.

A pt who had cardiac surgery 24 hours ago has a urine output averaging 19 ml/hr for 2 hours. The pt received a single bolus of 500 ml of IV fluid. Urine output for the subsequent hour was 25 ml. Daily laboratory results indicate the blood urea nitrogen is 45 mg/dL & the serum creatinine is 2.2 mg/dL. A nurse interprets the pt is at risk for: 1. Hypovolemia 2. UTI 3. Glomerulonephritis 4. Acute renal failure

4. The pt who undergoes cardiac surgery is at risk for renal injury from poor perfusion, hemolysis, low cardiac output, or vasopressor med therapy. Renal insult is signaled by decreased urine output, & increased BUN & creatinine levels. The pt may need meds such as dopamine (Intropin) to increase renal perfusion & possibly could need peritoneal dialysis or hemodialysis.

Dyspnea, cough, expectoration, weakness, & edema are classic signs & symptoms of which of the following conditions? 1. Pericarditis 2. Hypertension 3. MI 4. Heart failure

4. These are the classic signs of failure. Pericarditis is exhibited by a feeling of fullness in the chest & auscultation of a pericardial friction rub. Hypertension is usually exhibited by headaches, visual disturbances, & a flushed face. MI causes heart failure but isn't related to these symptoms.

Which of the following types of angina is most closely related with an impending MI? 1. Angina decubitus 2. Chronic stable angina 3. Noctural angina 4. Unstable angina

4. Unstable angina progressively increases in frequency, intensity, & duration & is related to an increased risk of MI w/in 3 to 18 months.

A pt has frequent bursts of ventricular tachycardia on the cardiac monitor. A nurse is most concerned with this dysrhythmia because: 1. It is uncomfortable for the pt, giving a sense of impending doom. 2. It produces a high cardiac output that quickly leads to cerebral & myocardial ischemia. 3. It is almost impossible to convert to a normal sinus rhythm. 4. It can develop into ventricular fibrillation at any time.

4. Ventricular tachycardia is a life-threatening dysrhythmia that results from an irritable ectopic focus that takes over as the pacemaker for the heart. The low cardiac output that results can lead quickly to cerebral & myocardial ischemia. Pt's frequently experience a feeling of impending death. Ventricular tachycardia is treated w/ antidysrhythmic meds or magnesium sulfate, cardioversion (pt awake), or defibrillation (loss of consciousness), Ventricular tachycardia can deteriorate into ventricular defibrillation at any time.

Which of the following conditions is most closely associated with weight gain, nausea, & a decrease in urine output? 1. Angina pectoris 2. Cardiomyopathy 3. Left-sided heart failure 4. Right-sided heart failure

4. Weight gain, nausea, & a decrease in urine output are secondary effects of right-sided heart failure. Cardiomyopathy is usually identified as a symptom of left-sided heart failure. Left-sided heart failure causes primarily pulmonary symptoms rather than systemic ones. Angina pectoris doesn't cause weight gain, nausea, or a decrease in urine output.

A 50 y/o pt is prescribed to take nitrate each day for his condition. You know the result of nitrate administration is: A. decreased myocardial oxygen demand B. increased myocardial oxygen demand C. increased left ventricular end-diastolic volume D. increased atrial pressure

A

A client admitted to the hospital for chest pain is diagnosed with angina. The nurse should teach the client that the most common characteristic of anginal pain is that it is: A. Relieved by rest B. Precipitated by light activity C. Described by sharp or knifelike D. Unaffected by the administration of vasodilators

A

A client is brought to the emergency room with chest pain. The client asks why an ECG has been prescribed. The nurse explains that an ECG will: A. Aid in detecting heart damage B. Detect altered heart sounds C. Determine the flow of blood to the heart muscle D. Evaluate the spatial relationship of structures within the heart

A

A client who had several episodes of chest pain is scheduled for an exercise ECG. Which explanation should the nurse include when teaching the client about this procedure? A. "This is a noninvasive test to check your heart's response to physical activity". B. "This test is the definitive method to identify the actual cause of your chest pain". C. "The findings of this test will be of minimal assistance in the treatment of angina". D. "The findings from this minimally invasive test will show how your body reacts to exercise".

A

A client with a hemoglobin level of 6.2 g/dL is receiving packed red blood cells. 20 min after infusion starts, the client complains of chest pain, difficulty breathing, and feeling cold. What is the first action the nurse should take? A. Stop the transfusion B. Notify the healthcare provider C. Provide several warm blankets D. Slow down the rate of infusion

A

A few days after experiencing a myocardial infarction (MI) and successful percutaneous coronary intervention, the patient states, "I just had a little chest pain. As soon as I get out of here, I'm going for my vacation as planned." Which reply would be most appropriate for the nurse to make? a. "What do you think caused your chest pain?" b. "Where are you planning to go for your vacation?" c. "Sometimes plans need to change after a heart attack." d. "Recovery from a heart attack takes at least a few weeks."

A

A nurse is administering oxygen to a client with chest pain who is restless. What method of oxygen administration will most likely prevent further increase in the clients anxiety level? A. Cannula B. Catheter C. Venturi mask D. Rebreather mask

A

A nurse is assessing the neurovascular status of a client who returned to the surgical nursing unit 4 hours ago after undergoing aortoiliac bypass graft. The affected leg is warm, and the nurse notes redness and edema. The pedal pulse is palpable and unchanged from admission. The nurse interprets that the neurovascular status is: A. Normal because of the increased blood flow through the leg B. Slightly deteriorating and should be monitored for another hour C. Moderately impaired, and the surgeon should be called. D. Adequate from the arterial approach, but venous complications are arising.

A

A patient who has recently started taking pravastatin (Pravachol) and niacin (Nicobid) reports the following symptoms to the nurse. Which is most important to communicate to the health care provider? a. Generalized muscle aches and pains b. Dizziness when changing positions quickly c. Nausea when taking the drugs before eating d. Flushing and pruritus after taking the medications

A

A patient who is being admitted to the emergency department with intermittent chest pain gives the following list of medications to the nurse. Which medication has the most immediate implications for the patient's care? a. Sildenafil (Viagra) b. Furosemide (Lasix) c. Captopril (Capoten) d. Warfarin (Coumadin)

A

A patient who is in the 10th week of outpatient cardiac rehabilitation continues to exhibit symptoms of depression. When developing a discharge plan, the cardiac/vascular nurse includes: A. a referral for counseling and possible medication. B. a trial of herbal remedies. C. alternative therapies, including yoga and massage. D. comprehensive information about the patient's cardiac status to help reduce his or her anxiety.

A

A patient with chronic congestive heart failure and atrial fibrillation is treated with a digitalis preparation and a loop diuretic. To prevent possible complications of this combination of drugs, the nurse needs to A. monitor serum potassium levels. B. keep an accurate measure of intake and output. C. teach the patient about dietary restriction of potassium. D. withhold the digitalis and notify the health care provider if the heart rate is irregular.

A

A patient with infective endocarditis develops sudden left leg pain with pallor, paresthesia, and a loss of peripheral pulses. The nurse's initial action should be to A. notify the physician. B. elevate the leg to promote venous return. C. wrap the leg in a blanket to provide warmth. D. perform passive range of motion to stimulate circulation to the leg.

A

A patient with newly diagnosed hypertension has a blood pressure of 158/98 after 12 months of exercise and diet modifications. The nurse advises the patient that A. medication may be required because the BP is still not within the normal range. B. continued monitoring of the BP every 3 to 6 months is all that will be necessary for treatment. C. because lifestyle modifications were not effective they do not need to be continued and drugs will be used. D. he will have to make more vigorous changes in his lifestyle if he wants to stay off medication for his hypertension.

A

A patient with no history of heart disease is seen in the clinic for periodic episodes of tachycardia with a regular rhythm. When obtaining the patient's history, the nurse should question the patient regarding the incidence of A. stress. B. asthma. C. diabetes. D. weight gain.

A

A physical assessment finding that the nurse would expect to be present in the patient with acute left-sided heart failure is A. bubbling crackles and tachycardia. B. hepatosplenomegaly and tachypnea. C. peripheral edema and cool, diaphoretic skin. D. frothy blood-tinged sputum and distended jugular veins.

A

A physician writes the following admitting orders for a patient who has fever and chills and in whom infective endocarditis is suspected: ceftriaxone (Rocephin) 1.0 g IVPB every 12 hours, aspirin for temperature above 102° F (38.9° C), and blood cultures in triplicate, complete blood count, serum chemistries, and ECG. When admitting the patient, the nurse should give the highest priority to A. obtaining the blood cultures. B. scheduling the ECG. C. administering the antipyretic agent. D. initiating the IV antibiotic.

A

A pt has a potassium level of 6.0 and a digoxin level of 3.0. What medication would the nurse be giving: A. digibind B. sodium citrate C. epinephrine D. lidocaine

A

A pt is on Spironolactone (Aldactone) and has a potassium level of 5.9. What should the nurse do? A. hold the med and contact the physician B. give the med as prescribed C. give the pt half the prescribed dose D. give the pt a banana

A

A thoracic aortic aneurysm is found when a patient has a routine chest x-ray. The nurse anticipates that additional diagnostic testing to determine the size and structure of the aneurysm will include: a. CT scan B. angiography c. echocardiography d. ultrasound

A

A week after experiencing a ruptured cerebral aneurysm, if a patient becomes extremely indecisive and has frequent episodes of incontinence, it is probably caused by A. a vasospasm. B. automatisms. C. focal seizures. D. early-stage dementia.

A

After teaching about ways to decrease risk factors for CAD, the nurse recognizes that additional instruction is needed when the patient says, A. "I would like to add weight lifting to my exercise program." B. "I can't keep my blood pressure normal without medication." C. "I can change my diet to decrease my intake of saturated fats." D. "I will change my lifestyle to reduce activities that increase my stress."

A

An elderly pt is prescribed spironolactone (Aldactone) with the addition of potassium chloride (Kaochlor). Which statement is a true statement regarding the use of these two meds together? A. spironolactone should not have potassium chloride added to the regimen b/c it is a potassium-sparing diuretic B. potassium is necessary when clients are placed on spironlocatone b/c it is a loop diuretic C. spironolactone and potassium chloride have no additive or antagonistic effects with each other D. potassium chloride added to spironolactone causes renal failure

A

An important nursing responsibility for a patient having an invasive cardiovascular diagnostic study is A. checking the peripheral pulses and percutaneous site. B. instructing the patient about radioactive isotope injection. C. informing the patient that general anesthesia will be given. D. assisting the patient to do a surgical scrub of the insertion site.

A

Before a pt receives metoprolol (Lopressor) for hypertension, the nurse should ask the pt about a history of which of the following? A. bronchospasms B. seizures C. peripheral vascular disease D. myasthenia gravis

A

Four hours after the onset of pain from an MI, a nurse should expect an increase in the A. creatine kinase-MB (CK-MB). B. leukocyte count. C. alkaline phosphatase (ALP). D. lactate dehydrogenase (LHD).

A

Mike, a 43-year old construction worker, has a history of hypertension. He smokes two packs of cigarettes a day, is nervous about the possibility of being unemployed, and has difficulty coping with stress. His current concern is calf pain during minimal exercise that decreased with rest. The nurse assesses Mike's symptoms as being associated with peripheral arterial occlusive disease. The nursing diagnosis is probably: a. Alteration in tissue perfusion related to compromised circulation b. Dysfunctional use of extremities related to muscle spasms c. Impaired mobility related to stress associated with pain d. Impairment in muscle use associated with pain on exertion.

A

Raymund is reviewing on cardio drugs for his upcoming exam. For a well-prepared student, he should know that vasodilators are agents that: A. relax smooth muscle B. are used to treat hypotension C. stimulate the adrenergic receptors D. cause respiratory depression

A

Sublingual nitroglycerin is given to a client who experiences chest pain. Which symptom can occur with nitroglycerin? A. tachycardia B. tinnitus C. diarrhea D. Diplopia

A

The drug used in the management of a patient with acute pulmonary edema that will decrease both preload and afterload and provide relief of anxiety is A. morphine. B. amrinone. C. dobutamine. D. aminophylline.

A

The healthcare provider prescribes Nitroglycerin ointment to be applied topically every 8 hours for a client who was admitted for chest pain and a MI. Which statement, if made by the client, would indicate understanding of the side effects of nitroglycerin ointment? "I may experience: A. A headache B. Increased BP readings C. A slow Pulse rate D. Confusion

A

The nurse finds a 76-year-old client's point of maximum impulse farther away from the fifth intercostal space, midclavicular line. What does this finding suggest to the nurse? a. The client has hypertension. b. The client had pneumonia. c. The client has benign prostatic hypertrophy. d. The client had a heart attack.

A

The nurse is caring for a patient who was admitted to the coronary care unit following an acute myocardial infarction (AMI) and percutaneous coronary intervention the previous day. Teaching for this patient would include a. when cardiac rehabilitation will begin. b. the typical emotional responses to AMI. c. information regarding discharge medications. d. the pathophysiology of coronary artery disease.

A

The nurse notes a client's PR interval upon ECG is greater than 0.20 seconds. What can this information suggest to the nurse? a. The client's sinoatrial node is not pacing correctly. b. There is an elongated ventricular depolarization. c. It takes longer for the blood to leave the lungs and return to the heart. d. The client elongated ventricular is filling.

A

The primary causes of death in patients with heart transplants in the first year include A. infection and rejection. B. rejection and arrhythmias. C. arrhythmias and infection. D. myocardial infarction and lymphoma.

A

Three days after experiencing a myocardial infarction (MI), a patient who is scheduled for discharge asks for assistance with hygiene activities, saying, "I am too nervous to take care of myself." Based on this information, which nursing diagnosis is appropriate? a. Ineffective coping related to anxiety b. Activity intolerance related to weakness c. Denial related to lack of acceptance of the MI d. Disturbed personal identity related to understanding of illness

A

When administering hydrochlorothiazide, which labs must be monitored? A. BUN, creatinine B. INR, electrolytes C. APPT, PTT

A

When applying a nitro patch it should be placed on A. non hairy part of the chest B. thigh C. upper back D. buttocks

A

When applying a nitro patch it should be placed on A. non hairy part of the chest B. thigh C. upper back D. buttocks

A

When assessing a patient with arterial insufficiency, the nurse should expect A. tissue atrophy. B. thin, fragile toenails. C. warm, erythematous legs. D. bounding arterial pulses

A

When assessing the patient, the nurse notes a palpable precordial thrill. This finding may be caused by A. heart murmurs. B. gallop rhythms. C. pulmonary edema. D. right ventricular hypertrophy.

A

When caring for a patient who has just arrived on the medical-surgical unit after having cardiac catheterization, which nursing intervention should the nurse delegate to a licensed practical/vocational nurse (LPN/LVN)? a. Give the scheduled aspirin and lipid-lowering medication. b. Perform the initial assessment of the catheter insertion site. c. Teach the patient about the usual postprocedure plan of care. d. Titrate the heparin infusion according to the agency protocol.

A

When providing discharge instructions to a patient with a new permanent pacemaker, the nurse teaches the patient to A. take and record a daily pulse rate. B. request special hand scanning at airport and other security gates. C. immobilize the arm and shoulder on the side of the pacemaker insertion for 6 weeks. D. avoid microwave ovens because they emit radio waves that alter pacemaker function.

A

When teaching about nitrate administration, the nurse should instruct the pt to: A. change positions slowly B. take pulse weekly C. reduce salt intake D. chew the sustained-release tablets

A

Which drug category has intermittent claudication as a characteristic side effect? A. peripheral vasodilators B. diuretics C. antilipidemics

A

Which information about a patient who has been receiving thrombolytic therapy for an acute myocardial infarction (AMI) is most important for the nurse to communicate to the health care provider? a. No change in the patient's chest pain b. An increase in troponin levels from baseline c. A large bruise at the patient's IV insertion site d. A decrease in ST-segment elevation on the electrocardiogram

A

Which is the most important action for the nurse to take before administering digoxin? A. monitor the potassium level B. assess the BP C. evaluate urinary output D. avoid giving with a thiazide diuretic

A

Which of the following must be given within 3 hours of the beginning of an MI or stroke? A. thrombolytics B. antiplatelets C. cardiac glycosides

A

Which statement by the pt reflects the need for additional pt education about the CCB diltiazem (Cardizem)? A. I can take this drug to stop acute anginal attacks B. I understand that food and antacids alter the absorption of this oral drug C. when the long-acting forms are taken, the drug cannot be crushed D. this drug may cause my blood pressure to drop, so I should be careful when getting up

A

Your patient is diagnosed with peripheral arterial disease and asks you what this disease is and what causes it? Which response is most appropriate? a) PAD is the thickening of arterial walls that results in gradual narrowing. This thickening and narrowing is related to arterial wall damage and inflammation. b) PAD is the thinning of arterial walls that results in gradual weakening. This weakening is related to high blood pressure. c) Let me get your doctor to explain this to you. d) PAD is the hardening of the arterial walls that results in thickening and eventual closure of the vessel.

A

Your pt starts showing signs of gynecomastia, which diuretic would you suspect they are on? A. potassium-sparing B. thiazide C. loop D. calcium channel blockers

A

a pt has a new prescription for an adrenergic drug. During a review of the pt's list of current meds, which would cause concern about a possible interaction with this new prescription: A. a benzodiazepine taken as needed for allergies B. a multivitamin with iron C. an oral anticoagulant D. NSAIDS

A

beta blockers should be avoided in which of the following conditions: A. bronchoconstriction B. hypertension C. Angina D. myocardial infarction

A

nitroglycerin is ordered for an elderly pt who is having an episode of chest pain. Which is a correct statement regarding the use of nitroglycerin spray instead of nitroglycerin tablets in this age group? A. there is increased absorption in the older client with the spray B. older clients attempt to chew the tablet C. tablets dissolve slower in this population D. the spray lasts longer in the system

A

potassium sparing diuretics work where? A. collecting distal tubules B. ascending loop of henle C. descending loop of henle

A

when giving antihypertensives drugs, the nurse must consider giving the first dose at bedtime for which of the following classes of drugs? A. alpha blockers B. diuretics C. ACE inhibitors D. vasodilators

A

A patient who has experienced atrial fibrillation for the past 3 days is admitted to the cardiac care unit. In addition to administering an antidysrhythmia medication, the healthcare provider should anticipate which of these orders? A. Initiate a heparin infusion B. Give atropine IV push C. Prepare for immediate cardioversion D. Prepare the patient for AV node ablation

A Because blood tends to pool and clot in the fibrillating atria, patients with atrial fibrillation are at high risk for embolic stroke, so heparin will be given.

A patient with a recent diagnosis of heart failure has been prescribed furosemide (Lasix) in an effort to physiologically do what for the patient? a. Reduce preload. b. Decrease afterload. c. Increase contractility. d. Promote vasodilation.

A Diuretics such as furosemide are used in the treatment of HF to mobilize edematous fluid, reduce pulmonary venous pressure, and reduce preload. They do not directly influence afterload, contractility, or vessel tone.

A key aspect of teaching for the patient on anticoagulant therapy includes which instructions? a. Monitor for and report any signs of bleeding. b. Do not take acetaminophen (Tylenol) for a headache. c. Decrease your dietary intake of foods containing vitamin K. d. Arrange to have blood drawn routinely to check drug levels.

A Rationale: Patients taking anticoagulants should be taught to monitor and report any signs of bleeding, which can be a serious complication. Other important patient teaching includes maintenance of a consistent intake of foods containing vitamin K, avoidance of supplements that contain vitamin K, and routine coagulation laboratory studies if a patient is taking warfarin.

When the patient is being examined for venous thromboembolism (VTE) in the calf, what diagnostic test should the nurse expect to teach the patient about first? a Duplex ultrasound b Contrast venography c Magnetic resonance venography d Computed tomography venography

A Rationale: The duplex ultrasound is the most widely used test to diagnose VTE. Contrast venography is rarely used now. Magnetic resonance venography is less accurate for calf veins than pelvic and proximal veins. Computed tomography venography may be used but is invasive and much more expensive than the duplex ultrasound.

The patient with chronic heart failure is being discharged from the hospital. What information should the nurse emphasize in the patient's discharge teaching to prevent progression of the disease to ADHF? a. Take medications as prescribed. b. Use oxygen when feeling short of breath. c. Only ask the physician's office questions. d. Encourage most activity in the morning when rested.

A The goal for the patient with chronic HF is to avoid exacerbations and hospitalization. Taking the medications as prescribed along with nondrug therapies such as alternating activity with rest will help the patient meet this goal. If the patient needs to use oxygen at home, it will probably be used all the time or with activity to prevent respiratory acidosis. Many HF patients are monitored by a care manager or in a transitional program to assess the patient for medication effectiveness and monitor for patient deterioration and encourage the patient. This nurse manager can be asked questions or can contact the health care provider if there is evidence of worsening HF.

A client with post-myocardial infarction develops acute bacterial pericarditis. Which of the following medications would the physician most likely prescribe as the primary drug? Ticarcillin disodium (Ticar) Acetaminophen (Tylenol) Ibuprofen (Motrin) Trioxsalen (Trisoralen)

A Rationale: Acute bacterial pericarditis is a complication that can occur post-myocardial infarction. Acute bacterial pericarditis usually requires antibiotics. NSAIDs usually are prescribed to relieve pain from the inflammatory process. If the NSAIDs do not relieve pain within 48 to 96 hours, corticosteroids are ordered. There is no mention of pain in the stem of the question. Trisoralen is used to repigment skin for persons with vitiligo.

It is important that the nurse be knowledgeable about cardiac output in order to: Evaluate blood flow to peripheral tissues. Determine the electrical activity of the myocardium. Provide information on the immediate need for oxygen. Implement nutritional changes.

A Rationale: Blood flow to the tissues is measured clinically as the cardiac output, and assists to predict tissue perfusion. Electrical activity is evaluated more effectively by EKG. While the cardiac output is important for perfusion and oxygenation of tissues, the oxygen saturation would provide more valuable information. Nutritional changes would be targeted to sodium and would depend on symptoms of disease. Integrated Process: Nursing Process; Planning Cognitive Level: Evaluation NCLEX-RN Test Plan: Health Promotion and Maintenance

Which of the following assessments would be an important finding for a patient with arterial disease? Intermittent claudication with exercise Brownish discoloration around the ankles Non-pitting edema on the lower extremities Altered sensation to touch

A Rationale: Intermittent claudication is a common finding in persons with arterial disease, usually due to progression of atherosclerosis and alteration of tissue perfusion to the extremities. In venous disease, valves of the veins in the extremities become incompetent, resulting in higher pressures than normal in the veins. The pressure is transmitted to the capillaries of the lower extremities, resulting in thickening and non-pitting edema of tissues around the ankles. Prolonged thickening results in the red blood cells' being pressed outside the capillaries. The cells eventually break down, resulting in collection of hemosiderin deposits being collected in the area. Altered sensation to touch would be due to neuropathic changes commonly found with diabetes mellitus.

Which of the following instructions should be included in the discharge teaching for a patient discharged with a transdermal nitroglycerin patch? a. "Apply the patch to a nonhairy, nonfatty area of the upper torso or arms." b. "Apply the patch to the same site each day to maintain consistent drug absorption." c. "If you get a headache, remove the patch for 4 hours and then reapply." d. "If you get chest pain, apply a second patch right next to the first patch."

A A nitroglycerin patch should be applied to a nonhairy, nonfatty area for the best and most consistent absorption rates. Sites should be rotated to prevent skin irritation, and the drug should be continued if headache occurs because tolerance will develop. Sublingual nitroglycerin should be used to treat chest pain.

Which of the following sounds is distinctly heard on auscultation over the abdominal region of an abdominal aortic aneurysm client? A. Bruit B. Crackles C. Dullness D. Friction rubs

A A bruit, a vascular sound resembling heart murmur, suggests partial arterial occlusion. Crackles are indicative of fluid in the lungs. Dullness is heard over solid organs, such as the liver. Friction rubs indicate inflammation of the peritoneal surface.

Which of the following instructions should be included in the discharge teaching for a patient discharged with a transdermal nitroglycerin patch? A. "Apply the patch to a non hairy, nonfatty area of the upper torso or arms." B. "Apply the patch to the same site each day to maintain consistent drug absorption." C. "If you get a headache, remove the patch for 4 hours and then reapply." D. "If you get chest pain, apply a second patch right next to the first patch.

A A nitroglycerin patch should be applied to a non hairy, nonfatty area for the best and most consistent absorption rates. Sites should be rotated to prevent skin irritation, and the drug should be continued if headache occurs because tolerance will develop. Sublingual nitroglycerin should be used to treat chest pain.

What is the most common symptom in a client with abdominal aortic aneurysm? A. Abdominal pain B. Diaphoresis C. Headache D. Upper back pain

A Abdominal pain in a client with an abdominal aortic aneurysm results from the disruption of normal circulation in the abdominal region. Lower back pain, not upper, is a common symptom, usually signifying expansion and impending rupture of the aneurysm. Headache and diaphoresis aren't associated with abdominal aortic aneurysm.

Which of the following terms describes the force against which the ventricle must expel blood? a. Afterload b. Cardiac output c. Overload d. Preload

A Afterload refers to the resistance normally maintained by the aortic and pulmonic valves, the condition and tone of the aorta, and the resistance offered by the systemic and pulmonary arterioles. Cardiac output is the amount of blood expelled from the heart per minute. Overload refers to an abundance of circulating volume. Preload is the volume of blood in the ventricle at the end of diastole.

A 60-year-old male client comes into the emergency department with complaints of crushing chest pain that radiates to his shoulder and left arm. The admitting diagnosis is acute myocardial infarction. Immediate admission orders include oxygen by NC at 4L/minute, blood work, chest x-ray, an ECG, and 2 mg of morphine given intravenously. The nurse should first: A. Administer the morphine B. Obtain a 12-lead ECG C. Obtain the lab work D. Order the chest x-ray

A Although obtaining the ECG, chest x-ray, and blood work are all important, the nurse's priority action would be to relieve the crushing chest pain.

A 60-year-old male client comes into the emergency department with complaints of crushing chest pain that radiates to his shoulder and left arm. The admitting diagnosis is acute myocardial infarction. Immediate admission orders include oxygen by NC at 4L/minute, blood work, chest x-ray, an ECG, and 2mg of morphine given intravenously. The nurse should first: a. Administer the morphine b. Obtain a 12-lead ECG c. Obtain the lab work d. Order the chest x-ray

A Although obtaining the ECG, chest x-ray, and blood work are all important, the nurse's priority action would be to relieve the crushing chest pain.

Which of the following types of cardiomyopathy can be associated with childbirth? A. Dilated B. Hypertrophic C. Myocarditis D. Restrictive

A Although the cause isn't entirely known, cardiac dilation and heart failure may develop during the last month of pregnancy of the first few months after birth. The condition may result from a preexisting cardiomyopathy not apparent prior to pregnancy. Hypertrophic cardiomyopathy is an abnormal symmetry of the ventricles that has an unknown etiology but a strong familial tendency. Myocarditis isn't specifically associated with childbirth. Restrictive cardiomyopathy indicates constrictive pericarditis; the underlying cause is usually myocardial.

Which of the following parameters should be checked before administering digoxin? A. Apical pulse B. Blood pressure C. Radial pulse D. Respiratory rate

A An apical pulse is essential or accurately assessing the client's heart rate before administering digoxin. The apical pulse is the most accurate point in the body. Blood pressure is usually only affected if the heart rate is too low, in which case the nurse would withhold digoxin. The radial pulse can be affected by cardiac and vascular disease and therefore, won't always accurately depict the heart rate. Digoxin has no effect on respiratory function.

A nurse is assessing the neurovascular status of a client who returned to the surgical nursing unit 4 hours ago after undergoing aortoiliac bypass graft. The affected leg is warm, and the nurse notes redness and edema. The pedal pulse is palpable and unchanged from admission. The nurse interprets that the neurovascular status is: a. Normal because of the increased blood flow through the leg b. Slightly deteriorating and should be monitored for another hour c. Moderately impaired, and the surgeon should be called. d. Adequate from the arterial approach, but venous complications are arising.

A An expected outcome of surgery is warmth, redness, and edema in the surgical extremity because of increased blood flow. Options 2, 3, and 4 are incorrect interpretations.

What is the most common cause of abdominal aortic aneurysm? A. Atherosclerosis B. DM C. .HPN D. Syphilis

A Atherosclerosis accounts for 75% of all abdominal aortic aneurysms. Plaques build up on the wall of the vessel and weaken it, causing an aneurysm. Although the other conditions are related to the development of an aneurysm, none is a direct cause.

Which of the following conditions most commonly results in CAD? A. Atherosclerosis B.DM C. MI D. Renal failure

A Atherosclerosis, or plaque formation, is the leading cause of CAD. DM is a risk factor for CAD but isn't the most common cause. Renal failure doesn't cause CAD, but the two conditions are related. Myocardial infarction is commonly a result of CAD.

Baroreceptors in the carotid artery walls and aorta respond to which of the following conditions? a. Changes in blood pressure b. Changes in arterial oxygen tension c. Changes in arterial carbon dioxide tension d. Changes in heart rate

A Baroreceptors located in the carotid arteries and aorta sense pulsatile pressure. Decreases in pulsatile pressure cause a reflex increase in heart rate. Chemoreceptors in the medulla are primarily stimulated by carbon dioxide. Peripheral chemoreceptors in the aorta and carotid arteries are primarily stimulated by oxygen.

The nurse teaches the client with angina about the common expected side effects of nitroglycerin, including: a. Headache b. High blood pressure c. Shortness of breath d. Stomach cramps

A Because of the widespread vasodilating effects, nitroglycerin often produces such side effects as headache, hypotension, and dizziness. The client should lie or sit down to avoid fainting. Nitro does not cause shortness of breath or stomach cramps.

Which of the following recurring conditions most commonly occurs in clients with cardiomyopathy? A. Heart failure B. DM C. MI D. Pericardial effusion

A Because the structure and function of the heart muscle is affected, heart failure most commonly occurs in clients with cardiomyopathy. Myocardial infarction results from prolonged myocardial ischemia due to reduced blood flow through one of the coronary arteries. Pericardial effusion is most predominant in clients with percarditis. Diabetes mellitus is unrelated to cardiomyopathy.

Which of the following classes of medications protects the ischemic myocardium by blocking catecholamines and sympathetic nerve stimulation? A. Beta-adrenergic blockers B. Calcium channel blockers C. Narcotics D. Nitrates

A Beta-adrenergic blockers work by blocking beta receptors in the myocardium, reducing the response to catecholamines and sympathetic nerve stimulation. They protect the myocardium, helping to reduce the risk of another infarction by decreasing the workload of the heart and decreasing myocardial oxygen demand. Calcium channel blockers reduce the workload of the heart by decreasing the heart rate. Narcotics reduce myocardial oxygen demand, promote vasodilation, and decreased anxiety. Nitrates reduce myocardial oxygen consumption by decreasing left ventricular end-diastolic pressure (preload) and systemic vascular resistance (afterload).

When assessing a newly admitted patient, the nurse notes a thrill along the left sternal border. To obtain more information about the cause of the thrill, which action will the nurse take next? a. Auscultate for any cardiac murmurs. b. Find the point of maximal impulse. c. Compare the apical and radial pulse rates. d. Palpate the quality of the peripheral pulses.

A Both thrills and murmurs are caused by turbulent blood flow, such as occurs when blood flows through a damaged valve. Relevant information includes the quality of the murmur, where in the cardiac cycle the murmur is heard, and where on the thorax the murmur is heard best. The other information also is important in the cardiac assessment but will not provide information that is relevant to the thrill.

What is the term used to describe an enlargement of the heart muscle? A. Cardiomegaly B. Cardiomyopathy C. Myocarditis D. Pericarditis

A Cardiomegaly denotes an enlarged heart muscle. Cardiomyopathy is a heart muscle disease of unknown origin. Myocarditis refers to inflammation of heart muscle. Pericarditis is an inflammation of the pericardium, the sac surrounding the heart.

Which of the following heart muscle diseases is unrelated to other cardiovascular disease? A. Cardiomyopathy B. Coronary artery disease C. Myocardial infarction D. Pericardial Effusion

A Cardiomyopathy isn't usually related to an underlying heart disease such as atherosclerosis. The etiology in most cases is unknown. Coronary artery disease and myocardial infarction are directly related to atherosclerosis. Pericardial effusion is the escape of fluid into the pericardial sac, a condition associated with pericarditis and advanced heart failure.

A client with rapid rate atrial fibrillation asks a nurse why the physician is going to perform carotid massage. The nurse responds that this procedure may stimulate the: A. Vagus nerve to slow the heart rate B. Vagus nerve to increase the heart rate; overdriving the rhythm C. Diaphragmatic nerve to slow the heart rate D. Diaphragmatic nerve to overdrive the rhythm

A Carotid sinus massage is one of the maneuvers used for vagal stimulation to decrease a rapid heart rate and possibly terminate a tachydysrhythmia. The others include inducing the gag reflex and asking the client to strain or bear down. Medication therapy often is needed as an adjunct to keep the rate down or maintain the normal rhythm.

Which information about a patient who has been receiving fibrinolytic therapy for an acute myocardial infarction (AMI) is most important for the nurse to communicate to the health care provider? a. No change in the patient's chest pain b. A large bruise at the patient's IV insertion site c. A decrease in ST segment elevation on the electrocardiogram (ECG) d. An increase in cardiac enzyme levels since admission

A Continued chest pain suggests that the fibrinolytic therapy is not effective and that other interventions such as percutaneous coronary intervention (PCI) may be needed. Bruising is a possible side effect of fibrinolytic therapy, but it is not an indication that therapy should be discontinued. The decrease of the ST segment elevation indicates that fibrinolysis is occurring and perfusion is returning to the injured myocardium. An increase in cardiac enzyme levels is expected with reperfusion and is related to the washout of enzymes into the circulation as the blocked vessel is opened.

Which of the following symptoms is most commonly associated with left-sided heart failure? A. Crackles B. Arrhythmias C. Hepatic engorgement D. Hypotension

A Crackles in the lungs are a classic sign of left-sided heart failure. These sounds are caused by fluid backing up into the pulmonary system. Arrhythmias can be associated with both right and left-sided heart failure. Left-sided heart failure causes hypertension secondary to an increased workload on the system.

A client enters the ER complaining of chest pressure and severe epigastric distress. His VS are 158/90, 94, 24, and 99*F. The doctor orders cardiac enzymes. If the client were diagnosed with an MI, the nurse would expect which cardiac enzyme to rise within the next 3 to 8 hours? a. Creatine kinase (CK or CPK) b. Lactic dehydrogenase (LDH) c. LDH-1 d. LDH-2

A Creatine kinase (CK, formally known as CPK) rises in 3-8 hours if an MI is present. When the myocardium is damaged, CPK leaks out of the cell membranes and into the blood stream. Lactic dehydrogenase rises in 24-48 hours, and LDH-1 and LDH-2 rises in 8-24 hours.

Good dental care is an important measure in reducing the risk of endocarditis. A teaching plan to promote good dental care in a client with mitral stenosis should include demonstration of the proper use of: A. A manual toothbrush B. An electric toothbrush C. An irrigation device D. Dental floss

A Daily dental care and frequent checkups by a dentist who is informed about the client's condition are required to maintain good oral health. Use of an electric toothbrush, an irrigation device, or dental floss may cause gums to bleed and allow bacteria to enter mucous membranes and the bloodstream, increasing the risk of endocarditis.

A 45-year-old male client with leg ulcers and arterial insufficiency is admitted to the hospital. The nurse understands that leg ulcers of this nature are usually caused by: a. Decreased arterial blood flow secondary to vasoconstriction b. Decreased arterial blood flow leading to hyperemia c. Atherosclerotic obstruction of the arteries d. Trauma to the lower extremities

A Decreased arterial flow is a result of vasospasm. The etiology is unknown. It is more problematic in colder climates or when the person is under stress. Hyperemia occurs when the vasospasm is relieved.

A nurse notes 2+ bilateral edema in the lower extremities of a client with myocardial infarction who was admitted 2 days ago. The nurse would plan to do which of the following next? Review the intake and output records for the last 2 days Change the time of diuretic administration from morning to evening Request a sodium restriction of 1 g/day from the physician. Order daily weights starting the following morning.

A Edema, the accumulation of excess fluid in the interstitial spaces, can be measured by intake greater than output and by a sudden increase in weight. Diuretics should be given in the morning whenever possible to avoid nocturia. Strict sodium restrictions are reserved for clients with severe symptoms.

A nurse notes 2+ bilateral edema in the lower extremities of a client with myocardial infarction who was admitted 2 days ago. The nurse would plan to do which of the following next? A. Review the intake and output records for the last 2 days B. Change the time of diuretic administration from morning to evening C. Request a sodium restriction of 1 g/day from the physician D. Order daily weights starting the following morning

A Edema, the accumulation of excess fluid in the interstitial spaces, can be measured by intake greater than output and by a sudden increase in weight. Diuretics should be given in the morning whenever possible to avoid nocturia. Strict sodium restrictions are reserved for clients with severe symptoms.

Hypertension is known as the silent killer. This phrase is associated with the fact that hypertension often goes undetected until symptoms of other system failures occur. This may occur in the form of: a. Cerebrovascular accident b. Liver disease c. Myocardial infarction d. Pulmonary disease

A Hypertension is referred to as the silent killer for adults, because until the adult has significant damage to other systems, the hypertension may go undetected. CVA's can be related to long-term hypertension. Liver or pulmonary disease is generally not associated with hypertension. Myocardial infarction is generally related to coronary artery disease.

A patient who has recently started taking rosuvastatin (Crestor) and niacin (Nicobid) reports all the following symptoms to the nurse. Which is most important to communicate to the health care provider? a. Generalized muscle aches and pains b. Skin flushing after taking the medications c. Dizziness when changing positions quickly d. Nausea when taking the drugs before eating

A Muscle aches and pains may indicate myopathy and rhabdomyolysis, which have caused acute renal failure and death in some patients who have taken the statin medications. These symptoms indicate that the rosuvastatin may need to be discontinued. The other symptoms are common side effects when taking niacin, and although the nurse should follow up with the patient, they do not indicate that a change in medication is needed.

Toxicity from which of the following medications may cause a client to see a green halo around lights? A. Digoxin B. Furosemide C. Metoprolol D. Enalapril

A One of the most common signs of digoxin toxicity is the visual disturbance known as the green halo sign. The other medications aren't associated with such an effect.

After the nurse teaches the patient about the use of atenolol (Tenormin) in preventing anginal episodes, which statement by a patient indicates that the teaching has been effective? a. "It is important not to suddenly stop taking the atenolol." b. "Atenolol will increase the strength of my heart muscle." c. "I can expect to feel short of breath when taking atenolol." d. "Atenolol will improve the blood flow to my coronary arteries."

A Patients who have been taking -blockers can develop intense and frequent angina if the medication is suddenly discontinued. Atenolol (Tenormin) decreases myocardial contractility. Shortness of breath that occurs when taking -blockers for angina may be due to bronchospasm and should be reported to the health care provider. Atenolol works by decreasing myocardial oxygen demand, not by increasing blood flow to the coronary arteries.

A client is having frequent premature ventricular contractions. A nurse would place priority on assessment of which of the following items? A. Blood pressure and peripheral perfusion B. Sensation of palpitations C. Causative factors such as caffeine D. Precipitating factors such as infection

A Premature ventricular contractions can cause hemodynamic compromise. The shortened ventricular filling time with the ectopic beats leads to decreased stroke volume and, if frequent enough, to decreased cardiac output. The client may be asymptomatic or may feel palpitations. PVCs can be caused by cardiac disorders or by any number of physiological stressors, such as infection, illness, surgery, or trauma, and by the intake of caffeine, alcohol, or nicotine.

Because a client has mitral stenosis and is a prospective valve recipient, the nurse preoperatively assesses the client's past compliance with medical regimens. Lack of compliance with which of the following regimens would pose the greatest health hazard to this client? A. Medication therapy B. Diet modification C. Activity Restrictions D. Dental care

A Preoperatively, anticoagulants may be prescribed for the client with advanced valvular heart disease to prevent emboli. Post-op, all clients with mechanical valves and some with bioprostheses are maintained indefinitely on anticoagulation therapy. Adhering strictly to a dosage schedule and observing specific precautions are necessary to prevent hemorrhage or thromboembolism. Some clients are maintained on lifelong antibiotic prophylaxis to prevent recurrence from rheumatic fever. Episodic prophylaxis is required to prevent infective endocarditis after dental procedures or upper respiratory, GI, or GU surgery.

When teaching a client about propranolol hydrochloride, the nurse should base the information on the knowledge that propranolol hydrochloride: a. Blocks beta-adrenergic stimulation and thus causes decreased heart rate, myocardial contractility, and conduction. b. Increases norepinephrine secretion and thus decreases blood pressure and heart rate. c. Is a potent arterial and venous vasodilator that reduces peripheral vascular resistance and lowers blood pressure. d. Is an angiotensin-converting enzyme inhibitor that reduces blood pressure by blocking the conversion of angiotensin I to angiotensin II.

A Propranolol hydrochloride is a beta-adrenergic blocking agent. Actions of propranolol hydrochloride include reducing heart rate, decreasing myocardial contractility, and slowing conduction.

Which of the following complications is indicated by a third heart sound (S3)? A. Ventricular dilation B. Systemic hypertension C. Aortic valve malfunction D. Increased atrial contractions

A Rapid filling of the ventricles causes vasodilation that is auscultated as S3. Increased atrial contraction or systemic hypertension can result is a fourth heart sound. Aortic valve malfunction is heard as a murmur.

Older clients experiencing anginal pain with complaints of fatigue or weakness usually are medicated with which of the following types of medication? Sublingual nitroglycerin Cardiac glycosides HMG-CoA reductase inhibitors Morphine sulfate

A Rationale: Angina frequently is managed with sublingual nitroglycerin, which causes vasodilation and increases blood flow to the coronary arteries. Cardiac glycosides are used to treat heart failure, and morphine is used to treat myocardial infarction. The HMG-CoA reductase inhibitors are used for patients with type 2 diabetes mellitus.

A patient in the intensive care unit with ADHF complains of severe dyspnea and is anxious, tachypneic, and tachycardic. All these medications have been ordered for the patient. The first action by the nurse will be to a. administer IV morphine sulfate 2 mg. b. give IV diazepam (Valium) 2.5 mg. c. increase dopamine (Intropin) infusion by 2 mcg/kg/min. d. increase nitroglycerin (Tridil) infusion by 5 mcg/min.

A Rationale: Morphine improves alveolar gas exchange, improves cardiac output by reducing ventricular preload and afterload, decreases anxiety, and assists in reducing the subjective feeling of dyspnea. Diazepam may decrease patient anxiety, but it will not improve the cardiac output or gas exchange. Increasing the dopamine may improve cardiac output but will also increase the heart rate and myocardial oxygen consumption. Nitroglycerin will improve cardiac output and may be appropriate for this patient, but it will not directly reduce anxiety and will not act as quickly as morphine to decrease dyspnea

A patient with ADHF who is receiving nesiritide (Natrecor) asks the nurse how the medication will work to help improve the symptoms of dyspnea and orthopnea. The nurse's reply will be based on the information that nesiritide will a. dilate arterial and venous blood vessels, decreasing ventricular preload and afterload. b. improve the ability of the ventricular myocardium to contract, strengthening contractility. c. enhance the speed of impulse conduction through the heart, increasing the heart rate. d. increase calcium sensitivity in vascular smooth muscle, boosting systemic vascular resistance.

A Rationale: Nesiritide, a recombinant form of BNP, causes both arterial and venous vasodilation, leading to reductions in preload and afterload. Inotropic medications, such as dopamine and dobutamine, may be used in ADHF to improve ventricular contractility. Nesiritide does not increase impulse conduction or calcium sensitivity in the heart.

An outpatient who has developed heart failure after having an acute myocardial infarction has a new prescription for carvedilol (Coreg). After 2 weeks, the patient returns to the clinic. The assessment finding that will be of most concern to the nurse is that the patient a. has BP of 88/42. b. has an apical pulse rate of 56. c. complains of feeling tired. d. has 2+ pedal edema.

A Rationale: The patient's BP indicates that the dose of carvedilol may need to be decreased because the mean arterial pressure is only 57. Bradycardia is a frequent adverse effect of -Adrenergic blockade, but the rate of 56 is not as great a concern as the hypotension. -adrenergic blockade will initially worsen symptoms of heart failure in many patients, and patients should be taught that some increase in symptoms, such as fatigue and edema, is expected during the initiation of therapy with this class of drugs.

A patient with a history of chronic heart failure is admitted to the emergency department with severe dyspnea and a dry, hacking cough. The patient has pitting edema in both ankles, blood pressure (BP) of 170/100, an apical pulse rate of 92, and respirations 28. The most important assessment for the nurse to accomplish next is to a. auscultate the lung sounds. b. assess the orientation. c. check the capillary refill. d. palpate the abdomen.

A Rationale: When caring for a patient with severe dyspnea, the nurse should use the ABCs to guide initial care. This patient's severe dyspnea and cough indicate that acute decompensated heart failure (ADHF) is occurring. ADHF usually manifests as pulmonary edema, which should be detected and treated immediately to prevent ongoing hypoxemia and cardiac/respiratory arrest. The other assessments will provide useful data about the patient's volume status and should also be accomplished rapidly, but detection (and treatment) of fluid-filled alveoli is the priority.

Following a treadmill test and cardiac catheterization, the client is found to have coronary artery disease, which is inoperative. He is referred to the cardiac rehabilitation unit. During his first visit to the unit he says that he doesn't understand why he needs to be there because there is nothing that can be done to make him better. The best nursing response is: a. "Cardiac rehabilitation is not a cure but can help restore you to many of your former activities." b. "Here we teach you to gradually change your lifestyle to accommodate your heart disease." c. "You are probably right but we can gradually increase your activities so that you can live a more active life." d. "Do you feel that you will have to make some changes in your life now?"

A Such a response does not have false hope to the client but is positive and realistic. The answer tells the client what cardiac rehabilitation is and does not dwell upon his negativity about it.

Following a treadmill test and cardiac catheterization, the client is found to have coronary artery disease, which is inoperative. He is referred to the cardiac rehabilitation unit. During his first visit to the unit he says that he doesn't understand why he needs to be there because there is nothing that can be done to make him better. The best nursing response is: A. "Cardiac rehabilitation is not a cure but can help restore you to many of your former activities." B. "Here we teach you to gradually change your lifestyle to accommodate your heart disease." C. "You are probably right but we can gradually increase your activities so that you can live a more active life." D. "Do you feel that you will have to make some changes in your life now?

A Such a response does not have false hope to the client but is positive and realistic. The answer tells the client what cardiac rehabilitation is and does not dwell upon his negativity about it.

A client is wearing a continuous cardiac monitor, which begins to sound its alarm. A nurse sees no electrocardiogram complexes on the screen. The first action of the nurse is to: Check the client status and lead placement Press the recorder button on the electrocardiogram console. Call the physician Call a code blue

A Sudden loss of electrocardiogram complexes indicates ventricular asystole or possible electrode displacement. Accurate assessment of the client and equipment is necessary to determine the cause and identify the appropriate intervention.

The nurse hears a murmur between the S1 and S2 heart sounds at the patient's left 5th intercostal space and midclavicular line. How will the nurse record this information? a. "Systolic murmur heard at mitral area." b. "Diastolic murmur heard at aortic area." c. "Systolic murmur heard at Erb's point." d. "Diastolic murmur heard at tricuspid area."

A The S1 signifies the onset of ventricular systole. S2 signifies the onset of diastole. A murmur occurring between these two sounds is a systolic murmur. The mitral area is the intersection of the left 5th intercostal space and the midclavicular line. The other responses describe murmurs heard at different landmarks on the chest and/or during the diastolic phase of the cardiac cycle.

Which of the following symptoms should the nurse teach the client with unstable angina to report immediately to her physician? a. A change in the pattern of her pain b. Pain during sex c. Pain during an argument with her husband d. Pain during or after an activity such as lawnmowing

A The client should report a change in the pattern of chest pain. It may indicate increasing severity of CAD.

Which of the following symptoms should the nurse teach the client with unstable angina to report immediately to her physician? A. A change in the pattern of her pain B. Pain during sex C. Pain during an argument with her husband D. Pain during or after an activity such as lawn mowing

A The client should report a change in the pattern of chest pain. It may indicate increasing severity of CAD.

A client has developed atrial fibrillation, which a ventricular rate of 150 beats per minute. A nurse assesses the client for: A. Hypotension and dizziness B. Nausea and vomiting C. Hypertension and headache D. Flat neck veins

A The client with uncontrolled atrial fibrillation with a ventricular rate more than 150 beats a minute is at risk for low cardiac output because of loss of atrial kick. The nurse assesses the client for palpitations, chest pain or discomfort, hypotension, pulse deficit, fatigue, weakness, dizziness, syncope, shortness of breath, and distended neck veins.

After an anterior wall myocardial infarction, which of the following problems is indicated by auscultation of crackles in the lungs? A. Left-sided heart failure B. Pulmonic valve malfunction C. Right-sided heart failure D. Tricuspid valve malfunction

A The left ventricle is responsible for the most of the cardiac output. An anterior wall MI may result in a decrease in left ventricular function. When the left ventricle doesn't function properly, resulting in left-sided heart failure, fluid accumulates in the interstitial and alveolar spaces in the lungs and causes crackles. Pulmonic and tricuspid valve malfunction causes right-sided heart failure.

Which of the following is the most common symptom of myocardial infarction? A. Chest pain B. Dyspnea C. Edema D. Palpitations

A The most common symptom of an MI is chest pain, resulting from deprivation of oxygen to the heart. Dyspnea is the second most common symptom, related to an increase in the metabolic needs of the body during an MI. Edema is a later sign of heart failure, often seen after an MI. Palpitations may result from reduced cardiac output, producing arrhythmias.

A patient who is being admitted to the emergency department with severe chest pain gives the following list of medications taken at home to the nurse. Which of the medications has the most immediate implications for the patient's care? a. sildenafil (Viagra) b. furosemide (Lasix) c. diazepam (Valium) d. captopril (Capoten)

A The nurse will need to avoid giving nitrates to the patient because nitrate administration is contraindicated in patients who are using sildenafil because of the risk of sudden death caused by vasodilation. The other home medications also should be documented and reported to the health care provider but do not have as immediate an impact on decisions about the patient's treatment.

Four days after having a myocardial infarction (MI), a patient who is scheduled for discharge asks for assistance with all the daily activities, saying, "I am too nervous to take care of myself." Based on this information, which nursing diagnosis is appropriate? a. Ineffective coping related to anxiety b. Activity intolerance related to weakness c. Denial related to lack of acceptance of the MI d. Social isolation related to lack of support system

A The patient data indicates that ineffective coping after the MI caused by anxiety about the impact of the MI is a concern. The other nursing diagnoses may be appropriate for some patients after an MI, but the data for this patient do not support denial, activity intolerance, or social isolation.

A pulsating abdominal mass usually indicates which of the following conditions? A. Abdominal aortic aneurysm B. Enlarged spleen C. Gastic distention D. Gastritis

A The presence of a pulsating mass in the abdomen is an abnormal finding, usually indicating an outpouching in a weakened vessel, as in abdominal aortic aneurysm. The finding, however, can be normal on a thin person. Neither an enlarged spleen, gastritis, nor gastic distention cause pulsation.

A patient who has chest pain is admitted to the emergency department (ED), and all the following diagnostic tests are ordered. Which one will the nurse arrange to be completed first? a. Electrocardiogram (ECG) b. Computed tomography (CT) scan c. Chest x-ray d. Troponin level

A The priority for the patient is to determine whether an acute myocardial infarction (AMI) is occurring so that reperfusion therapy can begin as quickly as possible. ECG changes occur very rapidly after coronary artery occlusion. Troponin levels will increase after about 3 hours. Data from the CT scan and chest x-ray may impact the patient's care but are not helpful in determining whether the patient is experiencing a myocardial infarction (MI).

What is the most appropriate nursing response to a myocardial infarction client who is fearful of dying? A. "Tell me about your feeling right now." B. "When the doctor arrives, everything will be fine." C. "This is a bad situation, but you'll feel better soon." D. "Please be assured we're doing everything we can to make you feel better."

A Validation of the client's feelings is the most appropriate response. It gives the client a feeling of comfort and safety. The other three responses give the client false hope. No one can determine if a client experiencing MI will feel or get better and therefore, these responses are inappropriate.

Which of the following signs and symptoms would most likely be found in a client with mitral regurgitation? A. Exertional dyspnea B. Confusion C. Elevated creatine phosphokinase concentration D. Chest pain

A Weight gain, due to fluid retention and worsening heart failure, causes exertional dyspnea in clients with mitral regurgitation. The rise in left atrial pressure that accompanies mitral valve disease is transmitted backward into pulmonary veins, capillaries, and arterioles and eventually to the right ventricle. Signs and symptoms of pulmonary and systemic venous congestion follow.

When evaluating the outcomes of preoperative teaching with a patient scheduled for a coronary artery bypass graft (CABG) using the internal mammary artery, the nurse determines that additional teaching is needed when the patient says, a. "I will have incisions in my leg where they will remove the vein." b. "They will circulate my blood with a machine during the surgery." c. "I will need to take an aspirin a day after the surgery to keep the graft open." d. "They will use an artery near my heart to bypass the area that is obstructed."

A When the internal mammary artery is used there is no need to have a saphenous vein removed from the leg. The other statements by the patient are accurate and indicate that the teaching has been effective.

A nurse is assessing an electrocardiogram rhythm strip. The P waves and QRS complexes are regular. The PR interval is 0.16 second, and QRS complexes measure 0.06 second. The overall heart rate is 64 beats per minute. The nurse assesses the cardiac rhythm as: A. Normal sinus rhythm B. Sinus bradycardia C. Sick sinus syndrome D. First-degree heart block

A measurements are normal, measuring 0.12 to 0.20 second and 0.4 to 0.10 second, respectively.

What criteria should the nurse use to determine normal sinus rhythm for a client on a cardiac monitor? Check all that apply. A. The RR intervals are relatively consistent B. One P wave precedes each QRS complex C. Four to eight complexes occur in a 6 second strip D. The ST segment is higher than the PR interval E. The QRS complex ranges from 0.12 to 0.20 second

A,B The consistency of the RR interval indicates regular rhythm. A normal P wave before each complex indicates the impulse originated in the SA node. The number of complexes in a 6 second strip is multiplied by 10 to approximate the heart rate; normal sinus rhythm is 60 to 100. Elevation of the ST segment is a sign of cardiac ischemia and is unrelated to the rhythm. The QRS duration should be less than 0.12 second; the PR interval should be 0.12 to 0.20 second.

A patient who has a diagnosis of atrial fibrillation as a heart rate of 152 beats per minute. The healthcare provider should assess for which of these problems related to the dysrhythmia? A. Chest pain B. Hypotension C. Headache D. Pulse deficit E. Dizziness

A,B,D,E Atrial kick is lost and cardiac output is diminished. The result is less blood supplying the brain (dizziness), hypotension, less blood perfusing the coronary arteries, and a pulse deficit because the stroke volume is not sufficient to produce a palpable peripheral pulse.

What criteria should the nurse use to determine normal sinus rhythm for a client on a cardiac monitor? Check all that apply. A. The RR intervals are relatively consistent B. One P wave precedes each QRS complex C. Four to eight complexes occur in a 6-second strip D. The ST segment is higher than the PR interval E. The QRS complex ranges from 0.12 to 0.2 seconds

AB (1) The consistency of the RR interval indicates a regular rhythm. (2) A normal P wave before each complex indicates the impulse originated in the SA node. (3) The number of complexes in a 6-second strip is multiplied by 10 to approximate the heart rate; normal sinus rhythm is 60 to 100. (4) Elevation of the ST segment is a sign of cardiac ischemia and is unrelated to the rhythm. (5) The QRS duration should be less than 0.12 seconds; the PR interval should be 0.12 to 0.2 seconds.

Which of the following are direct causes of chronic ischemic pain? (Select all that apply.) Aortic stenosis Acid reflux Pulmonary embolus Herpes zoster (shingles)

ABC Rationale: Herpes zoster (shingles) manifests as a vesicular rash along a dermatome, not chronic ischemic pain.

Which of the following are most likely to be early signs of cardiac problems in older persons? (Select all that apply.) Mental status changes Agitation Frequent falls Sudden changes in GI function

ABC Rationale: Many cardiovascular functions are complicated in that they involve many other systems. Mental status changes, agitation, and falls can be early signs of cardiac problems in the older person. Changes in function in the GI system are not typical signs of a cardiac problem.

A patient admitted with heart failure appears very anxious and complains of shortness of breath. Which nursing actions would be appropriate to alleviate this patient's anxiety (select all that apply)? a. Administer ordered morphine sulfate. b. Position patient in a semi-Fowler's position. c. Position patient on left side with head of bed flat. d. Instruct patient on the use of relaxation techniques. e. Use a calm, reassuring approach while talking to patient.

ABDE Morphine sulfate reduces anxiety and may assist in reducing dyspnea. The patient should be positioned in semi-Fowler's position to improve ventilation that will reduce anxiety. Relaxation techniques and a calm reassuring approach will also serve to reduce anxiety.

The nurse is preparing to teach the pt newly diagnosed with angina pectoris how to self-administer nitroglycerine (NTG) tablets. What pt teaching should the nurse emphasize? select all that apply A. stop all activities immediately and sit down if chest pain occurs B. if the pain is unrelieved after three NTG tablets, continue to rest and call the prescriber right away C. After discharge, keep NTG tablets on your bedside table day and night D. immediately after chest pain begins, notify your personal prescriber E. the NTG tablet should tingle when placed under the tongue

ABE

What medications should the nurse expect to include in the teaching plan to decrease the risk of cardiovascular events and death for PAD patients (select all that apply)? a Ramipril (Altace) b Cilostazol (Pletal) c Simvastatin (Zocor) d Clopidogrel (Plavix) e Warfarin (Coumadin) f Aspirin (acetylsalicylic acid)

ACD Angiotensin-converting enzyme inhibitors (e.g., ramipril [Altace]) are used to control hypertension. Statins (e.g., simvastatin [Zocor]) are used for lipid management. Aspirin is used as an antiplatelet agent. Cilostazol (Pletal) is used for intermittent claudication, but it does not reduce CVD morbidity and mortality risks. Clopidogrel may be used if the patient cannot tolerate aspirin. Anticoagulants (e.g., warfarin [Coumadin]) are not recommended to prevent CVD events in PAD patients.

When interpreting an ECG, the nurse would keep in mind which of the following about the P wave? Select all that apply. a. Reflects electrical impulse beginning at the SA node b. Indicated electrical impulse beginning at the AV node c. Reflects atrial muscle depolarization d. Identifies ventricular muscle depolarization e. Has duration of normally 0.11 seconds or less.

ACE In a client who has had an ECG, the P wave represents the activation of the electrical impulse in the SA node, which is then transmitted to the AV node. In addition, the P wave represents atrial muscle depolarization, not ventricular depolarization. The normal duration of the P wave is 0.11 seconds or less in duration and 2.5 mm or more in height.

When interpreting an ECG, the nurse would keep in mind which of the following about the P wave? Select all that apply. A. Reflects electrical impulse beginning at the SA node B. Indicated electrical impulse beginning at the AV node C. Reflects atrial muscle depolarization D. Identifies ventricular muscle depolarization E. Has duration of normally 0.11 seconds or less

ACE In a client who has had an ECG, the P wave represents the activation of the electrical impulse in the SA node, which is then transmitted to the AV node. In addition, the P wave represents atrial muscle depolarization, not ventricular depolarization. The normal duration of the P wave is 0.11 seconds or less in duration and 2.5 mm or more in height.

The patient has heart failure (HF) with an ejection fraction of less than 40%. What core measures should the nurse expect to include in the plan of care for this patient (select all that apply)? a. Left ventricular function is documented. b. Controlling dysrhythmias will eliminate HF. c. Prescription for digoxin (Lanoxin) at discharge d. Prescription for angiotensin-converting enzyme (ACE) inhibitor at discharge e. Education materials about activity, medications, weight monitoring, and what to do if symptoms worsen

ADE The Joint Commission has identified these three core measures for heart failure patients. Although controlling dysrhythmias will improve CO and workload, it will not eliminate HF. Prescribing digoxin for all HF patients is no longer done because there are newer effective drugs and digoxin toxicity occurs easily related to electrolyte levels and the therapeutic range must be maintained.

A 56 y/o man started antihypertensive drug therapy 3 months earlier and is in the office for a follow-up visit. While the nurse is taking his BP, he informs the nurse that he has had some problems with sexual intercourse. Which of the following would be the most appropriate response by the nurse: A. not to worry. Tolerance will develop B. the physician can work with you on changing the dose or drug C. sexual dysfunction happens with this therapy, and you must lean to accept it D. this is an unusual occurrence, but it is important to stay on your meds

B

A 62 y/o male pt has nitroglycerin (Nitrostat) added to his med regimen. Which statement made by this pt indicates that further education is needed? A. I will take this med if I have an episode of chest pain B. I will wait at least 1 hour after I take my sildanefil (Viagra) before using Nitrostat C. I can take up to 3 tablets every 5 minutes if my angina occurs D. I know that I must put this tablet under my tongue for it to work

B

A 68 y/o man has been taking the nitrate isosorbide for 2 years for angina. He recently has been experiencing erectile dysfunction and wants a prescription for sildenafil (Viagra). Which response would the nurse most likely hear from the prescriber? A. He will have to be switched to isosorbide mononitrate if he wants to take sildenafil B. taking sildenafil with the nitrate may result in severe hypotension C. I'll write a prescription, but if he uses it, he needs to stop talking the isosorbide for one dose D. these drugs are compatible with each other, and so I'll write a prescription

B

A P wave on an ECG represents an impulse A. arising at the SA node and repolarizing the atria. B. arising at the SA node and depolarizing the atria. C. arising at the AV node and depolarizing the atria. D. arising at the AV node and spreading to the bundle of His.

B

A client admitted for uncontrolled hypertension and chest pain was started on a daily diuretic two days ago upon admission, with prescriptions for a daily basic metabolic panel. The clients potassium level this morning is 2.7 mEq/L. Which action should the nurse take next? A. Notify the healthcare provider that the potassium level is above normal B. Notify the healthcare provider that the potassium level is below normal C. No action is required because the potassium level is within normal limits D. Hold the clients morning diuretic dose

B

A client arrives at the emergency room complaining of chest pain and dizziness. The client has a history of angina. The healthcare provider prescribes an ECG and lab tests. A change in which component of the ECG tracing should the nurse recognize as the client actively having a myocardial infarction? A. QRS complex B. ST segment C. P wave D. R wave

B

A client asks why smoking is a major risk factor for heart disease. In formulating a response, the nurse incorporates the understanding that nicotine: A. Causes vasodilation B. Causes vasoconstriction C. Increases the level of high-density lipoproteins D. Increases the oxygen-carrying capacity of hemoglobin

B

A client comes to the outpatient clinic and tells the nurse that he has had legs pains that began when he walks but cease when he stops walking. Which of the following conditions would the nurse assess for? A. An acute obstruction in the vessels of the legs B. Peripheral vascular problems in both legs C. Diabetes D. Calcium deficiency

B

A client who had a MI receives 15 mg of morphine sulfate for chest pain. 15 minutes after receiving the drug, The client complains of feeling dizzy. What action should the nurse take? A. Determine if this is an allergic reaction B. Place the client in the supine position and take the vital signs C. Elevate the clients head and keep the extremities warm D. Tell the client that this is not a typical sensation after receiving morphine sulfate

B

A high-risk population that should be targeted in the primary prevention of hypertension is A. smokers. B. African Americans. C. business executives. D. middle-aged women.

B

A home health nurse instructs a pt about the use of a nitrate patch. The nurse tells the pt that which of the following will prevent pt tolerance to nitrates? A. do not remove the patches B. have a 12-hr no-nitrate time C. have a 24-hr no-nitrate time D. keep nitrates on 48 hrs, then off 24 hrs

B

A patient is admitted to the hospital in hypertensive crisis. The nurse recognizes that the hypertensive urgency differs from hypertensive emergency in that A. the BP is always higher in a hypertensive emergency. B. hypertensive emergencies are associated with evidence of target organ damage. C. hypertensive urgency is treated with rest and tranquilizers to lower the BP. D. hypertensive emergencies require intraarterial catheter measurement of the BP.

B

A patient is being discharged after the insertion of a permanent pacemaker. Which statement made by the patient indicates an understanding regarding appropriate self-care? A. "Every morning I will perform arm and shoulder stretches." B. "Each day I'll take my pulse and record it in a log." C. "I'll have to get rid of my microwave oven." D. "I won't be able to use my electric blanket anymore."

B

A patient is being discharged after the insertion of a permanent pacemaker. Which statement made by the patient indicates an understanding regarding appropriate self-care? Please choose from one of the following options. A. "Every morning I will perform arm and shoulder stretches." B. "Each day I'll take my pulse and record it in a log." C. "I'll have to get rid of my microwave oven." D. "I won't be able to use my electric blanket anymore."

B

A patient previously lived in a rural part of Hungary where cooking with animal fat was common. Now in her 80s, she lives with her daughter's family in the United States. She has just learned that she has coronary artery disease. To best meet her physical needs, the nurse should A. suggest that she adopt a low-fat vegetarian dietary pattern. B. ask how she usually prepares foods and suggest ways to reduce the use of animal fat. C. advise her to avoid adding fat to foods and eat only skinless breast of chicken and fish as animal foods. D. advise her to eat more fruits and vegetables and to replace some of the animal fat with palm and coconut oil.

B

A patient who is recovering from an acute myocardial infarction (AMI) asks the nurse about when sexual intercourse can be resumed. Which response by the nurse is best? a. "Most patients are able to enjoy intercourse without any complications." b. "Sexual activity uses about as much energy as climbing two flights of stairs." c. "The doctor will provide sexual guidelines when your heart is strong enough." d. "Holding and cuddling are good ways to maintain intimacy after a heart attack."

B

A pregnant woman develops chest pain and is found to be in atrial fibrillation. Which medication would be appropriate to prescribe for this client? A. Warfarin B. Heparin C. Aspirin D. Atenolol

B

A pt with a rapid, irregular heart rhythm is being treated in the ER with adenosine. During administration of this drug, the nurse should be prepared to monitor the pt for which effect? A. nausea and vomiting B. transitory asystole C. muscle tetany D. hypertension

B

A pt with atrial fibrillation is receiving continuous heparin infusion at 1000 units/hr. The nurse would determine that the pt is receiving the therapeutic effect based on which of the following results? A. prothrombin time of 12.5 seconds B. apt time of 60 seconds C. apt time of 28 seconds D. apt time longer than 120 sec

B

A resting heart rate of 55 beats/min is a normal finding in a patient who A. is obese. B. is an athlete. C. is taking a diuretic. D. weighs less than 90 lb.

B

After a healthcare provider prescribed propranolol for a pt with frequent premature ventricular contractions, the nurse should include with of the following in the care plan? A. inform pt that excessive respiratory response to activity should gradually decrease B. measure heart rate daily before taking dose C. Pt will have increased resistance to infection D. current skin eruptions will improve within 30 days

B

After a healthcare provider prescribed propranolol for a pt with frequent premature ventricular contractions, the nurse should include with of the following in the care plan? A. inform pt that excessive respiratory response to activity should gradually decrease B. measure heart rate daily before taking dose C. Pt will have increased resistance to infection D. current skin eruptions will improve within 30 days

B

After the nurse teaches the patient about the use of carvedilol (Coreg) in preventing anginal episodes, which statement by a patient indicates that the teaching has been effective? a. "Carvedilol will help my heart muscle work harder." b. "It is important not to suddenly stop taking the carvedilol." c. "I can expect to feel short of breath when taking carvedilol." d. "Carvedilol will increase the blood flow to my heart muscle."

B

All potassium-sparing diuretics: A. are required during blood transfusions B. enhance aldosterone action C. cause hypokalemia D. are weak diuretics

B

As a competent nurse, you are aware that vasodilators are used mainly to treat: A. diabetes B. hypertension C. atrial fibrillation D. hypotension

B

Blurred vision or halos are signs of: A. beta blocker toxicity B. digoxin toxicity C. diuretic toxicity D. ACE inhibitor toxicity

B

Compared with pain from an MI, angina pain usually A. is unrelieved by rest. B. lasts less than 15 minutes. C. is accompanied by palpitations. D. radiates to the arm, jaw, or throat.

B

Diltiazem (Cardizem) is ordered for a patient with newly diagnosed Prinzmetal's (variant) angina. When teaching the patient, the nurse will include the information that diltiazem will a. reduce heart palpitations. b. decrease spasm of the coronary arteries. c. increase the force of the heart contractions. d. help prevent plaque from forming in the coronary arteries.

B

During preoperative prep of patient scheduled for an abdominal aortic aneurysm the nurse establishes basline data for the patient knowing that a. postop all pphysiologic processes will be altered b. The cause of the aneurysm is a systemic vasuclar disease c. surgery will be canceled if any function is not normal d. blood pressure and HR will be maintained below normal levels during surgery

B

During the patient's acute postop period following repair of an aneurysm, the nurse should ensure that: a. hypothermia is maintained to decrease 02 need b. the blood pressure and all peripheral pulses are evaluated at least every hour c. IV fluids are admin at a rate to keep urine at 100 ml / hr d. patient's bp is kept lower than baseline

B

Examination of a patient in a supine position reveals distended jugular veins from the base of the neck to the angle of the jaw. This finding indicates: a. decreased venous return. b. increased central venous pressure. c. increased pulmonary artery capillary pressure. d. left-sided heart failure.

B

Hypokalemia is a potential side effect of Digoxin. You need to educate your pt on consuming potassium-rich foods. Which of the following will you include in your list of potassium-rich foods? A. Eggs B. strawberries C. whole grain bread

B

Mira is managing her hypertension with an ACE inhibitor. Which of the following statements stated by her indicates a need for further teaching? A. I should not take my pills with food B. I need to increase my intake of orange juice, bananas, and green veggies C. I will avoid coffee, tea, and cola D. I will avoid salt substitutes

B

Nursing interventions indicated in the plan of care for the patient with acute lower extremity deep vein thrombosis include A. applying elastic compression stockings. B. administering anticoagulants as ordered. C. positioning the leg dependently to promote arterial circulation. D. encouraging walking and leg exercises to promote venous return.

B

Rest pain is a manifestation of peripheral arterial disease that occurs as a result of A. the beginning of a venous leg ulcer. B. inadequate blood flow to the nerves of the feet. C. inadequate blood flow to the muscles during exercise. D. inadequate blood flow to the skin after application of the heat.

B

SC heparin should be administered in the: A. flank B. abdominal fat C. thigh D. buttock

B

The auscultatory area in the left midclavicular line at the level of the fifth ICS is the A. aortic area. B. mitral area. C. tricuspid area. D. pulmonic area.

B

The cardiac monitor of a patient in the cardiac care unit following an acute MI indicates ventricular bigeminy. The nurse anticipates A. performing defibrillation. B. treatment with IV lidocaine. C. insertion of a temporary pacemaker. D. continuing monitoring without other treatment.

B

The half-life of heparin is: A. 10 minutes B. 1-1.5 hrs C. 8-12 hrs D. 1-2 days

B

The healthcare provider is performing an assessment on a patient who is taking propranolol (Inderal) for supraventricular tachycardia. Which assessment finding is an indication the patient is experiencing an adverse effect of this drug? A. Dry mouth B. Bradycardia C. Urinary retention D. Paresthesia

B

The healthcare provider is performing an assessment on a patient who is taking propranolol (Inderal) for supraventricular tachycardia. Which assessment finding is an indication the patient is experiencing an adverse effect of this drug? Please choose from one of the following options. A. Dry mouth B. Bradycardia C. Urinary retention D. Paresthesia

B

The nurse feels pulsations on a client's right sternal border, second space. What does this finding suggest to the nurse? a. A prolapsed mitral valve b. Aortic stenosis c. Nothing. This is a normal finding. d. Tricuspid valve regurgitation

B

The nurse is monitoring a pt taking digoxin (Lanoxin) for treatment of heart failure. Which assessment finding indicates a therapeutic effect of the drug? A. HR 110 beats/minute B. HR 58 beats/minute C. urinary output 40 mL/hr D. BP 90/50 mm Hg

B

The nurse plans care for the patient with an implantable cardioverter-defibrillator based on the knowledge that A. antiarrhythmia drugs can be discontinued. B. all members of the patient's family should learn CPR. C. the patient should not drive until 1 month after the ICD has been implanted. D. the patient is usually relieved to have the device implanted to prevent arrhythmias.

B

The nurse provides discharge instructions to a pt about the use of amiodarone. Which of the following statements indicates that the pt has the knowledge necessary to safely administer the drug? A. as soon as the physician says I can stop taking this medication, I will be able to enjoy the sun again B. the side effects of this med may not begin to show up for several weeks or even months after I start taking it C. If my pulse drops below 100 beats/minute, I should call the physician right away D. If I miss a dose of me, I should take it as soon as I remember it

B

The nurse will suspect that the patient with stable angina is experiencing a side effect of the prescribed metoprolol (Lopressor) if the a. patient is restless and agitated. b. blood pressure is 90/54 mm Hg. c. patient complains about feeling anxious. d. cardiac monitor shows a heart rate of 61 beats/minute.

B

To improve the physical activity level for a mildly obese 71-year-old patient, which action should the nurse plan to take? a. Stress that weight loss is a major benefit of increased exercise. b. Determine what kind of physical activities the patient usually enjoys. c. Tell the patient that older adults should exercise for no more than 20 minutes at a time. d. Teach the patient to include a short warm-up period at the beginning of physical activity.

B

When a pt is experiencing digitalis toxicity, in which of the following situations would it be appropriate to treat with digoxin immune Fab (Digibind)? A. hypokalemia B. hyperkalemia C. apical heart rate of 60 bpm D. supraventricular dysrhythmias

B

When admitting a patient with a non-ST-segment-elevation myocardial infarction (NSTEMI) to the intensive care unit, which action should the nurse perform first? a. Obtain the blood pressure. b. Attach the cardiac monitor. c. Assess the peripheral pulses. d. Auscultate the breath sounds.

B

When caring for a patient who is recovering from a sudden cardiac death (SCD) event and has no evidence of an acute myocardial infarction (AMI), the nurse will anticipate teaching the patient that a. sudden cardiac death events rarely reoccur. b. additional diagnostic testing will be required. c. long-term anticoagulation therapy will be needed. d. limited physical activity after discharge will be needed to prevent future events.

B

When caring for a patient with a cardiac dysrhythmia, which laboratory value is a priority for the healthcare provider to monitor? A. BUN and creatinine B. Sodium, potassium, and calcium C. Hemoglobin and hematocrit D. PT and INR

B

When evaluating the effectiveness of preoperative teaching with a patient scheduled for coronary artery bypass graft (CABG) surgery using the internal mammary artery, the nurse determines that additional teaching is needed when the patient says which of the following? a. "They will circulate my blood with a machine during the surgery." b. "I will have small incisions in my leg where they will remove the vein." c. "They will use an artery near my heart to go around the area that is blocked." d. "I will need to take an aspirin every day after the surgery to keep the graft open."

B

When teaching the pt about the signs and symptoms of cardiac glycoside toxicity, the nurse should alert the pt to watch for: A. visual changes B. flickering lights or halos C. dizziness when standing up D. increased urine output

B

When titrating IV nitroglycerin (Tridil) for a patient with a myocardial infarction (MI), which action will the nurse take to evaluate the effectiveness of the medication? a. Monitor heart rate. b. Ask about chest pain. c. Check blood pressure. d. Observe for dysrhythmias.

B

When ventricular fibrillation occurs in a CCU, the first person reaching the client should: A. Administer oxygen B. Defibrillate the client C. Initiate CPR D. Administer sodium bicarbonate intravenously

B

Which assessment data collected by the nurse who is admitting a patient with chest pain suggest that the pain is caused by an acute myocardial infarction (AMI)? a. The pain increases with deep breathing. b. The pain has lasted longer than 30 minutes. c. The pain is relieved after the patient takes nitroglycerin. d. The pain is reproducible when the patient raises the arms.

B

Which assessment finding by the nurse caring for a patient who has had coronary artery bypass grafting using a right radial artery graft is most important to communicate to the health care provider? a. Complaints of incisional chest pain b. Pallor and weakness of the right hand c. Fine crackles heard at both lung bases d. Redness on both sides of the sternal incision

B

Which group of drugs can be either selective or non-selective? a. sodium channel blockers B. beta blockers C. calcium channel blockers

B

Which of the following adverse effects is of most concern for the older adult pt taking anti-hypertensive drugs A. dry mouth B. hypotension C. restlessness D. constipation

B

Which patient at the cardiovascular clinic requires the most immediate action by the nurse? a. Patient with type 2 diabetes whose current blood glucose level is 145 mg/dL b. Patient with stable angina whose chest pain has recently increased in frequency c. Patient with familial hypercholesterolemia and a total cholesterol of 465 mg/dL d. Patient with chronic hypertension whose blood pressure today is 172/98 mm Hg

B

While obtaining subjective assessment data from a patient with hypertension, the nurse recognizes that a modifiable risk factor for the development of hypertension is A. hyperlipidemia. B. excessive alcohol intake. C. a family history of hypertension. D. consumption of a high-carbohydrate, high-calcium diet

B

With peripheral arterial insufficiency, leg pain during rest can be reduced by: a. Elevating the limb above heart level b. Lowering the limb so it is dependent c. Massaging the limb after application of cold compresses d. Placing the limb in a plane horizontal to the body

B

Your pt is currently taking Digoxin. What should you, as a nurse be prepared to administer in the event of digitalis toxicity? A. potassium B. digibind C. protamine sulfate D. heparin

B

a client who has been taking warfarin (Coumadin) is admitted with coffee-ground emesis. What is the nurse's primary action? A. Administer vitamin E B. Administer vitamin K C. Administer protamine sulfate D. Administer calcium gluconate

B

a nurse gives a subq injection of heparin sodium (heparin). Which is a true statement regarding this injection? A. massage the area after heparin is administered subq B. aspiration before injection can cause hematoma formation C. give the injection each time in the same general area D. hold the skin taut when giving the injection

B

a pt is receiving thrombolytic therapy with a continuous infusion of streptokinase. The pt suddenly becomes extremely anxious with complaints of itching. A nurse hears stridor and on examination of the pt notes generalized urticarial and hypotension. Which of the following should be the priority action? A. administer oxygen and protamine sulfate B. stop the infusion and call the physician C. cut the infusion rate in half and sit the pt up in bed D. administer benadryl and continue the infusion slowly

B

a pt receives a nitroglycerin drip. Which is the true statement regarding the use of this med in IV therapy? A. vented tubing is needed to administer this med B. a glass bottle and vented tubing are needed to administer this med C. admixture must be done under a laminar flow hood with proper handling techniques D. this med cannot be given through small diameter catheters in elderly pts

B

the nurse reviews lab studies of a pt receiving digoxin (Lanoxin). Intervention by the nurse is required if the results include which of the following laboratory values? A. serum digoxin level of 1.2 ng/dL B. Serum potassium level of 3 mEq/L C. hemoglobin of 14.4 g/ dL D. serum sodium level of 140 mEq/L

B

what is the digitalizing dose of Digoxin (Lanoxin)? A. 0.125-0.5 B. 0.5-2 C. 0.5-3

B

what is the most common side effect of ACE inhibitors? A. headache B. persistent dry cough C. visual disturbances

B

when a pt is experiencing digitalis toxicity, in which of the following situations would it be appropriate to treat with digoxin immune Fab (digibind) A. hypokalemia B. hyperkalemia C. apical heart rate of 60 bpm D. supraventricular arrhythmias

B

which of the following anticoagulants is used prophylactically? A. heparin B. lovenox B. coumadin

B

which of the following is the serum abnormality with thiazides? A. hyperkalemia B. hyperglycemia C. hyperchloremia

B

What should the nurse recognize as an indication for the use of dopamine (Intropin) in the care of a patient with heart failure? a. Acute anxiety b. Hypotension and tachycardia c. Peripheral edema and weight gain d. Paroxysmal nocturnal dyspnea (PND)

B Dopamine is a β-adrenergic agonist whose inotropic action is used for treatment of severe heart failure accompanied by hemodynamic instability. Such a state may be indicated by tachycardia accompanied by hypotension. PND, anxiety, edema, and weight gain are common signs and symptoms of heart failure, but these do not necessarily warrant the use of dopamine.

A nurse is caring for a patient with a diagnosis of deep venous thrombosis (DVT). The patient has an order to receive 30 mg enoxaparin (Lovenox). Which injection site should the nurse use to administer this medication safely? a. Buttock, upper outer quadrant b. Abdomen, anterior-lateral aspect c. Back of the arm, 2 inches away from a mole d. Anterolateral thigh, with no scar tissue nearby

B Rationale: Enoxaparin (Lovenox) is a low-molecular-weight (LMW) heparin that is given as a deep subcutaneous injection in the right and left anterolateral abdomen. All subcutaneous injections should be given away from scars, lesions, or moles.

The nurse is caring for a newly admitted patient with vascular insufficiency. The patient has a new order for enoxaparin (Lovenox) 30 mg subcutaneously. What should the nurse do to correctly administer this medication? a. Spread the skin before inserting the needle. b. Leave the air bubble in the prefilled syringe. c. Use the back of the arm as the preferred site. d. Sit the patient at a 30-degree angle before administration.

B Rationale: The nurse should not expel the air bubble from the prefilled syringe because it should be injected to clear the needle of medication and avoid leaving medication in the needle track in the tissue.

A patient with varicose veins has been prescribed compression stockings. How should the nurse teach the patient to use these? a. "Try to keep your stockings on 24 hours a day, as much as possible." b. "While you're still lying in bed in the morning, put on your stockings." c. "Dangle your feet at your bedside for 5 minutes before putting on your stockings." d. "Your stockings will be most effective if you can remove them for a few minutes several times a day."

B Rationale: The patient with varicose veins should apply stockings in bed, before rising in the morning. Stockings should not be worn continuously, but they should not be removed several times daily. Dangling at the bedside prior to application is likely to decrease their effectiveness.

A patient was just diagnosed with acute arterial ischemia in the left leg secondary to atrial fibrillation. Which early clinical manifestation must be reported to the physician immediately to save the patient's limb? a Paralysis b Paresthesia c Crampiness d Referred pain

B Rationale: The physician must be notified immediately if any of the six Ps of acute arterial ischemia occur to prevent ischemia from quickly progressing to tissue necrosis and gangrene. The six Ps are paresthesia, pain, pallor, pulselessness, and poikilothermia, with paralysis being a very late sign indicating the death of nerves to the extremity. Crampy leg sensation is more common with varicose veins. The pain is not referred.

The patient had aortic aneurysm repair. What priority nursing action will the nurse use to maintain graft patency? a Assess output for renal dysfunction. b Use IV fluids to maintain adequate BP. c Use oral antihypertensives to maintain cardiac output. d Maintain a low BP to prevent pressure on surgical site

B Rationale: The priority is to maintain an adequate BP (determined by the surgeon) to maintain graft patency. A prolonged low BP may result in graft thrombosis, and hypertension may cause undue stress on arterial anastomoses resulting in leakage of blood or rupture at the suture lines, which is when IV antihypertensives may be used. Renal output will be assessed when the aneurysm repair is above the renal arteries to assess graft patency, not maintain it.

A female patient with critical limb ischemia has had peripheral artery bypass surgery to improve her circulation. What care should the nurse provide on postoperative day 1? a Keep the patient on bed rest. b Assist the patient with walking several times. c Have the patient sit in the chair several times. d Place the patient on her side with knees flexed

B Rationale: To avoid blockage of the graft or stent, the patient should walk several times on postoperative day 1 and subsequent days. Having the patient's knees flexed for sitting in a chair or in bed increase the risk of venous thrombosis and may place stress on the suture lines.

A patient is admitted to the hospital with a diagnosis of abdominal aortic aneurysm. Which signs and symptoms would suggest that his aneurysm has ruptured? a.Sudden shortness of breath and hemoptysis b.Sudden, severe low back pain and bruising along his flank c.Gradually increasing substernal chest pain and diaphoresis d.Sudden, patchy blue mottling on feet and toes and rest pain

B The clinical manifestations of a ruptured abdominal aortic aneurysm include severe back pain, back or flank ecchymosis (Grey Turner's sign), and hypovolemic shock (tachycardia, hypotension, pale clammy skin, decreased urine output, altered level of consciousness, and abdominal tenderness).

The nurse is preparing to administer digoxin to a patient with heart failure. In preparation, laboratory results are reviewed with the following findings: sodium 139 mEq/L, potassium 5.6 mEq/L, chloride 103 mEq/L, and glucose 106 mg/dL. What should the nurse do next? a. Withhold the daily dose until the following day. b. Withhold the dose and report the potassium level. c. Give the digoxin with a salty snack, such as crackers. d. Give the digoxin with extra fluids to dilute the sodium level.

B The normal potassium level is 3.5 to 5.0 mEq/L. The patient is hyperkalemic, which makes the patient more prone to digoxin toxicity. For this reason, the nurse should withhold the dose and report the potassium level. The physician may order the digoxin to be given once the potassium level has been treated and decreases to within normal range.

A patient with a diagnosis of heart failure has been started on a nitroglycerin patch by his primary care provider. What should this patient be taught to avoid? a. High-potassium foods b. Drugs to treat erectile dysfunction c. Nonsteroidal antiinflammatory drugs d. Over-the-counter H2 -receptor blockers

B The use of erectile drugs concurrent with nitrates creates a risk of severe hypotension and possibly death. High-potassium foods, NSAIDs, and H2-receptor blockers do not pose a risk in combination with nitrates.

The patient at highest risk for venous thromboembolism (VTE) is a.a 62-year-old man with spider veins who is having arthroscopic knee surgery. b.a 32-year-old woman who smokes, takes oral contraceptives, and is planning a trip to Europe. c.a 26-year-old woman who is 3 days postpartum and received maintenance IV fluids for 12 hours during her labor. d.an active 72-year-old man at home recovering from transurethral resection of the prostate for benign prostatic hyperplasia.

B Three important factors (called Virchow's triad) in the etiology of venous thrombosis are (1) venous stasis, (2) damage of the endothelium (inner lining of the vein), and (3) hypercoagulability of the blood. Patients at risk for venous thrombosis usually have predisposing conditions for these three disorders (see Table 38-8). The 32-year-old woman has the highest risk: long trips without adequate exercise (venous stasis), tobacco use, and use of oral contraceptives. Note: The likelihood of hypercoagulability of blood is increased in women older than 35 years who use tobacco.

A common arrhythmia found in some older clients is chronic atrial fibrillation. Based on the nurse's knowledge of the disease pathology, which of the following prescriptions should the nurse expect to be ordered? Aspirin (acetylsalicylic acid) Warfarin sodium (Coumadin) Simvastatin (Zocor) Vinorelbine tartrate (Navelbine)

B Rationale: Chronic atrial fibrillation places a patient at high risk for clot formation. Warfarin sodium frequently is ordered as an anti-coagulant. Aspirin will not prevent clots associated with atrial fibrillation. Zocor is used to lower LDL and increase HDL. Navelbine is an anti-neoplastic.

When teaching a patient why spironolactone (Aldactone) and furosemide (Lasix) are prescribed together, the nurse bases teaching on the knowledge that: a. Moderate doses of two different types of diuretics are more effective than a large dose of one type b. This combination promotes diuresis but decreases the risk of hypokalemia c. This combination prevents dehydration and hypovolemia d. Using two drugs increases osmolality of plasma and the glomerular filtration rate

B Spironolactone is a potassium-sparing diuretic; furosemide is a potassium-loosing diuretic. Giving these together minimizes electrolyte imbalance

What is the first intervention for a client experiencing myocardial infarction? A. Administer morphine B. Administer oxygen C. Administer sublingual nitroglycerin D. Obtain an electrocardiogram

B Administering supplemental oxygen to the client is the first priority of care. The myocardium is deprived of oxygen during an infarction, so additional oxygen is administered to assist in oxygenation and prevent further damage. Morphine and sublingual nitroglycerin are also used to treat MI, but they're more commonly administered after the oxygen. An ECG is the most common diagnostic tool used to evaluate MI.

A client has driven himself to the ER. He is 50 years old, has a history of hypertension, and informs the nurse that his father died of a heart attack at 60 years of age. The client is presently complaining of indigestion. The nurse connects him to an ECG monitor and begins administering oxygen at 2 L/minute per NC. The nurse's next action would be to: a. Call for the doctor b. Start an intravenous line c. Obtain a portable chest radiograph d. Draw blood for laboratory studies

B Advanced cardiac life support recommends that at least one or two intravenous lines be inserted in one or both of the antecubital spaces. Calling the physician, obtaining a portable chest radiograph, and drawing blood are important but secondary to starting the intravenous line.

A nurse is caring for a client who had a percutaneous insertion of an inferior vena cava filter and was on heparin therapy before surgery. The nurse would inspect the surgical site most closely for signs of: a. Thrombosis and infection b. Bleeding and infection c. Bleeding and wound dehiscence. d. Wound dehiscence and evisceration.

B After inferior vena cava insertion, the nurse inspects the surgical site for bleeding and signs and symptoms of infection. Otherwise, care is the same as for any post-op client.

When do coronary arteries primarily receive blood flow? A. During inspiration b. During diastole C. During expiration D. During systole

B Although the coronary arteries may receive a minute portion of blood during systole, most of the blood flow to coronary arteries is supplied during diastole. Breathing patterns are irrelevant to blood flow

When do coronary arteries primarily receive blood flow? a. During inspiration b. During diastolic c. During expiration d. During systole

B Although the coronary arteries may receive a minute portion of blood during systole, most of the blood flow to coronary arteries is supplied during diastole. Breathing patterns are irrelevant to blood flow.

What is the definitive test used to diagnose an abdominal aortic aneurysm? A. Abdominal X-ray B. Arteriogram C. CT scan D. Ultrasound

B An arteriogram accurately and directly depicts the vasculature; therefore, it clearly delineates the vessels and any abnormalities. An abdominal aneurysm would only be visible on an X-ray if it were calcified. CT scan and ultrasound don't give a direct view of the vessels and don't yield as accurate a diagnosis as the arteriogram.

Atherosclerosis impedes coronary blood flow by which of the following mechanisms? A. Plaques obstruct the vein B. Plaques obstruct the artery C. Blood clots form outside the vessel wall D. Hardened vessels dilate to allow the blood to flow through

B Arteries, not veins, supply the coronary arteries with oxygen and other nutrients. Atherosclerosis is a direct result of plaque formation in the artery. Hardened vessels can't dilate properly and, therefore, constrict blood flow.

A nurse is watching the cardiac monitor, and a client's rhythm suddenly changes. There are no P waves; instead there are wavy lines. The QRS complexes measure 0.08 second, but they are irregular, with a rate of 120 beats a minute. The nurse interprets this rhythm as: A. Sinus tachycardia B. Atrial fibrillation C. Ventricular tachycardia D. Ventricular fibrillation

B Atrial fibrillation is characterized by a loss of P waves; an undulating, wavy baseline; QRS duration that is often within normal limits; and an irregular ventricular rate, which can range from 60 to 100 beats per minute (when controlled with medications) to 100 to 160 beats per minute (when uncontrolled).

When admitting a patient with a myocardial infarction (MI) to the intensive care unit, which action should the nurse carry out first? a. Obtain the blood pressure. b. Attach the cardiac monitor. c. Assess the peripheral pulses. d. Auscultate the breath sound

B Because dysrhythmias are the most common complication of MI, the first action should be to place the patient on a cardiac monitor. The other actions also are important and should be accomplished as quickly as possible.

Which of the following classes of drugs is most widely used in the treatment of cardiomyopathy? A. Antihypertensive B. Beta-adrenergic blockers C. Calcium channel blockers D. Nitrates

B By decreasing the heart rate and contractility, beta-adrenergic blockers improve myocardial filling and cardiac output, which are primary goals in the treatment of cardiomyopathy. Antihypertensives aren't usually indicated because they would decrease cardiac output in clients who are often already hypotensive. Calcium channel blockers are sometimes used for the same reasons as beta-adrenergic blockers; however, they aren't as effective as beta-adrenergic blockers and cause increase hypotension. Nitrates aren't' used because of their dilating effects, which would further compromise the myocardium.

The physician refers the client with unstable angina for a cardiac catherization. The nurse explains to the client that this procedure is being used in this specific case to: a. Open and dilate the blocked coronary arteries b. Assess the extent of arterial blockage c. Bypass obstructed vessels d. Assess the functional adequacy of the valves and heart muscle.

B Cardiac catherization is done in clients with angina primarily to assess the extent and severity of the coronary artery blockage, A decision about medical management, angioplasty, or coronary artery bypass surgery will be based on the catherization results.

The physician refers the client with unstable angina for a cardiac catheterization. The nurse explains to the client that this procedure is being used in this specific case to: A. Open and dilate the blocked coronary arteries B. Assess the extent of arterial blockage C. Bypass obstructed vessels D. Assess the functional adequacy of the valves and heart muscle

B Cardiac catheterization is done in clients with angina primarily to assess the extent and severity of the coronary artery blockage, A decision about medical management, angioplasty, or coronary artery bypass surgery will be based on the catheterization results.

Which of the following types of cardiomyopathy does not affect cardiac output? A. Dilated B. Hypertrophic C. Restrictive D. Obliterative

B Cardiac output isn't affected by hypertrophic cardiomyopathy because the size of the ventricle remains relatively unchanged. Dilated cardiomyopathy, and restrictive cardomyopathy all decrease cardiac output.

The nurse has received the laboratory results for a patient who developed chest pain 4 hours ago and may be having a myocardial infarction. The most important laboratory result to review will be a. LDL cholesterol. b. troponins T and I. c. C-reactive protein. d. creatine kinase-MB (CK-MB).

B Cardiac troponins start to elevate hours (average 4 to 6 hours) after myocardial injury and are specific to myocardium. Creatine kinase (CK-MB) is specific to myocardial injury and infarction, but it does not increase until 6 hours after the infarction occurs. LDL cholesterol and C-reactive protein are useful in assessing cardiovascular risk but are not helpful in determining whether a patient is having an acute myocardial infarction.

Which information collected by the nurse who is admitting a patient with chest pain suggests that the pain is caused by an acute myocardial infarction (AMI)? a. The pain increases with deep breathing. b. The pain has persisted longer than 30 minutes. c. The pain worsens when the patient raises the arms. d. The pain is relieved after the patient takes nitroglycerin.

B Chest pain that lasts for 20 minutes or more is characteristic of AMI. Changes in pain that occur with raising the arms or with deep breathing are more typical of pericarditis or musculoskeletal pain. Stable angina is usually relieved when the patient takes nitroglycerin.

A 2-gram sodium diet is prescribed for a client with severe hypertension. The client does not like the diet, and the nurse hears the client request that the spouse "Bring in some good home-cooked food." It would be most effective for the nurse to plan to: A. Call in the dietician for client teaching B. Wait for the client's family and discuss the diet with the client and family C. Tell the client that the use of salt is forbidden, because it will raise BP D. Catch the family members before they go into the client's room and tell them about the diet.

B Clients' families should be included in dietary teaching; families provide support that promotes adherence.

Which of the following conditions is linked to more than 50% of clients with abdominal aortic aneurysms? A. DM B. HPN C. PVD D. Syphilis

B Continuous pressure on the vessel walls from hypertension causes the walls to weaken and an aneurysm to occur. Atherosclerotic changes can occur with peripheral vascular diseases and are linked to aneurysms, but the link isn't as strong as it is with hypertension. Only 1% of clients with syphilis experience an aneurysm. Diabetes mellitus doesn't have direct link to aneurysm.

A nurse is caring for a client with unstable ventricular tachycardia. The nurse instructs the client to do which of the following, if prescribed, during an episode of ventricular tachycardia? A. Breathe deeply, regularly, and easily B. Inhale deeply and cough forcefully every 1 to 3 seconds C. Lie down flat in bed D. Remove any metal jewelry

B Cough cardiopulmonary resuscitation (CPR) sometimes is used in the client with unstable ventricular tachycardia. The nurse tells the client to use cough CPR, if prescribed, by inhaling deeply and coughing forcefully every 1 to 3 seconds. Cough CPR may terminate the dysrhythmia or sustain the cerebral and coronary circulation for a short time until other measures can be implemented.

For a patient who has been admitted the previous day to the coronary care unit with an acute myocardial infarction (AMI), the nurse will anticipate teaching about a. typical emotional responses to AMI. b. when patient cardiac rehabilitation will begin. c. discharge drugs such as aspirin and -blockers. d. the pathophysiology of coronary artery disease.

B Early after an AMI, the patient will want to know when resumption of usual activities can be expected. At this time, the patient's anxiety level or denial will prevent good understanding of complex information such as coronary artery disease (CAD) pathophysiology. Teaching about discharge medications should be done when the time for discharge is closer. The nurse should support the patient by decreasing anxiety rather than discussing the typical emotional response to myocardial infarction (MI).

With which of the following disorders is jugular vein distention most prominent? A. Abdominal aortic aneurysm B. Heart failure C. Myocardial infarction D. Pneumothorax

B Elevated venous pressure, exhibited as jugular vein distention, indicates a failure of the heart to pump. Jugular vein distention isn't a symptom of abdominal aortic aneurysm or pneumothorax. An MI, if severe enough, can progress to heart failure; however, in and of itself, an MI doesn't cause jugular vein distention.

A client has been admitted to the hospital with a diagnosis of suspected bacterial endocarditis. The complication the nurse will constantly observe for is: A. Presence of heart murmur B. Systemic emboli C. Fever D. Congestive heart failure

B Emboli are the major problem; those arising in the right heart chambers will terminate in the lungs and left chamber emboli may travel anywhere in the arteries. Heart murmurs, fever, and night sweats may be present, but do not indicate a problem with emboli. CHF may be a result, but this is not as dangerous an outcome as emboli

Which of the following actions is the first priority care for a client exhibiting signs and symptoms of coronary artery disease? A. Decrease anxiety B. Enhance myocardial oxygenation C. Administer sublignual nitroglycerin D. Educate the client about his symptoms

B Enhancing mocardial oxygenation is always the first priority when a client exhibits signs and symptoms of cardiac compromise. Without adequate oxygen, the myocardium suffers damage. Sublingual nitorglycerin is administered to treat acute angina, but its administration isn't the first priority. Although educating the client and decreasing anxiety are important in care delivery, nether are priorities when a client is compromised.

A patient with ST segment elevation in several electrocardiographic (ECG) leads is admitted to the emergency department (ED) and diagnosed as having an ST-segment-elevation myocardial infarction (STEMI). Which question should the nurse ask to determine whether the patient is a candidate for fibrinolytic therapy? a. "Do you take aspirin on a daily basis?" b. "What time did your chest pain begin?" c. "Is there any family history of heart disease?" d. "Can you describe the quality of your chest pain?"

B Fibrinolytic therapy should be started within 6 hours of the onset of the myocardial infarction (MI), so the time at which the chest pain started is a major determinant of the appropriateness of this treatment. The other information also will be needed, but it will not be a factor in the decision about fibrinolytic therapy.

When caring for a patient who has survived a sudden cardiac death (SCD) event and has no evidence of an acute myocardial infarction (AMI), the nurse will anticipate teaching the patient a. that sudden cardiac death events rarely reoccur. b. about the purpose of outpatient Holter monitoring. c. how to self-administer low-molecular-weight heparin. d. to limit activities after discharge to prevent future events.

B Holter monitoring is used to determine whether the patient is experiencing dysrhythmias such as ventricular tachycardia during normal daily activities. SCD is likely to recur. Heparin will not have any effect on the incidence of SCD, and SCD can occur even when the patient is resting.

A nurse notices frequent artifact on the ECG monitor for a client whose leads are connected by cable to a console at the bedside. The nurse examines the client to determine the cause. Which of the following items is unlikely to be responsible for the artifact? A. Frequent movement of the client B. Tightly secured cable connections C. Leads applied over hairy areas D. Leads applied to the limbs

B Motion artifact, or "noise," can be caused by frequent client movement, electrode placement on limbs, and insufficient adhesion to the skin, such as placing electrodes over hairy areas of the skin. Electrode placement over bony prominences also should be avoided. Signal interference can also occur with electrode removal and cable disconnection.

A client with myocardial infarction has been transferred from a coronary care unit to a general medical unit with cardiac monitoring via telemetry. A nurse plans to allow for which of the following client activities? Strict bed rest for 24 hours after transfer Bathroom privileges and self-care activities Unsupervised hallway ambulation with distances under 200 feet Ad lib activities because the client is monitored.

B On transfer from the CCU, the client is allowed self-care activities and bathroom privileges. Supervised ambulation for brief distances are encouraged, with distances gradually increased (50, 100, 200 feet).

A client with myocardial infarction has been transferred from a coronary care unit to a general medical unit with cardiac monitoring via telemetry. A nurse plans to allow for which of the following client activities? A. Strict bed rest for 24 hours after transfer B. Bathroom privileges and self-care activities C. Unsupervised hallway ambulation with distances under 200 feet D. Ad lib activities because the client is monitored

B On transfer from the CCU, the client is allowed self-care activities and bathroom privileges. Supervised ambulation for brief distances are encouraged, with distances gradually increased (50, 100, 200 feet).

While caring for a client who has sustained an MI, the nurse notes eight PVCs in one minute on the cardiac monitor. The client is receiving an IV infusion of D5W and oxygen at 2 L/minute. The nurse's first course of action should be to: A. Increase the IV infusion rate B. Notify the physician promptly C. Increase the oxygen concentration D. Administer a prescribed analgesic

B PVCs are often a precursor of life-threatening dysrhythmias, including ventricular tachycardia and ventricular fibrillation. An occasional PVC is not considered dangerous, but if PVCs occur at a rate greater than 5 or 6 per minute in the post MI client, the physician should be notified immediately. More than 6 PVCs per minute is considered serious and usually calls for decreasing ventricular irritability by administering medications such as lidocaine. Increasing the IV infusion rate would not decrease the number of PVCs. Increasing the oxygen concentration should not be the nurse's first course of action; rather, the nurse should notify the physician promptly. Administering a prescribed analgesic would not decrease ventricular irritability.

Amlodipine (Norvasc) is ordered for a patient with newly diagnosed Prinzmetal's (variant) angina. When teaching the patient, the nurse will include the information that amlodipine will a. reduce the "fight or flight" response. b. decrease spasm of the coronary arteries. c. increase the force of myocardial contraction. d. help prevent clotting in the coronary arteries.

B Prinzmetal's angina is caused by coronary artery spasm. Calcium channel blockers (e.g., amlodipine, nifedipine [Procardia]) are a first-line therapy for this type of angina. Platelet inhibitors, such as aspirin, help prevent coronary artery thrombosis, and -blockers decrease sympathetic stimulation of the heart. Medications or activities that increase myocardial contractility will increase the incidence of angina by increasing oxygen demand.

The nurse identifies the collaborative problem of potential complication: pulmonary edema for a patient in ADHF. When assessing the patient, the nurse will be most concerned about a. an apical pulse rate of 106 beats/min. b. an oxygen saturation of 88% on room air. c. weight gain of 1 kg (2.2 lb) over 24 hours. d. decreased hourly patient urinary output.

B Rationale: A decrease in oxygen saturation to less than 92% indicates hypoxemia. The nurse should administer supplemental oxygen immediately to the patient. An increase in apical pulse rate, 1-kg weight gain, and decreases in urine output also indicate worsening heart failure and require rapid nursing actions, but the low oxygen saturation rate requires the most immediate nursing action.

A patient who is receiving dobutamine (Dobutrex) for the treatment of ADHF has all of the following nursing actions included in the plan of care. Which action will be best for the RN to delegate to an experienced LPN/LVN? a. Teach the patient the reasons for remaining on bed rest. b. Monitor the patient's BP every hour. c. Adjust the drip rate to keep the systolic BP >90 mm Hg. d. Call the health care provider about a decrease in urine output.

B Rationale: An experienced LPN/LVN would be able to monitor BP and would know to report significant changes to the RN. Teaching patients and making adjustments to the drip rate for vasoactive medications are RN-level skills. Because the health care provider may order changes in therapy based on the decrease in urine output, the RN should call the health care provider about the decreased urine output.

When the nurse is developing a teaching plan to prevent the development of heart failure in a patient with stage 1 hypertension, the information that is most likely to improve compliance with antihypertensive therapy is that a. hypertensive crisis may lead to development of acute heart failure in some patients. b. hypertension eventually will lead to heart failure by overworking the heart muscle. c. high BP increases risk for rheumatic heart disease. d. high systemic pressure precipitates papillary muscle rupture.

B Rationale: Hypertension is a primary cause of heart failure because the increase in ventricular afterload leads to ventricular hypertrophy and dilation. Hypertensive crisis may precipitate acute heart failure is some patients, but this patient with stage 1 hypertension may not be concerned about a crisis that happens only to some patients. Hypertension does not directly cause rheumatic heart disease (which is precipitated by infection with group A -hemolytic streptococcus) or papillary muscle rupture (which is caused by myocardial infarction/necrosis of the papillary muscle).

A patient admitted to the hospital with an exacerbation of chronic heart failure tells the nurse, "I felt fine when I went to bed, but I woke up in the middle of the night feeling like I was suffocating!" The nurse can best document this assessment information as a. pulsus alternans. b. paroxysmal nocturnal dyspnea. c. two-pillow orthopnea. d. acute bilateral pleural effusion.

B Rationale: Paroxysmal nocturnal dyspnea is caused by the reabsorption of fluid from dependent body areas when the patient is sleeping and is characterized by waking up suddenly with the feeling of suffocation. Pulsus alternans is the alternation of strong and weak peripheral pulses during palpation. Orthopnea indicates that the patient is unable to lie flat because of dyspnea. Pleural effusions develop over a longer time period.

When developing a plan to decrease preload in the patient with heart failure, the nurse will include actions such as a. administering sedatives to promote rest and decrease myocardial oxygen demand. b. positioning the patient in a high-Fowler's position with the feet horizontal in the bed. c. administering oxygen per mask or nasal cannula. d. encouraging leg exercises to improve venous return.

B Rationale: Positioning the patient in a high-Fowler's position with the legs dependent will reduce preload by decreasing venous return to the right atrium. The other interventions may also be appropriate for patients with heart failure but will not help in decreasing preload.

Intravenous sodium nitroprusside (Nipride) is ordered for a patient with acute pulmonary edema. During the first hours of administration, the nurse will need to adjust the Nipride rate if the patient develops a. a drop in heart rate to 54 beats/min. b. a systolic BP <90 mm Hg. c. any symptoms indicating cyanide toxicity. d. an increased amount of ventricular ectopy.

B Rationale: Sodium nitroprusside is a potent vasodilator, and the major adverse effect is severe hypotension. After 48 hours of continuous use, cyanide toxicity is a possible (though rare) adverse effect. Reflex tachycardia (not bradycardia) is another adverse effect of this medication. Nitroprusside does not cause increased ventricular ectopy.

The nurse working in the heart failure clinic will know that teaching for a 74-year-old patient with newly diagnosed heart failure has been effective when the patient a. says that the nitroglycerin patch will be used for any chest pain that develops. b. calls when the weight increases from 124 to 130 pounds in a week. c. tells the home care nurse that furosemide (Lasix) is taken daily at bedtime. d. makes an appointment to see the doctor at least once yearly.

B Rationale: Teaching for a patient with heart failure includes information about the need to weigh daily and notify the health care provider about an increase of 3 pounds in 2 days or 5 pounds in a week. Nitroglycerin patches are used primarily to reduce preload (not to prevent chest pain) in patients with heart failure and should be used daily, not on an "as necessary" basis. Diuretics should be taken earlier in the day to avoid nocturia and sleep disturbance. Heart failure is a chronic condition that will require frequent follow-up rather than an annual health care provider examination.

A patient receiving the drug simvastatin (Zocor) should be taught this medication helps to prevent coronary heart disease by: Increasing lower-density lipoprotein. Controlling lower-density lipoprotein. Increasing triglycerides. Increasing very low-density lipoprotein.

B Rationale: The Heart Protective Study has also shown that controlling low-density lipoprotein (LDL) with simvastatin (Zocor) assists in the prevention of coronary heart disease by raising HDL. Responses 1, 3, and 4 are incorrect.

While admitting an 80-year-old patient with heart failure to the medical unit, the nurse obtains the information that the patient lives alone and sometimes confuses the "water pill" with the "heart pill." The nurse makes a note that discharge planning for the patient will need to include a. transfer to a dementia care service. b. referral to a home health care agency. c. placement in a long-term-care facility. d. arrangements for around-the-clock care.

B Rationale: The data about the patient suggest that assistance in developing a system for taking medications correctly at home is needed. A home health nurse will assess the patient's home situation and help the patient to develop a method for taking the two medications as directed. There is no evidence that the patient requires services such as dementia care, long-term-care, or around-the-clock home care.

A patient has been diagnosed with Right-Sided Congestive Heart Failure, and is confused about return of deoxygenated blood from the tissue. To clarify the confusion, which chamber of the heart receives blood from systemic circulation? Left atrium Right atrium Right ventricle Left ventricle

B Rationale: The right atrium is a thin-walled structure that receives deoxygenated blood from all the peripheral tissues by way of the superior and inferior vena cava and from the heart muscle by way of the coronary sinus.

Which of the following complications is of greatest concern when caring for a preoperative abdominal aneurysm client? A. HPN B. Aneurysm rupture C. Cardiac arrythmias D. Diminished pedal pulses

B Rupture of the aneurysm is a life-threatening emergency and is of the greatest concern for the nurse caring for this type of client. Hypertension should be avoided and controlled because it can cause the weakened vessel to rupture. Diminished pedal pulses, a sign of poor circulation to the lower extremities, are associated with an aneurysm but isn't life threatening. Cardiac arrhythmias aren't directly linked to an aneurysm.

Which of the following groups of symptoms indicated a ruptured abdominal aneurysm? A. Lower back pain, increased BP, decreased RBC, increased WBC B. Severe lower back pain, decreased BP, decreased RBC, increased WBC C. Severe lower back pain, decreased BP, decreased RBC, decreased WBC D. Intermittent lower back pain, decreased BP, decreased RBC, increased WBC

B Severe lower back pain indicates an aneurysm rupture, secondary to pressure being applied within the abdominal cavity. When rupture occurs, the pain is constant because it can't be alleviated until the aneurysm is repaired. Blood pressure decreases due to the loss of blood. After the aneurysm ruptures, the vasculature is interrupted and blood volume is lost, so blood pressure wouldn't increase. For the same reason, the RBC count is decreased - not increase. The WBC count increases as cells migrate to the site of injury.

A patient who has had an acute myocardial infarction (AMI) asks the nurse about when sexual intercourse can be resumed. Which response by the nurse is best? a. "Most patients are able to enjoy intercourse without any complications." b. "Sexual activity uses about as much energy as climbing two flights of stairs." c. "The doctor will discuss sexual intercourse when your heart is strong enough." d. "Holding and cuddling are good ways to maintain intimacy after a heart attack."

B Sexual activity places about as much physical stress on the cardiovascular system as climbing two flights of stairs. The other responses do not directly address the patient's question, or may not be accurate for this patient.

When teaching a patient why spironolactone (Aldactone) and furosemide (Lasix) are prescribed together, the nurse bases teaching on the knowledge that: A. Moderate doses of two different types of diuretics are more effective than a large dose of one type B. This combination promotes diuresis but decreases the risk of hypokalemia C. This combination prevents dehydration and hypovolemia D. Using two drugs increases osmolality of plasma and the glomerular filtration rate

B Spironolactone is a potassium-sparing diuretic; furosemide is a potassium-losing diuretic. Giving these together minimizes electrolyte imbalance.

Stimulation of the sympathetic nervous system produces which of the following responses? A. Bradycardia B. Tachycardia C. Hypotension D. Decreased myocardial contractility

B Stimulation of the sympathetic nervous system causes tachycardia and increased contractility. The other symptoms listed are related to the parasympathetic nervous system, which is responsible for slowing the heart rate.

When admitting a patient for a coronary arteriogram and angiogram, which information about the patient is most important for the nurse to communicate to the health care provider? a. The patient's pedal pulses are +1. b. The patient is allergic to shellfish. c. The patient has not eaten anything today. d. The patient had an arteriogram a year ago.

B The contrast dye used for the procedure is iodine based, so patients who have shellfish allergies will require treatment with medications such as corticosteroids and antihistamines before the arteriogram. The other information also is communicated to the health care provider but will not require a change in the usual prearteriogram orders or medications.

Which of the following landmarks is the correct one for obtaining an apical pulse? A. Left intercostal space, midaxillary line B. Left fifth intercostal space, midclavicular line C. Left second intercostal space, midclavicular line D. Left seventh intercostal space, midclavicular line

B The correct landmark for obtaining an apical pulse is the left intercostal space in the midclavicular line. This is the point of maximum impulse and the location of the left ventricular apex. The left second intercostal space in the midclavicular line is where the pulmonic sounds are auscultated. Normally, heart sounds aren't heard in the midaxillary line or the seventh intercostal space in the midclavicular line.

A 24-year old man seeks medical attention for complaints of claudication in the arch of the foot. A nurse also notes superficial thrombophlebitis of the lower leg. The nurse would next assess the client for: a. Familial tendency toward peripheral vascular disease b. Smoking history c. Recent exposures to allergens d. History of insect bites

B The mixture of arterial and venous manifestations (claudication and phlebitis, respectively) in the young male client suggests Buerger's disease. This is an uncommon disorder characterized by inflammation and thrombosis of smaller arteries and veins. This disorder typically is found in young adult males who smoke. The cause is not known precisely but is suspected to have an autoimmune component.

Which action will the nurse implement for a patient who arrives for a calcium-scoring CT scan? a. Administer oral sedative medications. b. Teach the patient about the procedure. c. Ask whether the patient has eaten today. d. Insert a large gauge intravenous catheter.

B The nurse will need to teach the patient that the procedure is rapid and involves little risk. The other actions are not necessary.

In which of the following areas is an abdominal aortic aneurysm most commonly located? A. Distal to the iliac arteries B. Distal to the renal arteries C. Adjacent to the aortic branch D. Proximal to the renal arteries

B The portion of the aorta distal to the renal arteries is more prone to an aneurysm because the vessel isn't surrounded by stable structures, unlike the proximal portion of the aorta. Distal to the iliac arteries, the vessel is again surrounded by stable vasculature, making this an uncommon site for an aneurysm. There is no area adjacent to the aortic arch, which bends into the thoracic (descending) aorta.

When administered a thrombolytic drug to the client experiencing an MI, the nurse explains to him that the purpose of this drug is to: a. Help keep him well hydrated b. Dissolve clots he may have c. Prevent kidney failure d. Treat potential cardiac arrhythmias.

B Thrombolytic drugs are administered within the first 6 hours after onset of a MI to lyse clots and reduce the extent of myocardial damage.

A nurse notes that a client with sinus rhythm has a premature ventricular contraction that falls on the T wave of the preceding beat. The client's rhythm suddenly changes to one with no P waves or definable QRS complexes. Instead there are coarse wavy lines of varying amplitude. The nurse assesses this rhythm to be: A. Ventricular tachycardia B. Ventricular fibrillation C. Atrial fibrillation D. Asystole

B Ventricular fibrillation is characterized by irregular, chaotic undulations of varying amplitudes. Ventricular fibrillation has no measurable rate and no visible P waves or QRS complexes and results from electrical chaos in the ventricles.

A nurse is watching the cardiac monitor and notices that the rhythm suddenly changes. There are no P waves, the QRS complexes are wide, and the ventricular rate is regular but over 100. The nurse determines that the client is experiencing: A. Premature ventricular contractions B. Ventricular tachycardia C. Ventricular fibrillation D. Sinus tachycardia

B Ventricular tachycardia is characterized by the absence of P waves, wide QRS complexes (usually greater than 0.14 second), and a rate between 100 and 250 impulses per minute. The rhythm is usually regular.

A client who has been receiving heparin therapy also is started on warfarin sodium (coumadin). The client asks the nurse why both medications are being administered. In formulating a response, the nurse incorporates the understanding that warfarin sodium: a. Stimulates the breakdown of specific clotting factors by the liver, and it takes 2-3 days for this is exhibit an anticoagulant effect. b. Inhibits synthesis of specific clotting factors in the liver, and it takes 3 to 4 days for this medication to exert an anticoagulation effect. c. Stimulates production of the body's own thrombolytic substances, but it takes 2-4 days for it to begin. d. Has the same mechanism action of heparin, and the crossover time is needed for the serum level of warfarin sodium to be therapeutic.

B Warfarin sodium works in the liver and inhibits synthesis of four vitamin K-dependent clotting factors (X, IX, VII, and II), but it takes 3 to 4 days before the therapeutic effect of warfarin is exhibited.

A client who has been receiving heparin therapy also is started on warfarin. The client asks a nurse why both medications are being administered. In formulating a response, the nurse incorporates the understanding that warfarin: a. Stimulates the breakdown of specific clotting factors by the liver, and it takes 2-3 days for this to exert an anticoagulant effect. b. Inhibits synthesis of specific clotting factors in the liver, and it takes 3-4 days for this medication to exert an anticoagulant effect. c. Stimulates production of the body's own thrombolytic substances, but it takes 2-4 days for this to begin. d. Has the same mechanism of action as Heparin, and the crossover time is needed for the serum level of warfarin to be therapeutic.

B Warfarin works in the liver and inhibits synthesis of four vitamin K-dependent clotting factors (X, IX, VII, and II), but it takes 3 to 4 days before the therapeutic effect of warfarin is exhibited.

A few days after experiencing a myocardial infarction (MI), the patient states, "I just had a little chest pain. As soon as I get out of here, I'm going for my vacation as planned." Which response should the nurse make? a. "Where are you planning to go for your vacation?" b. "What do you think caused your chest pain episode?" c. "Sometimes plans need to change after a heart attack." d. "Recovery from a heart attack takes at least a few weeks."

B When the patient is experiencing denial, the nurse should assist the patient in testing reality until the patient has progressed beyond this step of the emotional adjustment to MI. Asking the patient about vacation plans reinforces the patient's plan, which is not appropriate in the immediate post-MI period. Reminding the patient in denial about the MI is likely to make the patient angry and lead to distrust of the nursing staff.

The client is prescribed digoxin (Lanoxin) for treatment of HR. Which of the following statements by the client indicates the need for further teaching by the nurse? A. "I should not get short of breath anymore." B "This drug will help my heart muscle pump less." C. "I may notice my heart rate decrease." D. "I may feel tired during early treatment."

B. "This drug will help my heart muscle pump less." Rationale: The ability to increase the strength of contractions is a characteristic of cardiac glycosides. It may result in a decrease in pulse. Initially the client may experience some fatigue. Symptoms of CHF, such as dyspnea, should improve.

Lisinopril (Prinivil) is part of the treatment regimen for a client with HF. The nurse monitors the client for which electrolyte imbalance of this drug? A. Hyponatremia B. Hyperkalemia C. Hypokalemia D. Hypernatremia

B. Hyperkalemia Rationale: ACE inhibitors block aldosterone secretion, which results in sodium loss and potassium retention. Hyperkalemia may occur, especially when the drug is taken concurrently with potassium-sparing diuretics.

Conditions such as shock and severe dehydration resulting from extracellular fluid loss cause: A. Hypoxia B. Hypovolemia C. Hypervolemia D. Uncontrolled bleeding

B. Hypovolemia Conditions such as shock and severe dehydration cause extracellular fluid loss and reduced circulating blood volume (hypovolemia).

Patients with a heart transplantation are at risk for which complications in the first year after transplantation (select all that apply)? a. cancer b. infection c. rejection d. vasculopathy e. sudden cardiac death

BCE Rationale: A variety of complications can occur after heart transplantation. In the first year after transplantation, the major causes of death are acute rejection and infection. Heart transplant recipients also are at risk for sudden cardiac death. Later, malignancy (especially lymphoma) and cardiac vasculopathy (accelerated CAD) are major causes of death.

The nurse suspects left-sided heart failure in a newly admitted client when the nurse notes which of the following symptoms? (Select all that apply.) A. Distended neck veins B. Bilateral crackles in the lungs C. Weight gain of 2 lb in past 2 days D. Shortness of breath, especially at night .

BD Left-sided heart failure results in ineffective ejection of blood from the left ventricle. This causes a backup of blood into the lungs. Thus, symptoms of left-sided heart failure are usually related to the lungs

Which are probable clinical findings in a person with an acute lower extremity VTE (select all that apply)? a. Pallor and coolness of foot and calf b. Mild to moderate calf pain and tenderness c. Grossly diminished or absent pedal pulses d. Unilateral edema and induration of the thigh e. Palpable cord along a superficial varicose vein

BD Rationale: The patient with lower extremity venous thromboembolism (VTE) may or may not have unilateral leg edema, extremity pain, a sense of fullness in the thigh or calf, paresthesias, warm skin, erythema, or a systemic temperature greater than 100.4 F (38 C). If the calf is involved, it may be tender to palpation.

Select all that apply. Which of the following are characteristic manifestations of paroxysmal nocturnal dyspnea (PND)? A. Vomiting B. Nighttime awakening C. Migraine-type headache D. Episodes of snoring accompanied by apnea E. Relief of symptoms when the patient sits upright F. Occurrence 2 to 3 hours after the patient goes to sleep

BEF

A 38-year-old man is treated for hypertension with amiloride/hydrochlorothiazide (Maxzide) and metoprolol (Lopressor). Four months after his last clinic visit, his BP returns to pretreatment levels and he admits he has not been taking his medication regularly. The best response by the nurse is, A. "Try always to take your medication when you carry out another daily routine so you do not forget to take it." B. "If you would exercise more and stop smoking, you probably would not need to be taking medications for hypertension." C. "The drugs you are taking cause sexual dysfunction in many patients. Are you experiencing any problems in this area?" D. "You need to remember that hypertension can be only controlled with medication, not cured, and you must always take your medication."

C

A 62-year-old woman weighs 92 kg and has a history of daily alcohol intake, smoking, high blood pressure, high sodium intake, and sedentary lifestyle. The nurse identifies the risk factors most highly related to peripheral arterial disease in this patient as A. sex and age. B. weight and alcohol intake. C. cigarette smoking and hypertension. D. sedentary lifestyle and high sodium intake.

C

A cardiac/vascular nurse reviews recommended activities with a patient who sustained a myocardial infarction. The patient states, "It doesn't really matter what I do or don't do. I will either get better or die." This statement reflects: A. acceptance of changed health status. B. an internal locus of control. C. feelings of loss of control. D. projection.

C

A client has been diagnosed with a myocardial infarction that has damaged a part of the right atrium. Which of the following could happen as a result of this damage? a. Increase in collateral circulation b. Nothing c. A change in the rate and rhythm of the client's heartbeat d. Onset of aortic regurgitation

C

A client presenting to the ER with chest pain and dizziness was found to be having a MI and subsequently suffered cardiac arrest. The ER health care team was able to successfully resuscitate the client. Lab work shows that the client is now acidotic. The nurse understands that the acidic serum pH most likely is caused from: A. fat forming ketoacids that are broken down. B. The client receiving too much sodium bicarbonate during resuscitation efforts. C. The decreased tissue perfusion that subsequently caused lactic acid production. D. An irregular heartbeat the client experienced during cardiac arrest.

C

A client receives a dose of furosemid (Lasix) 120 mg intravenously for treatment of congestive heart failure. Which symptom indicates that an adverse reaction is most likely occurring? A. bradycardia B. weight gain C. hypotension D. crackles

C

A client who is diagnosed as having a MI is admitted to the coronary care unit with prescriptions for bed rest and medication for chest pain. Within an hour after admission, the nurse finds the client walking around the unit. What is the nurse's best initial response? A. "Tell me what you are doing out of bed?" B. "It must be frustrating to be confined in bed." C. "You need to rest. You should get back into bed." D. "Please get back into bed immediately. The health care provider wants you to rest."

C

A client with a coronary occlusion is experiencing chest pain and distress. What is the primary reason that the nurse should administer oxygen to this client? A. Prevent dyspnea B. Prevent cyanosis C. Increase oxygen concentration to heart cells D. Increase oxygen tension in the circulating blood

C

A compensatory mechanism involved in congestive heart failure that leads to inappropriate fluid retention and additional workload of the heart is A. ventricular dilation. B. ventricular hypertrophy. C. neurohormonal response. D. sympathetic nervous system activation.

C

A healthcare provider orders NTG to be administered by IV drip. The nurse carrying out the order would monitor for which of the following as a priority specific to medication? A. shortness of breath when raising head of bed B. urine output C. blood pressure and heart rate D. facial flushing and headache

C

A nurse teaching a pt on correct self administration of sublingual nitroglycerin (Nitrostat) tablets realizes the pt requires further teaching when they state: A. I will call an ambulance while administering the 3rd tablet B. I will not drive while taking nitroglycerin tablets C. I will keep my tablets in a clear bottle D. I will not take nitroglycerin with erectile dysfunction medication

C

A nurse with adequate knowledge about the administration of IV nitroglycerin will recognize that which of the following statements is correct? A. the intravenous form is given by bolus injection B. because the IV forms are short-lived, the dosing must be every 2 hours C. IV nitroglycerin must be protected form exposure to light through use of special tubing D. IV nitroglycerin can be given via gravity drip infusions

C

A nurse, explaining the cardiac circulation to cardiac rehabilitation clients, wants to include the oxygenation of the heart muscle. Which of the following structures carries deoxygenated blood to the lungs? a. Right main coronary vein b. Pulmonary vein c. Pulmonary artery d. Great cardiac vein

C

A patient admitted to the coronary care unit (CCU) with an ST-segment-elevation myocardial infarction (STEMI) is restless and anxious. The blood pressure is 86/40 and heart rate is 123. Based on this information, which nursing diagnosis is a priority for the patient? a. Acute pain related to myocardial infarction b. Anxiety related to perceived threat of death c. Stress overload related to acute change in health d. Decreased cardiac output related to cardiogenic shock

C

A patient is admitted to the CCU with chest pain of 24 hours' duration, ECG findings consistent with an acute MI, and occasional ventricular arrhythmias. The nurse plans care for the patient based on the expectation that the patient will be managed with A. endotracheal intubation. B. subcutaneous nitroglycerin. C. continuous ECG monitoring. D. thrombolytic therapy with tissue plasminogen activator.

C

A patient is experiencing a heart rate of 200 beats/min. If the ECG pattern demonstrates absent P waves and normal and consistent QRS complexes and duration, a nurse should interpret these findings as indicative of A. ventricular tachycardia. B. second-degree heart block. C. supraventricular tachycardia. D. premature ventricular contractions

C

A patient is recovering from a myocardial infarction (MI) and develops chest pain on day 3 that increases when taking a deep breath and is relieved by leaning forward. Which action should the nurse take next? a. Assess the feet for pedal edema. b. Palpate the radial pulses bilaterally. c. Auscultate for a pericardial friction rub. d. Check the heart monitor for dysrhythmias.

C

A patient is scheduled for exercise nuclear imaging stress testing. The nurse explains to the patient that this test involves A. placement of electrodes inside the right-sided heart chambers through a vein to record the electrical activity of the heart directly. B. exercising on a treadmill or stationary bicycle with continuous ECG (electrocardiographic) monitoring to detect ischemic changes during exercise. C. intravenous (IV) administration of a radioisotope of technetium-99 sestamibi at the maximum HR during exercise to identify areas of cardiac damage. D. placement of a small transducer in four positions on the chest to record the direction and flow of blood through the heart by the reflection of sound waves.

C

A patient who has chest pain is admitted to the emergency department (ED) and all of the following are ordered. Which one should the nurse arrange to be completed first? a. Chest x-ray b. Troponin level c. Electrocardiogram (ECG) d. Insertion of a peripheral IV

C

A patient with ST-segment elevation in three contiguous electrocardiographic (ECG) leads is admitted to the emergency department (ED) and diagnosed as having an ST-segment-elevation myocardial infarction (STEMI). Which question should the nurse ask to determine whether the patient is a candidate for thrombolytic therapy? a. "Do you have any allergies?" b. "Do you take aspirin on a daily basis?" c. "What time did your chest pain begin?" d. "Can you rate your chest pain using a 0 to 10 scale?"

C

A patient with a history of coronary artery disease is being treated for a myocardial infarction (MI). During treatment, acute mitral valve regurgitation occurs. What is the most likely cause of the acute mitral valve dysfunction? Please choose from one of the following options. A. Ventricular fibrillation B. Infective endocarditis C. Rupture of the chordae tendinae D. Atherosclerosis

C

A patient with a tricuspid valve disorder will have impaired blood flow between the A. vena cava and right atrium. B. left atrium and left ventricle. C. right atrium and right ventricle. D. right ventricle and pulmonary artery.

C

A patient with an MI of the anterior wall of the left ventricle most likely has an occlusion of then A. right marginal artery. B. left circumflex artery. C. left anterior descending artery. D. right anterior descending artery.

C

A patient with diabetes mellitus and chronic stable angina has a new order for captopril (Capoten). The nurse should teach the patient that the primary purpose of captopril is to a. lower heart rate. b. control blood glucose levels. c. prevent changes in heart muscle. d. reduce the frequency of chest pain.

C

A pt has a new prescription from transdermal nitroglycerin patches. The nurse teaches the pt that these patches are most appropriately used for which of the following? A. to relieve exertional angina B. to prevent palpatations C. to prevent the occurrence of angina D. to reduce the severity of anginal episodes

C

A pt is diagnosed with an acute myocardial infarction and is receiving tissue plasminogen activator, alteplase (Activase). Which of the following is a priority nursing intervention? A. monitor for renal failure B. monitor for psychosocial status C. monitor for signs of bleeding D. have heparin sodium available

C

A pt is taking digoxin and furosemide (Lasix) to manage congestive heart failure. The nurse determiens that the pt understands diet therapy when the pt makes which meal choice? A. veggie beef soup, mac and cheese, and a roll B. beef ravioli w/ bread C. baked white fish, mashed potatoes, and carrot salad D. roasted chicken, brown rice, and stewed tomatoes

C

A student nurse is asked to give an example of a long-acting nitrate. She is correct by saying: A. Nitroglycerin sublingual B. nitroglycerin IV C. isosorbide PO D. Nitroglycerin transmucosal

C

After reviewing information shown in the accompanying figure from the medical records of a 43-year-old, which risk factor modification for coronary artery disease should the nurse include in patient teaching? a. Importance of daily physical activity b. Effect of weight loss on blood pressure c. Dietary changes to improve lipid levels d. Ongoing cardiac risk associated with history of tobacco use

C

After the nurse has finished teaching a patient about the use of sublingual nitroglycerin (Nitrostat), which patient statement indicates that the teaching has been effective? a. "I can expect some nausea as a side effect of nitroglycerin." b. "I should only take the nitroglycerin if I start to have chest pain." c. "I will call an ambulance if I still have pain after taking 3 nitroglycerin 5 minutes apart." d. "Nitroglycerin helps prevent a clot from forming and blocking blood flow to my heart."

C

An ECG is prescribed for a client who reports chest pain. What early finding does the nurse expect on the lead over the infarcted area? A. Flattened T waves B. Absence of P waves C. Elevated ST segments D. Disappearance of Q waves

C

An elderly client receives instructions regarding the use of warfarin sodium (Coumadin). Which statement indicates the client understands the possible food interactions which may occur with this medication? A. I'm going to miss having my evening glass of wine now B. I told my daughter to buy bananas for me. I'll have to eat more of those now C. I will have to watch my intake of salads, something that I really love D. I am going to begin eating more fish and pork and leave beef alone now

C

Distant, muffled heart sounds and distended neck veins may indicate A. flail chest. B. ruptured spleen. C. cardiac tamponade. D. thoracic vertebral fracture.

C

During the administration of the thrombolytic agent to a patient with an acute myocardial infarction (AMI), the nurse should stop the drug infusion if the patient experiences a. bleeding from the gums. b. increase in blood pressure. c. a decrease in level of consciousness. d. a nonsustained episode of ventricular tachycardia.

C

Following an acute myocardial infarction (AMI), a patient ambulates in the hospital hallway. When the nurse is evaluating the patient's response to the activity, which assessment data would indicate that the exercise level should be decreased? a. Blood pressure (BP) changes from 118/60 to 126/68 mm Hg. b. Oxygen saturation drops from 99% to 95%. c. Heart rate increases from 66 to 92 beats/minute. d. Respiratory rate goes from 14 to 20 breaths/minute.

C

Following an ascending aortic aneurysm repair, the nurse monitors for and immediately reports: a. shallow resp and poor coughing b. decreased drainaged from chest tubes c. change in level of consciousness and ability to speak d. lower extremity pulses that are decreased from pre-op baseline

C

Heparin is ordered for a patient with a non-ST-segment-elevation myocardial infarction (NSTEMI). What is the purpose of the heparin? a. Heparin enhances platelet aggregation. b. Heparin decreases coronary artery plaque size. c. Heparin prevents the development of new clots in the coronary arteries. d. Heparin dissolves clots that are blocking blood flow in the coronary arteries.

C

If a nurse is teaching foot care to a patient with peripheral vascular disease, which of the following is a correctly written learning objective? A. "The nurse will instruct the patient on appropriate foot care." B. "The nurse will demonstrate to the patient the proper technique for trimming toenails." C. "By discharge, the patient will list three ways to protect the feet from injury." D. "The patient will understand the rationale for proper foot care after instruction

C

If the Purkinje system is damaged, conduction of the electrical impulse is impaired through the A. atria. B. AV node. C. ventricles. D. bundle of His.

C

In planning activity for the patient recovering from an MI, the nurse recognizes that the healing heart wall is most vulnerable to stress A. 3 weeks after the infarction. B. 4 to 6 days after the infarction. C. 10 to 14 days after the infarction. D. when healing is complete at 6 to 8 weeks.

C

In preparation for discharge, the nurse teaches a patient with chronic stable angina how to use the prescribed short-acting and long-acting nitrates. Which patient statement indicates that the teaching has been effective? a. "I will check my pulse rate before I take any nitroglycerin tablets." b. "I will put the nitroglycerin patch on as soon as I get any chest pain." c. "I will stop what I am doing and sit down before I put the nitroglycerin under my tongue." d. "I will be sure to remove the nitroglycerin patch before taking any sublingual nitroglycerin."

C

In teaching a patient about coronary artery disease, the nurse explains that the changes that occur in this disorder involve A. diffuse involvement of plaque formation in coronary veins. B. formation of fibrous tissue around coronary artery orifices. C. accumulation of lipid and fibrous tissue within the coronary arteries. D. chronic vasoconstriction of coronary arteries leading to permanent vasospasm.

C

Significant risk factors for peripheral arterial disease include A. sedentary lifestyle, stress, obesity. B. advanced age, female gender, familial tendency. C. cigarette smoking, hyperlipidemia, hypertension. D. protein S deficiency, protein C deficiency, factor V Leiden mutation.

C

The action of medication is inotropic when it: A. decreases afterload B. increases heart rate C. increases the force of contraction D. is uses to treat CHF

C

The client being treated for uncontrolled hypertension and chest pain calls out to the nurse and reports he or she is having a nosebleed. Upon entry to the clients room, immediately applies pressure. Which action should the nurse take next? A. Add humidity to the clients oxygen prescribed at 2 L/minute via nasal cannula B. Assess the client for further injuries indicative of a possible fall C. Auscultate the clients blood pressure D. Assess the clients pulse rate

C

The most common pathologic finding in individuals with sudden cardiac death is A. cardiomyopathies. B. mitral valve disease. C. atherosclerotic heart disease. D. left ventricular hypertrophy.

C

The nurse caring for a client admitted for chest pain and a MI is preparing to apply nitroglycerin ointment. Before applying, the nurse should: A. Assess the client's pulse rate B. Prepare the site with an alcohol swab C. Remove ointment previously applied D. Expect the client to be relieved of pain within 20 minutes

C

The nurse is caring for a client who has undergone cardiac catheterization. The client says to the nurse, "The doctor said my cardiac output was 5.5 L/min. What is normal cardiac output?" Which of the following is the nurse's best response? A. "It is best to ask your doctor." B. "Did the test make you feel upset?" C. "The normal cardiac output for an adult is 4 to 6 L/min." D. "Are you able to explain why are you asking this question?"

C

The nurse is instructing a client about his high cholesterol level and wants to include behavioral considerations. Which of the following should be included in this instruction? a. The need for annual flu inoculation b. The need for an annual cholesterol panel c. The need to stop smoking d. The need to reduce stress

C

The nurse obtains the following data when assessing a patient who experienced an ST-segment-elevation myocardial infarction (STEMI) 2 days previously. Which information is most important to report to the health care provider? a. The troponin level is elevated. b. The patient denies ever having a heart attack. c. Bilateral crackles are auscultated in the mid-lower lobes. d. The patient has occasional premature atrial contractions (PACs).

C

The nurse plans care for the patient with dilated cardiomyopathy based on the knowledge that A. family members may be at risk because of the infectious nature of the disease. B. medical management of the disorder focuses on treatment of the underlying cause. C. the prognosis of the patient is poor, and emotional support is a high priority of care. D. the condition may be successfully treated with surgical ventriculomyotomy and myectomy.

C

The nurse plans close monitoring for the patient during electrophysiologic testing because this test A. requires the use of dyes that irritate the myocardium. B. causes myocardial ischemia resulting in dysrhythmias. C. induces dysrhythmias that may require defibrillation to correct. D. involves the use of anticoagulants to prevent thrombus and embolism.

C

The nurse suspects the presence of a deep vein thrombosis based on the findings of A. paresthesia and coolness of the leg. B. pain in the calf that occurs with exercise. C. generalized edema of the involved extremity. D. pallor and cyanosis of the involved extremity.

C

Warfarin works by obstructing certain clotting factors in the clotting cascade. Which info is correct regarding this mechanism of action? A. interference with calcium occurs within the clotting cascade B. increased solubility of vitamin D occurs in the mucosal lining of the stomach C. decreased functioning of vitamin K occurs within the production sites D. binding with magnesium occurs in the hepatic cells

C

What is the antidote for heparin? A. vitamin K B. immunefab C. protamine sulfate

C

When a patient reports chest pain, unstable angina must be identified and treated because A. the pain may be severe and disabling. B. ECG changes and dysrhythmias may occur during an attack. C. atherosclerotic plaque deterioration may cause complete thrombus of the vessel lumen. D. the spasm of a major coronary artery may cause total occlusion of the vessel with progression to MI.

C

When a person's blood pressure rises, the homeostatic mechanism to compensate for an elevation involves stimulation of A. chemoreceptors that inhibit the sympathetic nervous system, causing vasodilation. B. baroreceptors that inhibit the parasympathetic nervous system, causing vasodilation. C. baroreceptors that inhibit the sympathetic nervous system, causing a decreased heart rate. D. chemoreceptors that stimulate the sympathetic nervous system, causing an increased heart rate.

C

When assessing a pt who has been taking amiodarone for 6 months, which adverse reaction might the nurse identify? A. glycosuria B. dysphagia C. photophobia D. urticaria

C

When assessing the cardiovascular system of a 79-year-old patient, the nurse expects to find A. a narrowed pulse pressure. B. diminished carotid artery pulses. C. difficulty in isolating the apical pulse. D. an increased heart rate in response to stress.

C

When caring for a patient who has started anticoagulant therapy with warfarin (Coumadin), the nurse knows not to expect therapeutic benefits for: a. At least 12 hours b. The first 24 hours c. 2-3 days d. 1 week

C

When nitrates are administered early to the acute MI pt, the result is: A. Hypotension B. Bradycardia C. reduced mortality D. reduced morbidity

C

When nitrates are administered early to the acute MI pt, the result is: A. Hypotension B. Bradycardia C. reduced mortality D. reduced morbidity

C

Which electrocardiographic (ECG) change is most important for the nurse to report to the health care provider when caring for a patient with chest pain? a. Inverted P wave b. Sinus tachycardia c. ST-segment elevation d. First-degree atrioventricular block

C

Which nursing intervention will be most effective when assisting the patient with coronary artery disease (CAD) to make appropriate dietary changes? a. Give the patient a list of low-sodium, low-cholesterol foods that should be included in the diet. b. Emphasize the increased risk for heart problems unless the patient makes the dietary changes. c. Help the patient modify favorite high-fat recipes by using monosaturated oils when possible. d. Inform the patient that a diet containing no saturated fat and minimal salt will be necessary.

C

Which of the following characteristics is typical of the pain associated with DVT? A. Dull ache B. No pain C. Sudden onset D. Tingling

C

Which of the following is a contraindication for digoxin administration? A. BP of 140/90 B. HR>80 C. HR<60 D. RR>18

C

Which of the following serum abnormalities is unique to loop diuretics? A. hypokalemia B. hypomagnesia C. hyponatremia

C

Which of the following statements would indicate that a patient taking antihypertensive drugs understands the management of hypertension? A. "I need to have my blood pressure checked monthly." B. "I can still smoke while taking these drugs, as long as I cut down." C. "These pills will help control, but not cure, my high blood pressure." D. "When my blood pressure is back to normal, I can stop taking these pills."

C

You know that the action of nitrates is: A. smooth muscle contraction B. vasoconstriction C. smooth muscle relaxation D. increase preload

C

Your pt is on coumadin. You must instruct them to limit their intake of what? A. milk B. juice C. tea

C

a pt has been admitted with chest pain. He states that it began while he was watching tv on his couch. Which kind of angina does he likely have? A. stable B. unstable C. variant

C

a pt with congestive heart failure is receiving digoxin. What is the desired effect: A. neck vein distention B. decreased appetite C. increased urinary output D. increased pedal edema

C

a pt with congestive heart failure is receiving digoxin. What is the desired effect: A. neck vein distention B. decreased appetite C. increased urinary output D. increased pedal edema

C

alpha-adrenergic blocking agents include medications such as doxazosin and prazosin hydrochloride. The mechanism of action for these meds is to: A. inhibit the parasympathetic system B. stimulate the sympathetic system C. inhibit the sympathetic system D. stimulate the parasympathetic system

C

during assessment of a pt who is receiving digoxin, which finding would indicate an increased possibility of toxicity? A. apical pulse rate of 60 bpm B. digoxin level of 1.5 C. serum potassium level of 2.0 D. serum potassium level of 4.8

C

nursing interventions for a pt receiving enoxaparin (Lovenox) may include: A. monitoring multiple lab tests and values B. monitoring for development of deep vein thrombosis C. teaching the pt or family to give subcutaneous injections at home D. teaching to observe for excessive bleeding

C

positive inotropic action does which of the following? A. decreases heart rate B. decreases cellular conduction C. increases contractility

C

the nurse knows that the mechanism of action of Clopidogrel (Plavix) is to: A. inhibit the actions of vitamin K B. turn plasminogen to plasmin C. inhibit the aggregation of platelets D. inhibit angiotensin I from converting to angiotensin II

C

the pt states, "I always put my nitroglycerine patch in the same place so I do not forget to take it off." Which of the following is the best response by the nurse? A. change the patch every 24 hours B. massage into the skin if ointment comes in contact with your hands C. rotate the NTG patch to a different hairless area each day D. after removing the patch, scrub the are vigorously with soap and water

C

when a pt is being taught about the potential adverse effects of an ACE inhibitor, which of the following should be mentioned as possibly occurring when this drug is taken to treat hypertension A. hypokalemia B. nausea C. dry, nonproductive cough D. sedation

C

which dietary change must a pt make when starting treatment with the med spironolactone? A. eat extra helpings of bananas B. increase intake of water C. avoid salt substitutes D. increase intake of green leafy veggies

C

which of the following drugs are known as "clot busters"? A. anticoagulants B. antiplatelets C. thrombolytics

C

which of the following is NOT a nursing intervention relevant to administration of nitroglycerin? A. inform the pt there may be a burning sensation under their tongue upon ingestion B. inform the pt that this med must be kept in its original bottle C. inform the pt that a headache is the most common side effect associated with this drug

C

which of the following is a class of antidysrhythmic drug? A. osmotic B. carbonic-anhydrase inhibitor C. beta adrenergic blocker

C

What is the priority assessment by the nurse caring for a patient receiving IV nesiritide (Natrecor) to treat heart failure? a. Urine output b. Lung sounds c. Blood pressure d. Respiratory rate

C Although all identified assessments are appropriate for a patient receiving IV nesiritide, the priority assessment would be monitoring for hypotension, the main adverse effect of nesiritide.

The nurse would determine that a postoperative patient is not receiving the beneficial effects of enoxaparin (Lovenox) after noting what during a routine shift assessment? a. Generalized weakness and fatigue b. Crackles bilaterally in the lung bases c. Pain and swelling in lower extremity d. Abdominal pain with decreased bowel sounds

C Rationale: Enoxaparin is a low-molecular-weight heparin used to prevent the development of deep vein thromboses (DVTs) in the postoperative period. Pain and swelling in the lower extremity can indicate development of DVT and therefore may signal ineffective medication therapy.

Priority nursing measures after an abdominal aortic aneurysm repair include a. assessment of cranial nerves and mental status. b. administration of IV heparin and monitoring of aPTT. c. administration of IV fluids and monitoring of kidney function. d. elevation of the legs and application of elastic compression stockings

C Rationale: Postoperative priorities include administration of IV fluids and maintenance of renal perfusion. An adequate blood pressure is important for maintaining graft patency, and administration of IV fluids and blood components (as indicated) is essential for adequate blood flow. The nurse should evaluate renal function by measuring hourly urine output and monitoring daily blood urea nitrogen (BUN) and serum creatinine levels. Irreversible renal failure may occur after aortic surgery, particularly in individuals at high risk.

A patient with chronic heart failure who has been following a low-sodium diet tells the nurse at the clinic about a 5-pound weight gain in the last 3 days. The nurse's first action will be to a. ask the patient to recall the dietary intake for the last 3 days because there may be hidden sources of sodium in the patient's diet. b. instruct the patient in a low-calorie, low-fat diet because the weight gain has likely been caused by excessive intake of inappropriate foods. c. assess the patient for clinical manifestations of acute heart failure because an exacerbation of the chronic heart failure may be occurring. d. educate the patient about the use of diuretic therapy because it is likely that the patient will need medications to reduce the hypervolemia.

C Rationale: The 5-pound weight gain over 3 days indicates that the patient's chronic heart failure may be worsening; it is important that the patient be immediately assessed for other clinical manifestations of decompensation, such as lung crackles. A dietary recall to detect hidden sodium in the diet and teaching about diuretic therapy are appropriate interventions but are not the first nursing actions indicated. There is no evidence that the patient's weight gain is caused by excessive dietary intake of fat or calories, so the answer beginning "instruct the patient in a low-calorie, low-fat diet" describes an inappropriate action

A patient with infective endocarditis develops sudden left leg pain with pallor, paresthesia, and a loss of peripheral pulses. The nurse's initial action should be to a. elevate the leg to promote venous return. b. start anticoagulant therapy with IV heparin. c. notify the physician of the change in peripheral perfusion. d. place the bed in reverse Trendelenburg to promote perfusion

C Rationale: The patient has potentially developed acute arterial ischemia (sudden interruption in the arterial blood supply to the extremity), caused by an embolism from a cardiac thrombus that occurred as a complication of infective endocarditis. Clinical manifestations of acute arterial ischemia include any or all of the six Ps : pain, pallor, paralysis, pulselessness, paresthesia, and poikilothermia. Without immediate intervention, ischemia may progress quickly to tissue necrosis and gangrene within a few hours. If the nurse detects these signs, the physician should be notified immediately.

The nurse is caring for a preoperative patient who has an order for vitamin K by subcutaneous injection. The nurse should verify that which laboratory study is abnormal before administering the dose? a. Hematocrit (Hct) b. Hemoglobin (Hgb) c. Prothrombin time (PT) d. Partial thromboplastin time (PTT)

C Rationale: Vitamin K counteracts hypoprothrombinemia and/or reverses the effects of warfarin (Coumadin) and thus decreases the risk of bleeding. High values for either the prothrombin time (PT) or the international normalized ratio (INR) demonstrates the need for this medication.

The nurse is caring for a patient who has been receiving warfarin (Coumadin) and digoxin (Lanoxin) as treatment for atrial fibrillation. Because the warfarin has been discontinued before surgery, the nurse should diligently assess the patient for which complication early in the postoperative period until the medication is resumed? a. Decreased cardiac output b. Increased blood pressure c. Cerebral or pulmonary emboli d. Excessive bleeding from incision or IV sites

C Rationale: Warfarin is an anticoagulant that is used to prevent thrombi from forming on the walls of the atria during atrial fibrillation. Once the medication is terminated, thrombi could again form. If one or more thrombi detach from the atrial wall, they could travel as cerebral emboli from the left atrium or pulmonary emboli from the right atrium.

A 50-year-old woman weighs 95 kg and has a history of tobacco use, high blood pressure, high sodium intake, and sedentary lifestyle. When developing an individualized care plan for her, the nurse determines that the most important risk factors for peripheral artery disease (PAD) that need to be modified are a.weight and diet. b.activity level and diet. c.tobacco use and high blood pressure. d.sedentary lifestyle and high blood pressure.

C Significant risk factors for peripheral artery disease include tobacco use, hyperlipidemia, elevated levels of high-sensitivity C-reactive protein, diabetes mellitus, and uncontrolled hypertension; the most important is tobacco use. Other risk factors include family history, hypertriglyceridemia, hyperuricemia, increasing age, obesity, sedentary lifestyle, and stress.

Which of the following diagnostic tests is preferred for evaluating heart valve function? Chest x-ray Duplex Doppler Echocardiogram Electrocardiogram

C Rationale: The echocardiogram is the preferred test to evaluate heart valves, because it allows the visualization of the valves as they open and close. A chest x-ray will determine the size of the heart, the duplex measures blood flow through major arteries, and an electrocardiogram identifies electrical activity.

A client enters the ER complaining of severe chest pain. A myocardial infarction is suspected. A 12 lead ECG appears normal, but the doctor admits the client for further testing until cardiac enzyme studies are returned. All of the following will be included in the nursing care plan. Which activity has the highest priority? a. Monitoring vital signs b. Completing a physical assessment c. Maintaining cardiac monitoring d. Maintaining at least one IV access site

C Even though initial tests seem to be within normal range, it takes at least 3 hours for the cardiac enzyme studies to register. In the meantime, the client needs to be watched for bradycardia, heart block, ventricular irritability, and other arrhythmias. Other activities can be accomplished around the MI monitoring.

Direct-acting vasodilators have which of the following effects on the heart rate? a. Heart rate decreases b. Heart rate remains significantly unchanged c. Heart rate increases d. Heart rate becomes irregular

C Heart rate increases in response to decreased blood pressure caused by vasodilation.

A client is experiencing tachycardia. The nurse's understanding of the physiological basis for this symptom is explained by which of the following statements? a. The demand for oxygen is decreased because of pleural involvement b. The inflammatory process causes the body to demand more oxygen to meet its needs. c. The heart has to pump faster to meet the demand for oxygen when there is lowered arterial oxygen tension. d. Respirations are labored.

C The arterial oxygen supply is lowered and the demand for oxygen is increased, which results in the heart's having to beat faster to meet the body's needs for oxygen. .

In order to prevent the development of tolerance, the nurse instructs the patient to: a. Apply the nitroglycerin patch every other day b. Switch to sublingual nitroglycerin when the patient's systolic blood pressure elevates to >140 mm Hg c. Apply the nitroglycerin patch for 14 hours each and remove for 10 hours at night d. Use the nitroglycerin patch for acute episodes of angina only

C Tolerance can be prevented by maintaining an 8- to 12-hour nitrate-free period each day.

Following an acute myocardial infarction (AMI), a patient ambulates in the hospital hallway. When the nurse is evaluating the patient's response, which of these assessment data would indicate that the exercise level should be decreased? a. BP changes from 118/60 to 126/68 mm Hg. b. Oxygen saturation drops from 100% to 98%. c. Heart rate increases from 66 to 90 beats/minute. d. Respiratory rate goes from 14 to 22 breaths/minute.

C A change in heart rate of more than 20 beats or more indicates that the patient should stop and rest. The increases in BP and respiratory rate, and the slight decrease in oxygen saturation, are normal responses to exercise.

A patient is scheduled for a cardiac catheterization with coronary angiography. Before the test, the nurse informs the patient that a. electrocardiographic (ECG) monitoring will be required for 24 hours after the test. b. it will be important to lie completely still during the procedure. c. a warm feeling may be noted when the contrast dye is injected. d. monitored anesthesia care will be provided during the procedure.

C A sensation of warmth or flushing is common when the iodine-based contrast material is injected, which can be anxiety-producing unless it has been discussed with the patient. The patient may receive a sedative drug before the procedure, but monitored anesthesia care is not used. ECG monitoring is used during the procedure to detect dysrhythmias, but there is not a risk for dysrhythmias after the procedure. The patient is not immobile during cardiac catheterization and may be asked to cough or take deep breaths.

A patient admitted to the coronary care unit (CCU) with an ST-segment-elevation myocardial infarction (STEMI) is restless and anxious. The blood pressure is 86/40 and heart rate is 110. Based on this information, which nursing diagnosis is a priority for the patient? a. Acute pain related to myocardial ischemia b. Anxiety related to perceived threat of death c. Decreased cardiac output related to cardiogenic shock d. Activity intolerance related to decreased cardiac output

C All the nursing diagnoses may be appropriate for this patient, but the hypotension indicates that the priority diagnosis is decreased cardiac output, which will decrease perfusion to all vital organs (e.g., brain, kidney, heart).

Which of these statements made by a patient with coronary artery disease after the nurse has completed teaching about the therapeutic lifestyle changes (TLC) diet indicates that further teaching is needed? a. "I will switch from whole milk to 1% or nonfat milk." b. "I like fresh salmon and I will plan to eat it more often." c. "I will miss being able to eat peanut butter sandwiches." d. "I can have a cup of coffee with breakfast if I want one."

C Although only 30% of the daily calories should come from fats, most of the fat in the TLC diet should come from monosaturated fats such as are found in nuts, olive oil, and canola oil. The patient can include peanut butter sandwiches as part of the TLC diet. The other patient comments indicate a good understanding of the TLC diet.

What is the most common complication of a myocardial infarction? A. Cardiogenic shock B. Heart failure C. Arrhythmias D. Pericarditis

C Arrhythmias, caused by oxygen deprivation to the myocardium, are the most common complication of an MI. cardiogenic shock, another complication of MI, is defined as the end stage of left ventricular dysfunction. The condition occurs in approximately 15% of clients with MI. Because the pumping function of the heart is compromised by an MI, heart failure is the second most common complication. Pericarditis most commonly results from a bacterial of viral infection but may occur after MI.

Which of the following complications of an abdominal aortic repair is indicated by detection of a hematoma in the perineal area? A. Hernia B. Stage 1 pressure ulcer C. Retroperitoneal rupture at the repair site D. Rapid expansion of the aneurysm

C Blood collects in the retroperitoneal space and is exhibited as a hematoma in the perineal area. This rupture is most commonly caused by leakage at the repair site. A hernia doesn't cause vascular disturbances, nor does a pressure ulcer. Because no bleeding occurs with rapid expansion of the aneurysm, a hematoma won't form.

A client with no history of cardiovascular disease comes into the ambulatory clinic with flu-like symptoms. The client suddenly complains of chest pain. Which of the following questions would best help a nurse to discriminate pain caused by a non-cardiac problem? A. "Have you ever had this pain before?" B. "Can you describe the pain to me?" C. "Does the pain get worse when you breathe in?" D. "Can you rate the pain on a scale of 1-10, with 10 being the worst?"

C Chest pain is assessed by using the standard pain assessment parameters. Options 1, 2, and 4 may or may not help discriminate the origin of pain. Pain of pleuropulmonary origin usually worsens on inspiration.

A client with no history of cardiovascular disease comes into the ambulatory clinic with flulike symptoms. The client suddenly complains of chest pain. Which of the following questions would best help a nurse to discriminate pain caused by a non-cardiac problem? "Have you ever had this pain before?" "Can you describe the pain to me?" "Does the pain get worse when you breathe in?" "Can you rate the pain on a scale of 1-10, with 10 being the worst?"

C Chest pain is assessed by using the standard pain assessment parameters. Options 1, 2, and 4 may or may not help discriminate the origin of pain. Pain of pleuropulmonary origin usually worsens on inspiration.

The most important long-term goal for a client with hypertension would be to: a. Learn how to avoid stress b. Explore a job change or early retirement c. Make a commitment to long-term therapy d. Control high blood pressure

C Compliance is the most critical element of hypertensive therapy. In most cases, hypertensive clients require lifelong treatment and their hypertension cannot be managed successfully without drug therapy. Stress management and weight management are important components of hypertension therapy, but the priority goal is related to compliance.

Which of the followng conditions is most commonly responsible for myocardial infarction? A. Aneurysm B. Heart failure C. Coronary artery thrombosis D. Renal failure

C Coronary artery thrombosis causes occlusion of the artery, leading to myocardial death. An aneurysm is an outpouching of a vessel and doesn't cause an MI. Renal failure can be associated with MI but isn't a direct cause. Heart failure is usually the result of an MI.

A client enters the ER complaining of severe chest pain. A myocardial infarction is suspected. A 12 lead ECG appears normal, but the doctor admits the client for further testing until cardiac enzyme studies are returned. All of the following will be included in the nursing care plan. Which activity has the highest priority? A. Monitoring vital signs B. Completing a physical assessment C. Maintaining cardiac monitoring D. Maintaining at least one IV access site

C Even though initial tests seem to be within normal range, it takes at least 3 hours for the cardiac enzyme studies to register. In the meantime, the client needs to be watched for bradycardia, heart block, ventricular irritability, and other arrhythmias. Other activities can be accomplished around the MI monitoring.

A nurse is viewing the cardiac monitor in a client's room and notes that the client has just gone into ventricular tachycardia. The client is awake and alert and has good skin color. The nurse would prepare to do which of the following? A. Immediately defibrillate B. Prepare for pacemaker insertion C. Administer amiodarone (Cordarone) intravenously D. Administer epinephrine (Adrenaline) intravenously

C First-line treatment of ventricular tachycardia in a client who is hemodynamically stable is the use of anti-dysrhythmics such as amiodarone (Cordarone), lidocaine (Xylocaine), and procainamide (Pronestyl). Cardioversion also may be needed to correct the rhythm (cardioversion is recommended for stable ventricular tachycardia). Defibrillation is used with pulseless ventricular tachycardia. Epinephrine would stimulate and already excitable ventricle and is contraindicated.

One hour after administering IV furosemide (Lasix) to a client with heart failure, a short burst of ventricular tachycardia appears on the cardiac monitor. Which of the following electrolyte imbalances should the nurse suspect? a. Hypocalcemia b. Hypermagnesemia c. Hypokalemia d. Hypernatremia

C Furosemide is a potassium-depleting diuretic than can cause hypokalemia. In turn, hypokalemia increases myocardial excitability, leading to ventricular tachycardia.

One hour after administering IV furosemide (Lasix) to a client with heart failure, a short burst of ventricular tachycardia appears on the cardiac monitor. Which of the following electrolyte imbalances should the nurse suspect? A. Hypocalcemia B. Hypermagnesemia C. Hypokalemia D. Hypernatremia

C Furosemide is a potassium-depleting diuretic than can cause hypokalemia. In turn, hypokalemia increases myocardial excitability, leading to ventricular tachycardia.

When auscultating the apical pulse of a client who has atrial fibrillation, the nurse would expect to hear a rhythm that is characterized by: A. The presence of occasional coupled beats B. Long pauses in an otherwise regular rhythm C. A continuous and totally unpredictable irregularity D. Slow but strong and regular beat

C In atrial fibrillation, multiple ectopic foci stimulate the atria to contract. The AV node is unable to transmit all of these impulses to the ventricles, resulting in a pattern of highly irregular ventricular contractions.

Septal involvement occurs in which type of cardiomyopathy? A. Congestive B. Dilated C. Hypertrophic D. Restrictive

C In hypertrophic cardiomyopathy, hypertrophy of the ventricular septum - not the ventricle chambers - is apparent. This abnormality isn't seen in other types of cardiomyopathy.

Which of the following symptoms might a client with right-sided heart failure exhibit? A. Adequate urine output B. Polyuria C. Oliguria D. Polydipsia

C Inadequate deactivation of aldosterone by the liver after right-sided heart failure leads to fluid retention, which causes oliguria. Adequate urine output, polyuria, and polydipsia aren't associated with right-sided heart failure.

What position should the nurse place the head of the bed in to obtain the most accurate reading of jugular vein distention? A. High-fowler's B. Raised 10 degrees C. Raised 30 degrees D. Supine position

C Jugular venous pressure is measured with a centimeter ruler to obtain the vertical distance between the sternal angle and the point of highest pulsation with the head of the bed inclined between 15 and 30 degrees. Inclined pressure can't be seen when the client is supine or when the head of the bed is raised 10 degrees because the point that marks the pressure level is above the jaw (therefore, not visible). In high Fowler's position, the veins would be barely discernible above the clavicle.

To assist the patient with coronary artery disease (CAD) in making appropriate dietary changes, which of these nursing interventions will be most effective? a. Instruct the patient that a diet containing no saturated fat and minimal sodium will be necessary. b. Emphasize the increased risk for cardiac problems unless the patient makes the dietary changes. c. Assist the patient to modify favorite high-fat recipes by using monosaturated oils when possible. d. Provide the patient with a list of low-sodium, low-cholesterol foods that should be included in the diet.

C Lifestyle changes are more likely to be successful when consideration is given to the patient's values and preferences. The highest percentage of calories from fat should come from monosaturated fats. Although low-sodium and low-cholesterol foods are appropriate, providing the patient with a list alone is not likely to be successful in making dietary changes. Removing saturated fat from the diet completely is not a realistic expectation; up to 7% of calories in the therapeutic lifestyle changes (TLC) diet can come from saturated fat. Telling the patient about the increased risk without assisting further with strategies for dietary change is unlikely to be successful.

Which information obtained by the nurse who is admitting the patient for magnetic resonance imaging (MRI) will be most important to report to the health care provider before the MRI? a. The patient has an allergy to shellfish and iodine. b. The patient has a history of coronary artery disease. c. The patient has a permanent ventricular pacemaker in place. d. The patient took all the prescribed cardiac medications today.

C MRI is contraindicated for patients with implanted metallic devices such as pacemakers. The other information also will be reported to the health care provider but does not impact on whether or not the patient can have an MRI.

Which hereditary disease is most closely linked to aneurysm? A. Cystic fibrosis B. Lupus erythematosus C. Marfan's syndrome D. Myocardial infarction

C Marfan's syndrome results in the degeneration of the elastic fibers of the aortic media. Therefore, clients with the syndrome are more likely to develop an aortic aneurysm. Although cystic fibrosis is hereditary, it hasn't been linked to aneurysms. Lupus erythematosus isn't hereditary. Myocardial infarction is neither hereditary nor a disease.

A 68-year-old woman is scheduled to undergo mitral valve replacement for severe mitral stenosis and mitral regurgitation. Although the diagnosis was made during childhood, she did not have any symptoms until 4 years ago. Recently, she noticed increased symptoms, despite daily doses of digoxin and furosemide. During the initial interview with the nice lady, the nurse would most likely learn that the client's childhood health history included: A. Chicken pox B. poliomyelitis C. Rheumatic fever D. meningitis

C Most clients with mitral stenosis have a history of rheumatic fever or bacterial endocarditis.

During the previous few months, a 56-year-old woman felt brief twinges of chest pain while working in her garden and has had frequent episodes of indigestion. She comes to the hospital after experiencing severe anterior chest pain while raking leaves. Her evaluation confirms a diagnosis of stable angina pectoris. After stabilization and treatment, the client is discharged from the hospital. At her follow-up appointment, she is discouraged because she is experiencing pain with increasing frequency. She states that she is visiting an invalid friend twice a week and now cannot walk up the second flight of steps to the friend's apartment without pain. Which of the following measures that the nurse could suggest would most likely help the client deal with this problem? a. Visit her friend earlier in the day. b. Rest for at least an hour before climbing the stairs. c. Take a nitroglycerin tablet before climbing the stairs. d. Lie down once she reaches the friend's apartment.

C Nitroglycerin may be used prophylactically before stressful physical activities such as stair climbing to help the client remain pain free. Visiting her friend early in the day would have no impact on decreasing pain episodes. Resting before or after an activity is not as likely to help prevent an activity-related pain episode.

During the previous few months, a 56-year-old woman felt brief twinges of chest pain while working in her garden and has had frequent episodes of indigestion. She comes to the hospital after experiencing severe anterior chest pain while raking leaves. Her evaluation confirms a diagnosis of stable angina pectoris. After stabilization and treatment, the client is discharged from the hospital. At her follow-up appointment, she is discouraged because she is experiencing pain with increasing frequency. She states that she is visiting an invalid friend twice a week and now cannot walk up the second flight of steps to the friend's apartment without pain. Which of the following measures that the nurse could suggest would most likely help the client deal with this problem? A. Visit her friend earlier in the day B. Rest for at least an hour before climbing the stairs C. Take a nitroglycerin tablet before climbing the stairs D. Lie down once she reaches the friend's apartment

C Nitroglycerin may be used prophylactically before stressful physical activities such as stair climbing to help the client remain pain free. Visiting her friend early in the day would have no impact on decreasing pain episodes. Resting before or after an activity is not as likely to help prevent an activity-related pain episode.

As an initial step in treating a client with angina, the physician prescribes nitroglycerin tablets, 0.3mg given sublingually. This drug's principle effects are produced by: a. Antispasmotic effect on the pericardium b. Causing an increased mycocardial oxygen demand c. Vasodilation of peripheral vasculature d. Improved conductivity in the myocardium

C Nitroglycerin produces peripheral vasodilation, which reduces myocardial oxygen consumption and demand. Vasodilation in coronary arteries and collateral vessels may also increase blood flow to the ischemic areas of the heart. Nitroglycerin decreases myocardial oxygen demand. Nitroglycerin does not have an effect on pericardial spasticity or conductivity in the myocardium.

Medical treatment of coronary artery disease includes which of the following procedures? A. Cardiac catheterization B. Coronary artery bypass surgery C. Oral medication administration D. Percutaneous transluminal coronary angioplasty

C Oral medication administration is a noninvasive, medical treatment for coronary artery disease. Cardiac catheterization isn't a treatment but a diagnostic tool. Coronary artery bypass surgery and percutaneous transluminal coronary angioplasty are invasive, surgical treatments.

To evaluate a client's condition following cardiac catheterization, the nurse will palpate the pulse: a. In all extremities b. At the insertion site c. Distal to the catheter insertion d. Above the catheter insertion

C Palpating pulses distal to the insertion site is important to evaluate for thrombophlebitis and vessel occlusion. They should be bilateral and strong.

To evaluate a client's condition following cardiac catheterization, the nurse will palpate the pulse: A. In all extremities B. At the insertion site C. Distal to the catheter insertion D. Above the catheter insertion

C Palpating pulses distal to the insertion site is important to evaluate for thrombophlebitis and vessel occlusion. They should be bilateral and strong.

A 57-year-old client with a history of asthma is prescribed propranolol (Inderal) to control hypertension. Before administered propranolol, which of the following actions should the nurse take first? A. Monitor the apical pulse rate B. Instruct the client to take medication with food C. Question the physician about the order D. Caution the client to rise slowly when standing

C Propranolol and other beta-adrenergic blockers are contraindicated in a client with asthma, so the nurse should question the physician before giving the dose. The other responses are appropriate actions for a client receiving propranolol, but questioning the physician takes priority. The client's apical pulse should always be checked before giving propranolol; if the pulse rate is extremely low, the nurse should withhold the drug and notify the physician.

An elderly patient with a 40-pack-year history of smoking and a recent myocardial infarction is admitted to the medical unit with acute shortness of breath; the nurse need to rule out pneumonia versus heart failure. The diagnostic test that the nurse will monitor to help in determining whether the patient has heart failure is a. 12-lead electrocardiogram (ECG). b. arterial blood gases (ABGs). c. B-type natriuretic peptide (BNP). d. serum creatine kinase (CK).

C Rationale: BNP is secreted when ventricular pressures increase, as with heart failure, and elevated BNP indicates a probable or very probable diagnosis of heart failure. 12-lead ECGs, ABGs, and CK may also be used in determining the causes or effects of heart failure but are not as clearly diagnostic of heart failure as BNP.

A home health care patient has recently started taking oral digoxin (Lanoxin) and furosemide (Lasix) for control of heart failure. The patient data that will require the most immediate action by the nurse is if the patient's a. weight increases from 120 pounds to 122 pounds over 3 days. b. liver is palpable 2 cm below the ribs on the right side. c. serum potassium level is 3.0 mEq/L after 1 week of therapy. d. has 1 to 2+ edema in the feet and ankles in the morning.

C Rationale: Hypokalemia potentiates the actions of digoxin and increases the risk for digoxin toxicity, which can cause life-threatening dysrhythmias. The other data indicate that the patient's heart failure requires more effective therapies, but they do not require nursing action as rapidly as the low serum potassium level.

During assessment of a 72-year-old with ankle swelling, the nurse notes jugular venous distention (JVD) with the head of the patient's bed elevated 45 degrees. The nurse knows this finding indicates a. decreased fluid volume. b. incompetent jugular vein valves. c. elevated right atrial pressure. d. jugular vein atherosclerosis.

C Rationale: The jugular veins empty into the superior vena cava and then into the right atrium, so JVD with the patient sitting at a 45-degree angle reflects elevated right atrial pressure. JVD is an indicator of excessive fluid volume (increased preload), not decreased fluid volume; it is not caused by incompetent jugular vein valves or atherosclerosis.

During a visit to an elderly patient with chronic heart failure, the home care nurse finds that the patient has severe dependent edema and that the legs appear to be weeping serous fluid. Based on these data, the best nursing diagnosis for the patient is a. activity intolerance related to venous congestion. b. disturbed body image related to massive leg swelling. c. impaired skin integrity related to peripheral edema. d. impaired gas exchange related to chronic heart failure.

C Rationale: The patient's findings of severe dependent edema and weeping serous fluid from the legs support the nursing diagnosis of impaired skin integrity. There is less evidence for the nursing diagnoses of activity intolerance, disturbed body image, and impaired gas exchange, although the nurse will further assess the patient to determine whether there are other clinical manifestations of heart failure to indicate that these diagnoses are appropriate.

What supplemental medication is most frequently ordered in conjuction with furosemide (Lasix)? A. Chloride B. Digoxin C. Potassium D. Sodium

C Supplemental potassium is given with furosemide because of the potassium loss that occurs as a result of this diuretic. Chloride and sodium aren't loss during diuresis. Digoxin acts to increase contractility but isn't given routinely with furosemide.

When assessing a client for an abdominal aortic aneurysm, which area of the abdomen is most commonly palpated? A. Right upper quadrant B. Directly over the umbilicus C. Middle lower abdomen to the left of the midline D. Midline lower abdomen to the right of the midline

C The aorta lies directly left of the umbilicus; therefore, any other region is inappropriate for palpation.

A client is experiencing tachycardia. The nurse's understanding of the physiological basis for this symptom is explained by which of the following statements? A. The demand for oxygen is decreased because of pleural involvement B. The inflammatory process causes the body to demand more oxygen to meet its needs C. The heart has to pump faster to meet the demand for oxygen when there is lowered arterial oxygen tension D. Respirations are labored

C The arterial oxygen supply is lowered and the demand for oxygen is increased, which results in the heart's having to beat faster to meet the body's needs for oxygen.

During the administration of the fibrinolytic agent to a patient with an acute myocardial infarction (AMI), the nurse should stop the drug infusion if the patient experiences a. bleeding from the gums. b. surface bleeding from the IV site. c. a decrease in level of consciousness. d. a nonsustained episode of ventricular tachycardia.

C The change in level of consciousness indicates that the patient may be experiencing intracranial bleeding, a possible complication of fibrinolytic therapy. Bleeding of the gums and prolonged bleeding from IV sites are expected side effects of the therapy. The nurse should address these by avoiding any further injuries, but they are not an indication to stop infusion of the fibrinolytic medication. A nonsustained episode of ventricular tachycardia is a common reperfusion dysrhythmia and may indicate that the therapy is effective.

Which assessment finding by the nurse who is caring for a patient who has had coronary artery bypass grafting using a right radial artery graft is most important to communicate to the physician? a. Complaints of incisional chest pain b. Crackles audible at both lung bases c. Pallor and weakness of the right hand d. Redness on either side of the chest incision

C The changes in the right hand indicate compromised blood flow, which requires immediate evaluation and actions such as prescribed calcium channel blockers or surgery. The other changes are expected and/or require nursing interventions

Sublingual nitroglycerin tablets begin to work within 1 to 2 minutes. How should the nurse instruct the client to use the drug when chest pain occurs? a. Take one tablet every 2 to 5 minutes until the pain stops. b. Take one tablet and rest for 10 minutes. Call the physician if pain persists after 10 minutes. c. Take one tablet, then an additional tablet every 5 minutes for a total of 3 tablets. Call the physician if pain persists after three tablets. d. Take one tablet. If pain persists after 5 minutes, take two tablets. If pain still persists 5 minutes later, call the physician.

C The correct protocol for nitroglycerin used involves immediate administration, with subsequent doses taken at 5-minute intervals as needed, for a total dose of 3 tablets. Sublingual nitroglycerin appears in the blood stream within 2 to 3 minutes and is metabolized within about 10 minutes.

After the nurse has finished teaching a patient about use of sublingual nitroglycerin (Nitrostat), which patient statement indicates that the teaching has been effective? a. "I can expect indigestion as a side effect of nitroglycerin." b. "I can only take the nitroglycerin if I start to have chest pain." c. "I will call an ambulance if I still have pain 5 minutes after taking the nitroglycerin." d. "I will help slow down the progress of the plaque formation by taking nitroglycerin."

C The emergency medical services (EMS) system should be activated when chest pain or other symptoms are not completely relieved 5 minutes after taking one nitroglycerin. Nitroglycerin can be taken to prevent chest pain or other symptoms from developing (e.g., before intercourse). Gastric upset is not an expected side effect of nitroglycerin. Nitroglycerin does not impact the underlying pathophysiology of coronary artery atherosclerosis.

Which of the following blood vessel layers may be damaged in a client with an aneurysm? A. Externa B. Interna C. Media D. Interna and Media

C The factor common to all types of aneurysms is a damaged media. The media has more smooth muscle and less elastic fibers, so it's more capable of vasoconstriction and vasodilation. The interna and externa are generally no damaged in an aneurysm.

Which of the following factors can cause blood pressure to drop to normal levels? a. Kidneys' excretion of sodium only b. Kidneys' retention of sodium and water c. Kidneys' excretion of sodium and water d. Kidneys' retention of sodium and excretion of water

C The kidneys respond to a rise in blood pressure by excreting sodium and excess water. This response ultimately affects systolic pressure by regulating blood volume.

Which of the following factors can cause blood pressure to drop to normal levels? A. Kidneys' excretion of sodium only B. Kidneys' retention of sodium and water C. Kidneys' excretion of sodium and water D. Kidneys' retention of sodium and excretion of water

C The kidneys respond to a rise in blood pressure by excreting sodium and excess water. This response ultimately affects systolic pressure by regulating blood volume.

Which of the following arteries primarily feeds the anterior wall of the heart? a. Circumflex artery b. Internal mammary artery c. Left anterior descending artery d. Right coronary artery

C The left anterior descending artery is the primary source of blood flow for the anterior wall of the heart. The circumflex artery supplies the lateral wall, the internal mammary supplies the mammary, and the right coronary artery supplies the inferior wall of the heart.

Which of the following arteries primarily feeds the anterior wall of the heart? A. Circumflex artery B. Internal mammary artery C. Left anterior descending artery D. Right coronary artery

C The left anterior descending artery is the primary source of blood for the anterior wall of the heart. The circumflex artery supplies the lateral wall, the internal mammary artery supplies the mammary, and the right coronary artery supplies the inferior wall of the heart.

The nurse teaches the patient being evaluated for rhythm disturbances with a Holter monitor to a. exercise more than usual while the monitor is in place. b. remove the electrodes when taking a shower or tub bath. c. keep a diary of daily activities while the monitor is worn. d. connect the recorder to a telephone transmitter once daily.

C The patient is instructed to keep a diary describing daily activities while Holter monitoring is being accomplished to help correlate any rhythm disturbances with patient activities. Patients are taught that they should not take a shower or bath during Holter monitoring and that they should continue with their usual daily activities. The recorder stores the information about the patient's rhythm until the end of the testing, when it is removed and the data are analyzed.

A transesophageal echocardiogram (TEE) is ordered for a patient with possible endocarditis. Which of these actions included in the standard TEE orders will the nurse need to accomplish first? a. Administer O2 per mask. b. Start a large-gauge IV line. c. Place the patient on NPO status. d. Give lorazepam (Ativan) 1 mg IV.

C The patient will need to be NPO for 6 hours preceding the TEE, so the nurse should place the patient on NPO status as soon as the order is received. The other actions also will need to be accomplished but not until just before or during the procedure

Three days after a myocardial infarction (MI), the patient develops chest pain that increases when taking a deep breath and is relieved by leaning forward. Which action should the nurse take next? a. Palpate the radial pulses bilaterally. b. Assess the feet for peripheral edema. c. Auscultate for a pericardial friction rub. d. Check the cardiac monitor for dysrhythmias.

C The patient's symptoms are consistent with the development of pericarditis, a possible complication of MI. The other assessments listed are not consistent with the description of the patient's symptoms.

Prolonged occlusion of the right coronary artery produces an infarction in which of he following areas of the heart? A. Anterior B. Apical C. Inferior D. Lateral

C The right coronary artery supplies the right ventricle, or the inferior portion of the heart. Therefore, prolonged occlusion could produce an infarction in that area. The right coronary artery doesn't supply the anterior portion ( left ventricle ), lateral portion ( some of the left ventricle and the left atrium ), or the apical portion ( left ventricle ) of the heart.

Prolonged occlusion of the right coronary artery produces an infarction in which of the following areas of the heart? a. Anterior b. Apical c. Inferior d. Lateral

C The right coronary artery supplies the right ventricle, or the inferior portion of the heart. Therefore, prolonged occlusion could produce an infarction in that area. The right coronary artery doesn't supply the anterior portion (left ventricle), lateral portion (some of the left ventricle and the left atrium), or the apical portion (left ventricle) of the heart.

A client is at risk for pulmonary embolism and is on anticoagulant therapy with warfarin (Coumadin). The client's prothrombin time is 20 seconds, with a control of 11 seconds. The nurse assesses that this result is: The same as the client's own baseline level Lower than the needed therapeutic level Within the therapeutic range Higher than the therapeutic range

C The therapeutic range for prothrombin time is 1.5 to 2 times the control for clients at risk for thrombus. Based on the client's control value, the therapeutic range for this individual would be 16.5 to 22 seconds. Therefore the result is within therapeutic range.

A client is at risk for pulmonary embolism and is on anticoagulant therapy with warfarin (Coumadin). The client's prothrombin time is 20 seconds, with a control of 11 seconds. The nurse assesses that this result is: A. The same as the client's own baseline level B. Lower than the needed therapeutic level C. Within the therapeutic range D. Higher than the therapeutic range

C The therapeutic range for prothrombin time is 1.5 to 2 times the control for clients at risk for thrombus. Based on the client's control value, the therapeutic range for this individual would be 16.5 to 22 seconds. Therefore the result is within therapeutic range.

The nurse has just received change-of-shift report about the following four patients. Which patient should the nurse assess first? a. 38-year-old who has pericarditis and is complaining of sharp, stabbing chest pain b. 45-year-old who had a myocardial infarction (MI) 4 days ago and is anxious about the planned discharge c. 51-year-old with unstable angina who has just returned to the unit after having a percutaneous coronary intervention (PCI) d. 60-year-old with variant angina who is to receive a scheduled dose of nifedipine (Procardia)

C This patient is at risk for bleeding from the arterial access site for the PCI, so the nurse should assess the patient's blood pressure, pulse, and the access site immediately. The other patients also should be assessed as quickly as possible, but assessment of this patient has the highest priority.

Which of the following blood tests is most indicative of cardiac damage? A. Lactate dehydrogenase B. Complete blood count C. Troponin I D. Creatine kinase

C Troponin I levels rise rapidly and are detectable within 1 hour of myocardial injury. Troponin I levels aren't detectable in people without cardiac injury. Lactate dehydrogenase is present in almost all body tissues and not specific to heart muscle. LDH isoenzymes are useful in diagnosing cardiac injury. CBC is obtained to review blood counts, and a complete chemistry is obtained to review electrolytes. Because CK levels may rise with skeletal muscle injury, CK isoenzymes are required to detect cardiac injury.

Which of the following blood tests is most indicative of cardiac damage? a. Lactate dehydrogenase b. Complete blood count (CBC) c. Troponin I d. Creatine kinase (CK)

C Troponin I levels rise rapidly and are detectable within 1 hour of myocardial injury. Troponin levels aren't detectable in people without cardiac injury.

When teaching a client with a cardiac problem, who is on a high-unsaturated fatty-acid diet, the nurse should stress the importance of increasing the intake of: A. Enriched whole milk B. Red meats, such as beef C. Vegetables and whole grains D. Liver and other glandular organ meats

C Vegetables and whole grains are low in fat and may reduce the risk for heart disease.

A murmur is heard at the second left intercostal space along the left sternal border. Which valve is this? a. Aortic b. Mitral c. Pulmonic d. Tricupsid

C Abnormalities of the pulmonic valve are auscultated at the second left intercostal space along the left sternal border. Aortic valve abnormalities are heard at the second intercostal space, to the right of the sternum. Mitral valve abnormalities are heard at the fifth intercostal space in the midclavicular line. Tricupsid valve abnormalities are heard at the 3rd and 4th intercostal spaces along the sternal border.

A 57-year-old client with a history of asthma is prescribed propanolol (Inderal) to control hypertension. Before administered propranolol, which of the following actions should the nurse take first? a. Monitor the apical pulse rate b. Instruct the client to take medication with food c. Question the physician about the order d. Caution the client to rise slowly when standing.

C Propranolol and other beta-adrenergic blockers are contraindicated in a client with asthma, so the nurse should question the physician before giving the dose. The other responses are appropriate actions for a client receiving propranolol, but questioning the physician takes priority. The client's apical pulse should always be checked before giving propranolol; if the pulse rate is extremely low, the nurse should withhold the drug and notify the physician.

The action of medication is inotropic when it: A. decreases afterload B. increases heart rate C. increases the force of contraction D. is uses to treat CHF

C'

The client's serum digoxin level is 2.2 ng/dL and the heart rate is 120 and irregular. The nurse expects to administer which of the following drugs? A. Digoxin immune Fab (Digibind) B. Furosemide (Lasix) 60 mg I.V. C. Digoxin 0.5 mg bolus I.V. D. Potassium 40 mEq added to I.V. fluids

C. Digoxin immune Fab (Digibind) Rationale: Digibind binds and removes digoxin from the body and prevents toxic effects of digoxin overdose. A serum level of 2.2 is elevated, and the client is exhibiting signs of digoxin toxicity. The question does not indicate that the potassium level is low. Giving additional digoxin would exacerbate the toxicity. Giving Lasix may reduce potassium levels and contribute to increased toxicity.

The nurse developing a teaching plan for a client receiving thiazide diuretics should include the following. A. Teaching the client to take apical pulse. B. Decreasing potassium-rich foods in the diet. C. Including citrus fruits, melons, and vegetables in the diet. D. Teaching the client to check blood pressure t.i.d.

C. Including citrus fruits, melons, and vegetables in the diet. Rationale: Thiazide diuretics are potassium wasting, and levels should be closely monitored. Encouraging foods rich in potassium could help maintain potassium levels. Taking an apical pulse is indicated before administering cardiac glycosides and beta blockers. It would not be necessary to check blood pressure TID unless the client was experiencing hypotension.

The nurse is concerned when a client's heart rate, which is normally 95 beats per minute, rises to 220 beats per minute, because a rate this high will: A. Exhaust the client B. Decrease metabolic rate C. Reduce coronary artery perfusion D. Provide too much blood flow to major organs

C. Reduce coronary artery perfusion Coronary arteries fill and perfuse the myocardium (heart muscle) during diastole. When the heart rate is elevated, more time is spent in systole and less in diastole; hence, the myocardium may not be perfused adequately. The client may be exhausted, but the primary concern is myocardial perfusion. Major organs will adjust to increased blood flow. This is usually not a problem. With a heart rate this high, metabolic rate will be increased, not decreased.

The nurse reviews lab studies of a client receiving digoxin (Lanoxin). Intervention by the nurse is required if the results include a A.Serum sodium level of 140 mEq/L. B. Serum digoxin level of 1.2 ng/dL. C.Serum potassium level of 3.0 mEq/L. D. Hemoglobin 14.4 g/dL.

C. Serum potassium level of 3.0 mEq/L. Rationale: Normal serum potassium level is 3.5-5.0 mEq/L. Hypokalemia may predispose the client to digitalis toxicity. The other lab values are WNL.

The client is prescribed a beta-blocker as adjunct therapy to treatment of heart failure. The nurse recognizes that beta blockers act by A. Increasing contractility and cardiac output. B. Decreasing preload. C. Slowing the heart and decreasing afterload. D. Decreasing peripheral resistance

C. Slowing the heart and decreasing afterload. Rationale: Beta-blockers improve symptoms of HF by slowing heart rate and decreasing blood pressure. The decreased afterload causes decreased workload on the heart.

A client admitted for uncontrolled hypertension and chest pain was prescribed a low sodium diet and started on furosemide (Lasix). The nurse should instruct the client to include which foods in the diet? A. cabbage B. Liver C. Apples D. Bananas

D

A client is admitted to the ER with crushing chest pain. A diagnosis of acute coronary syndrome is suspected. The nurse expects that the client's initial treatment will include which medication? A. Gabapentin B. Midazolam C. Alprazolam D. Aspirin

D

A major consideration in the management of the older adult with hypertension is to A. prevent pseudohypertension from converting to true hypertension. B. recognize that the older adult is less likely to comply with the drug therapy than a younger adult. C. ensure that the patient receives larger initial doses of antihypertensive drugs because of impaired absorption. D. use careful technique in assessing the BP of the patient because of the possible presence of an auscultatory gap.

D

A nurse is caring for a pt receiving heparin IV. The nurse anticipates that which lab study will be prescribed to monitor the therapeutic effect of heparin? A. hematocrit B. hemoglobin C. prothrombin time D. activated partial thromboplastin time

D

A nurse provides discharge instructions to a post op pt who is taking warfarin sodium (Coumadin). Which statement, if made by the pt, reflects the need for further teaching? A. I will take my pills every day at the same time B. I will be certain to limit my alcohol consumption C. I have already called my family to pick up my Medic-Alert bracelet D. I will take enteric-coated Aspirin for my headaches

D

A patient describes nonradiating substernal chest pain precipitated by climbing three flights of stairs. If the patient has taken one sublingual NTG tablet and asks what to do next if the pain is unrelieved, the nurse's best response would be A. "Call 911." B. "Drive to the emergency department at once." C. "I'm not qualified to advise you about this. Let me call your doctor." D. "Lie down and take another NTG tablet 5 minutes after the first tablet.

D

A patient had a non-ST-segment-elevation myocardial infarction (NSTEMI) 3 days ago. Which nursing intervention included in the plan of care is most appropriate for the registered nurse (RN) to delegate to an experienced licensed practical/vocational nurse (LPN/LVN)? a. Evaluation of the patient's response to walking in the hallway b. Completion of the referral form for a home health nurse follow-up c. Education of the patient about the pathophysiology of heart disease d. Reinforcement of teaching about the purpose of prescribed medications

D

A patient who has had chest pain for several hours is admitted with a diagnosis of rule out acute myocardial infarction (AMI). Which laboratory test should the nurse monitor to help determine whether the patient has had an AMI? a. Myoglobin b. Homocysteine c. C-reactive protein d. Cardiac-specific troponin

D

A patient with a deep vein thrombosis suddenly develops dyspnea, tachypnea, and chest pain. Initially the most appropriate action by the nurse is to A. auscultate for abnormal lung sounds. B. administer oxygen and notify the physician. C. ask the patient to cough and deep breathe to clear the airways. D. elevate the head of the bed 30 to 45 degrees to facilitate breathing.

D

A patient with a stable blood pressure and no symptoms has the following electrocardiogram characteristics: atrial rate—74 and regular; ventricular rate—62 and irregular; P wave—normal contour; PR interval—lengthens progressively until a P wave is not conducted; QRS—normal contour. The nurse would expect that treatment would involve A. epinephrine 1 mg IV push. B. isoproterenol IV continuous drip. C. immediate insertion of a temporary pacemaker. D. careful observation for symptoms of hypotension.

D

A patient with hyperlipidemia has a new order for colesevelam (Welchol). Which nursing action is most appropriate when giving the medication? a. Have the patient take this medication with an aspirin. b. Administer the medication at the patient's usual bedtime. c. Have the patient take the colesevelam with a sip of water. d. Give the patient's other medications 2 hours after the colesevelam.

D

A pt who is being discharged home will be giving his own enoxaparin sodium (Lovenox) subq. Which statement by the client indicates the need for further education? A. i remember that i need to hold the needle at a 45 degree angle B. i know to pinch the skin up for the injection C. i will not pull back on the plunger before injecting the med D. i am going to use the right side of my lower abdomen all the time

D

A significant cause of venous thrombosis is: a. Altered blood coagulation b. Stasis of blood c. Vessel wall injury d. All of the above

D

After receiving change-of-shift report about the following four patients, which patient should the nurse assess first? a. 39-year-old with pericarditis who is complaining of sharp, stabbing chest pain b. 56-year-old with variant angina who is to receive a dose of nifedipine (Procardia) c. 65-year-old who had a myocardial infarction (MI) 4 days ago and is anxious about the planned discharge d. 59-year-old with unstable angina who has just returned to the unit after having a percutaneous coronary intervention (PCI)

D

After reviewing a patient's history, vital signs, physical assessment, and laboratory data, which information shown in the accompanying figure is most important for the nurse to communicate to the health care provider? a. Q waves on ECG b. Elevated troponin levels c. Fever and hyperglycemia d. Tachypnea and crackles in lungs

D

An 80-year-old patient who lives in a skilled nursing facility and likes to walk is taking a medication that lowers blood pressure by dilating blood vessels. The most appropriate nursing action for this patient is to A. monitor blood pressure every day. B. order complete bed rest. C. monitor intake and output. D. assist the patient when ambulating.

D

During the cardiac assessment, the nurse finds a client has jugular vein distention. What does this mean to the nurse? a. The client is fine. b. The client is dehydrated. c. The client has an infection. d. The client could have fluid overload

D

During the cardiac assessment, the nurse learns a client had rheumatic heart fever as a child. For which of the following cardiac conditions should this client be assessed? a. Pulmonic stenosis b. Aortic stenosis c. Mitral regurgitation d. Mitral stenosis

D

During the cardiac focused interview, the client tells the nurse he has smoked for 30 years. Which of the following would be appropriate for the nurse to respond to this client? a. High dose vitamin therapy reduces the risks of smoking. b. Exercise reduces the harmful effects of smoking. c. Do you also take recreational drugs? d. Smoking has been linked to high blood pressure and other heart problems.

D

During the examination of an eight-month-pregnant client, the nurse measures the blood pressure at 160/98 and notes bilateral edema of the ankles. What do these findings suggest to the nurse? a. This client is going into labor early. b. The client is in normal health. c. The client is hyperventilating. d. The client could be preeclamptic.

D

Etiology of heart failure after MI is related to the A. increased myocardial workload. B. increased oxygen demands of the myocardium. C. inability of the heart chambers to fill adequately. D. impairment of the contractile function of the ventricle.

D

If a patient has decreased cardiac output caused by fluid volume deficit and marked vasodilation, the regulatory mechanism that will increase the blood pressure by improving both of these is A. release of antidiuretic hormone (ADH). B. secretion of prostaglandins PGE C. stimulation of the sympathetic nervous system. D. activation of the renin-angiotensin-aldosterone system.

D

If a patient is being given a drug that blocks the action of the sympathetic division of the autonomic nervous system, the patient should be monitored for which of the following cardiovascular clinical manifestations? A. Increased heart rate and increased blood pressure B. Increased heart rate and decreased blood pressure C. Decreased heart rate and increased blood pressure D. Decreased heart rate and decreased blood pressure

D

If a patient self-administers 325 mg of acetylsalicylic acid (aspirin) daily, the health care provider should assess for which of the following most common side effects of salicylate therapy? A. Skin tears B. Hypothermia C. Hepatotoxicity D. GI distress

D

If a patient who has experienced an MI develops left ventricular heart failure, for what sign of poor organ perfusion should a nurse remain alert? A. Headache B. Hypertension C. Heart rate of 55 to 60 beats/min D. Urine output of less than 30 ml/hour

D

If a patient with dilated cardiomyopathy has an atrial fibrillation that has been unresponsive to drug therapy for several days, the nurse should anticipate that further treatment of the patient will require A. carotid massage. B. immediate electrical cardioversion. C. a transvenous implantable cardioverter-defibrillator. D. treatment with anticoagulants before additional treatment.

D

In teaching a patient with hypertension about controlling the condition, the nurse recognizes that A. all patients with elevated BP require medication. B. it is not necessary to limit salt in the diet if taking a diuretic. C. obese persons must achieve a normal weight in order to lower BP. D. lifestyle modifications are indicated for all persons with elevated BP.

D

Nadolol (Corgard) is prescribed for a patient with chronic stable angina and left ventricular dysfunction. To determine whether the drug is effective, the nurse will monitor for a. decreased blood pressure and heart rate. b. fewer complaints of having cold hands and feet. c. improvement in the strength of the distal pulses. d. the ability to do daily activities without chest pain.

D

Nurse Margie just administered an ACE inhibitor to her pt. Before ambulating the pt for the first time after administration, the nurse should monitor for: A. hypokalemia B. irregular heartbeat C. edema D. hypotension

D

Prolonged pressure on the skin causes reddened areas at the point of contact due to A. arterial vasodilation from smooth muscle relaxation. B. compression of veins resulting in venous engorgement. C. occlusion of major arteries causing infarction of the tissue. D. tissue damage and inflammation resulting from impaired capillary blood flow.

D

Target organ damage that can occur from hypertension includes A. headache and dizziness. B. retinopathy and diabetes. C. hypercholesterolemia and renal dysfunction. D. renal dysfunction and left ventricular hypertrophy.

D

The action of an ACE inhibitor interrupts the renin-angiotensin-aldosterone mechanism, thereby producing which of the following? A. reduced renal blood flow B. reduced sodium and water retention C. increased peripheral vascular resistance D. increased sodium excretion and potassium reabsorption

D

The clinical spectrum of acute coronary syndrome includes A. unstable angina and STEMI. B. unstable angina and NSTEMI. C. stable angina and sudden cardiac death. D. unstable angina, STEMI, and NSTEMI.

D

The healthcare provider is examining the electrocardiogram (EKG) of a patient and notes the PR interval is 6 small boxes in length. What is the significance of this finding? A. Stress is causing increased sympathetic stimulation. B. This should be documented as an expected finding. C. There may be some scar tissue in one of the ventricles. D. There may be a delay in the conduction through the AV node.

D

The nurse instructs the patient discharged on anticoagulant therapy to A. limit intake of vitamin C. B. report symptoms of nausea to the physician. C. have blood drawn routinely to check electrolytes. D. be aware of and report signs or symptoms of bleeding.

D

The nurse is calculating the cardiac output for a client with a heart rate of 88 beats per minute. What other measurements does the nurse need to complete this calculation? a. Cardiac index b. QT interval c. T wave d. Stroke volume

D

The nurse is scheduled to administer a dose of digoxin to and adult pt with atrial fibrillation. The pt has a potassium level of 4.3 mEq/L. The nurse should perform which of the following activities next? A. withhold dose only for that day B. obtain order for dose of potassium before giving digoxin C. withhold dose and notify prescriber D. administer dose as ordered

D

The nurse monitors the patient receiving treatment for acute decompensated heart failure with the knowledge that marked hypotension is most likely to occur with the IV administration of A. furosemide (Lasix). B. milrinone (Primacor). C. nitroglycerin (Tridil). D. nitroprusside (Nipride).

D

The nurse prepares a patient for electrical cardioversion knowing that cardioversion differs from defibrillation in that A. defibrillation requires a greater dose of electrical current. B. defibrillation is synchronized to countershock during the QRS complex. C. cardioversion is indicated only for treatment of atrial tachyarrhythmias. D. cardioversion may be done on a nonemergency basis with sedation of the patient.

D

The primary purpose of a soft, high-fiber diet immediately following a myocardial infarction (MI) is to A. promote easy digestion. B. lower cholesterol levels. C. maintain bowel health to decrease gas. D. create a high-bulk, soft stool to minimize Valsalvar maneuver.

D

The therapeutic drug level for digoxin is: A. 0.1-2.0 B. 1.0-2.0 C. 0.1-0.5 D. 0.5-2.0

D

What is the leading cause of PAD? a) smoking b) hyperlipidemia c) hypertension d) atherosclerosis

D

When caring for a patient with acute coronary syndrome who has returned to the coronary care unit after having angioplasty with stent placement, the nurse obtains the following assessment data. Which data indicate the need for immediate action by the nurse? a. Heart rate 102 beats/min b. Pedal pulses 1+ bilaterally c. Blood pressure 103/54 mm Hg d. Chest pain level 7 on a 0 to 10 point scale

D

When checking the capillary filling time of a patient, the color returns in 10 seconds. The nurse recognizes this finding as indicative of A. a normal response. B. thrombus formation in the veins. C. lymphatic obstruction of venous return. D. impaired arterial flow to the extremities.

D

When developing a teaching plan for a 61-year-old man with the following risk factors for coronary artery disease (CAD), the nurse should focus on the a. family history of coronary artery disease. b. increased risk associated with the patient's gender. c. increased risk of cardiovascular disease as people age. d. elevation of the patient's low-density lipoprotein (LDL) level.

D

Which information given by a patient admitted with chronic stable angina will help the nurse confirm this diagnosis? a. The patient states that the pain "wakes me up at night." b. The patient rates the pain at a level 3 to 5 (0 to 10 scale). c. The patient states that the pain has increased in frequency over the last week. d. The patient states that the pain "goes away" with one sublingual nitroglycerin tablet.

D

Which of the following changes in vital signs would most strongly suggest cardiac tamponade? A. Tachypnea B. Bradycardia C. Muffled heart sounds D. A narrowing pulse pressure

D

Which of the following information should be given to a patient before taking hydrochlorothiazide for control of hypertension? A. "You can develop a slower pulse rate." B. "You might notice some swelling in your feet." C. "You can develop shortness of breath or a cough." D. "Your diet should include foods high in potassium."

D

Which of the following is a potential side effect of IV furosemide (Lasix)? A. drowsiness B. diarrhea C. cystitis D. hearing loss

D

Which of the following is a potential side effect of IV furosemide (lasix)? A. drowsiness B. diarrhea C. cystitis D. hearing loss

D

Which of the following pts is at greatest risk for digital toxicity? A. a 25 y/o pt with congestive heart disease B. a 50 y/o pt with CHF C. a 60 y/o pt who had an MI D. an 80 y/o pt with CHF

D

Which of the following pts is most likely to experience adverse effects from tx with diuretics? A. a 21 y/o college student B. a 40 y/o unmarried man C. a 60 y/o widower D. a 75 y/o man

D

Which statement made by a patient with coronary artery disease after the nurse has completed teaching about therapeutic lifestyle changes (TLC) diet indicates that further teaching is needed? a. "I will switch from whole milk to 1% milk." b. "I like salmon and I will plan to eat it more often." c. "I can have a glass of wine with dinner if I want one." d. "I will miss being able to eat peanut butter sandwiches."

D

While assessing a pt with angina who is to start Beta Blocker therapy, the nurse is aware that the presence of which condition may be a problem if these drugs are used: A. hypertension B. essential tremors C. exertional angina D. asthma

D

a 68 y/o male is experiencing severe hypotension after receiving IV nitroglycerin. What is the most probable cause for this reaction? A. pt is having a MI B. the pt has a high potassium level C. the pt routinely exercises, but has not be active in over a week D. the pt failed to report that he is on Viagra

D

all potassium-sparing diuretics: A. are required during blood transfusions B. enhance aldosterone action C. cause hypokalemia D. are weak diuretics

D

f a patient with heart failure is being treated with digoxin and has developed hypokalemia, the nurse should A. administer digoxin twice daily. B. reduce the digoxin dose to every other day. C. administer an IV bolus of potassium. D. monitor the patient for toxic effects that can occur at normal doses.

D

After having an MI, the nurse notes the patient has jugular venous distention, gained weight, developed peripheral edema, and has a heart rate of 108/minute. What should the nurse suspect is happening? a. ADHF b. Chronic HF c. Left-sided HF d. Right-sided HF

D An MI is a primary cause of heart failure. The jugular venous distention, weight gain, peripheral edema, and increased heart rate are manifestations of right-sided heart failure.

The nurse is administering a dose of digoxin (Lanoxin) to a patient with heart failure (HF). The nurse would become concerned with the possibility of digitalis toxicity if the patient reported which symptom(s)? a. Muscle aches b. Constipation c. Pounding headache d. Anorexia and nausea

D Anorexia, nausea, vomiting, blurred or yellow vision, and cardiac dysrhythmias are all signs of digitalis toxicity. The nurse would become concerned and notify the health care provider if the patient exhibited any of these symptoms.

Beyond the first year after a heart transplant, the nurse knows that what is a major cause of death? a. Infection b. Acute rejection c. Immunosuppression d. Cardiac vasculopathy

D Beyond the first year after a heart transplant, malignancy (especially lymphoma) and cardiac vasculopathy (accelerated CAD) are the major causes of death. During the first year after transplant, infection and acute rejection are the major causes of death. Immunosuppressive therapy will be used for posttransplant management to prevent rejection and increases the patient's risk of an infection.

The recommended treatment for an initial VTE in an otherwise healthy person with no significant co-morbidities would include a. IV argatroban (Acova) as an inpatient. b. IV unfractionated heparin as an inpatient. c. subcutaneous unfractionated heparin as an outpatient. d. subcutaneous low-molecular-weight heparin as an outpatient.

D Rationale: Patients with confirmed VTE should receive initial treatment with low-molecular-weight heparin (LMWH), unfractionated heparin (UH), fondaparinux, or rivaroxaban, followed by warfarin for 3 months to maintain the international normalized ratio (INR) between 2.0 and 3.0 for 24 hours. Patients with multiple comorbid conditions, complex medical issues, or a very large VTE usually are hospitalized for treatment and typically receive intravenous UH. LMWH only for 3 months is another option for patients with acute VTE. Depending on the clinical presentation, patients often can be managed safely and effectively as outpatients.

A male patient was admitted for a possible ruptured aortic aneurysm, but had no back pain. Ten minutes later his assessment includes the following: sinus tachycardia at 138, BP palpable at 65 mm Hg, increasing waist circumference, and no urine output. How should the nurse interpret this assessment about the patient's aneurysm? a Tamponade will soon occur. b The renal arteries are involved. c Perfusion to the legs is impaired. d He is bleeding into the abdomen.

D Rationale: The lack of back pain indicates the patient is most likely exsanguinating into the abdominal space, and the bleeding is likely to continue without surgical repair. A blockade of the blood flow will not occur in the abdominal space as it would in the retroperitoneal space where surrounding anatomic structures may control the bleeding. The lack of urine output does not indicate renal artery involvement, but that the bleeding is occurring above the renal arteries, which decreases the blood flow to the kidneys. There is no assessment data indicating decreased perfusion to the legs.

The nurse is preparing to administer a scheduled dose of enoxaparin (Lovenox) 30 mg subcutaneously. What should the nurse do to administer this medication correctly? a. Remove the air bubble in the prefilled syringe. b. Aspirate before injection to prevent IV administration. c. Rub the injection site after administration to enhance absorption. d. Pinch the skin between the thumb and forefinger before inserting the needle

D Rationale: The nurse should gather together or "bunch up" the skin between the thumb and the forefinger before inserting the needle into the subcutaneous tissue. The nurse should not remove the air bubble in the prefilled syringe, aspirate, nor rub the site after injection.

The nurse is caring for a patient with a recent history of deep vein thrombosis (DVT). The patient now needs to undergo surgery for appendicitis. The nurse is reviewing the laboratory results for this patient before administering an ordered dose of vitamin K. The nurse determines that the medication is both safe to give and is most needed when the international normalized ratio (INR) is which result? a. 1.0 b. 1.2 c. 1.6 d. 2.2

D Rationale: itamin K is the antidote to warfarin (Coumadin), which the patient has most likely been taking before admission for treatment of DVT. Warfarin is an anticoagulant that impairs the ability of the blood to clot. Therefore it is necessary to give vitamin K before surgery to reduce the risk of hemorrhage. The largest value of the INR indicates the greatest impairment of clotting ability, making 2.2 the correct selection.

A male patient with a long-standing history of heart failure has recently qualified for hospice care. What measure should the nurse now prioritize when providing care for this patient? a. Taper the patient off his current medications. b. Continue education for the patient and his family. c. Pursue experimental therapies or surgical options. d. Choose interventions to promote comfort and prevent suffering

D The central focus of hospice care is the promotion of comfort and the prevention of suffering. Patient education should continue, but providing comfort is paramount. Medications should be continued unless they are not tolerated. Experimental therapies and surgeries are not commonly used in the care of hospice patients.

A stable patient with acute decompensated heart failure (ADHF) suddenly becomes dyspneic. Before positioning the patient on the bedside, what should the nurse assess first? a. Urine output b. Heart rhythm c. Breath sounds d. Blood pressure

D The nurse should evaluate the blood pressure before dangling the patient on the bedside because the blood pressure can decrease as blood pools in the periphery and preload decreases. If the patient's blood pressure is low or marginal, the nurse should put the patient in the semi-Fowler's position and use other measures to improve gas exchange.

An elderly client is being monitored for evidence of congestive heart failure. To detect early signs of heart failure, the nurse would instruct the certified nursing attendant (CNA) to do which of the following during care of the patient? Observe electrocardiogram readings and report deviations to the nurse. Assist the client with ambulation three times during the shift. Monitor vital signs every 15 minutes and report each reading to the nurse. Accurately weigh the patient, and report and record the readings.

D Rationale: Due to fluid accumulation, an expanded blood volume can result when the heart fails. Body weight is a sensitive indicator of water and sodium retention, which will manifest itself with edema, dyspnea - especially nocturnal - and pedal edema. Patients also should be instructed about the need to perform daily weights upon discharge to monitor body water. It is not within the role of the CNA to monitor ECG readings, and ambulation is not an assessment. Vital signs every 15 minute are not necessary for this level of patient care.

Nurses can best help older clients prevent hypertension by teaching: Low-fat, low-cholesterol diets. The importance of exercise. How to handle stressful situations. How to maintain a normal blood pressure.

D Rationale: Hypertension is a major risk factor for other cardiovascular conditions. In persons older than 50, systolic blood pressure greater than 140 mm Hg is a much more important cardiovascular disease risk factor than is diastolic blood pressure. The risk of cardiovascular disease, beginning at 115/75 mm Hg, doubles with each increment of 20/10 mm Hg. Answers 1, 2, and 3 are important elements to include in education of a patient with blood pressure elevation, and are included in the correct answer.

Which of the following drug classifications should the nurse question if prescribed for a person with congested heart failure (CHF)? Angiotensin-converting enzyme (ACE) inhibitor Beta-adrenergic blocker Alpha adrenergic antagonist Rosiglitazone (Avandia)

D Correct answer: Thiazolidinediones, like rosiglitazone (Avandia), are glucose-reducing drugs that are prescribed for persons with type 2 diabetes mellitus. ACE inhibitors, such as Lisinopril, are first-line drugs used to treat CHF. Propranolol (Inderal), a beta blocker, has remained one of the most widely used beta-blocking drugs. It blocks both beta1 and beta2 receptors in various organs, resulting in reduction of heart rate and the force of contraction, and suppresses impulse conduction through the AV node, all of which slows the progression of the disease process. Carvedilol (Coreg) is another beta-adrenergic blocker used to treat heart failure.

When caring for a patient who has just arrived on the medical-surgical unit after having cardiac catheterization, which nursing action should the nurse delegate to an LPN/LVN? a. Perform the initial assessment of the catheter insertion site. b. Teach the patient about the usual postprocedure plan of care. c. Check the rate on the infusion pump used to administer heparin. d. Administer the scheduled aspirin and lipid-lowering medication.

D Administration of oral medications is within the scope of practice for LPNs/LVNs. The initial assessment of the patient, patient teaching, and administration of intravenous anticoagulant medications should be done by the RN.

Which of the following cardiac conditions does a fourth heart sound (S4) indicate? A. Dilated aorta B. Normally functioning heart C. Decreased myocardial contractility D. Failure of the ventricle to eject all the blood during systol

D An S4 occurs as a result of increased resistance to ventricular filling after atrial contraction. This increased resistance is related to decrease compliance of the ventricle. A dilated aorta doesn't cause an extra heart sound, though it does cause a murmur. Decreased myocardial contractility is heard as a third heart sound. An s4 isn't heard in a normally functioning heart.

A nurse is assessing the blood pressure of a client diagnosed with primary hypertension. The nurse ensures accurate measurement by avoiding which of the following? Seating the client with arm bared, supported, and at heart level. Measuring the blood pressure after the client has been seated quietly for 5 minutes. Using a cuff with a rubber bladder that encircles at least 80% of the limb. Taking a blood pressure within 15 minutes after nicotine or caffeine ingestion.

D BP should be taken with the client seated with the arm bared, positioned with support and at heart level. The client should sit with the legs on the floor, feet uncrossed, and not speak during the recording. The client should not have smoked tobacco or taken in caffeine in the 30 minutes preceding the measurement. The client should rest quietly for 5 minutes before the reading is taken. The cuff bladder should encircle at least 80% of the limb being measured. Gauges other than a mercury sphygmomanometer should be calibrated every 6 months to ensure accuracy.

When developing a health teaching plan for a 60-year-old man with the following risk factors for coronary artery disease (CAD), the nurse should focus on the a. family history of coronary artery disease. b. increased risk associated with the patient's gender. c. high incidence of cardiovascular disease in older people. d. elevation of the patient's serum low density lipoprotein (LDL) level.

D Because family history, gender, and age are nonmodifiable risk factors, the nurse should focus on the patient's LDL level. Decreases in LDL will help reduce the patient's risk for developing CAD.

A client is receiving spironolactone to treat hypertension. Which of the following instructions should the nurse provide? a. "Eat foods high in potassium." b. "Take daily potassium supplements." c. "Discontinue sodium restrictions." d. "Avoid salt substitutes."

D Because spironolactone is a potassium-sparing diuretic, the client should avoid salt substitutes because of their high potassium content. The client should also avoid potassium-rich foods and potassium supplements. To reduce fluid-volume overload, sodium restrictions should continue.

Nadolol (Corgard) is prescribed for a patient with angina. To determine whether the drug is effective, the nurse will monitor for a. decreased blood pressure and apical pulse rate. b. fewer complaints of having cold hands and feet. c. improvement in the quality of the peripheral pulses. d. the ability to do daily activities without chest discomfort.

D Because the medication is ordered to improve the patient's angina, effectiveness is indicated if the patient is able to accomplish daily activities without chest pain. Blood pressure (BP) and apical pulse rate may decrease, but these data do not indicate that the goal of decreased angina has been met. The noncardioselective -blockers can cause peripheral vasoconstriction, so the nurse would not expect an improvement in peripheral pulse quality or skin temperature

After myocardial infarction, serum glucose levels and free fatty acids are both increase. What type of physiologic changes are these? A. Electrophysiologic B. Hematologic C. Mechanical D. Metabolic

D Both glucose and fatty acids are metabolites whose levels increase after a myocardial infarction. Mechanical changes are those that affect the pumping action of the heart, and electro physiologic changes affect conduction. Hematologic changes would affect the blood.

A client with a bundle branch block is on a cardiac monitor. The nurse should expect to observe: A. Sagging ST segments B. Absence of P wave configurations C. Inverted T waves following each QRS complex D. Widening of QRS complexes to 0.12 second or greater

D Bundle branch block interferes with the conduction of impulses from the AV node to the ventricle supplied by the affected bundle. Conduction through the ventricles is delayed, as evidenced by a widened QRS complex.

Cholesterol, frequently discussed in relation to atherosclerosis, is a substance that: A. May be controlled by eliminating food sources B. Is found in many foods, both plant and animal sources C. All persons would be better off without because it causes the disease process D. Circulates in the blood, the level of which usually decreases when unsaturated fats are substituted for saturated fats

D Cholesterol is a sterol found in tissue; it is attributed in part to diets high in saturated fats.

Exceeding which of the following serum cholesterol levels significantly increases the risk of coronary artery disease? A. 100 mg/dl B. 150 mg/dl C. 175 mg/dl D. 200 mg/dl

D Cholesterol levels above 200 mg/dl are considered excessive. They require dietary restriction and perhaps medication. Exercise also helps reduce cholesterol levels. The other levels listed are all below the nationally accepted levels for cholesterol and carry a lesser risk for CAD.

Which information given by a patient admitted with chronic stable angina will help the nurse confirm this diagnosis? a. The patient rates the pain at a level 3 to 5 (0 to 10 scale). b. The patient states that the pain "wakes me up at night." c. The patient says that the frequency of the pain has increased over the last few weeks. d. The patient states that the pain is resolved after taking one sublingual nitroglycerin tablet.

D Chronic stable angina is typically relieved by rest or nitroglycerin administration. The level of pain is not a consistent indicator of the type of angina. Pain occurring at rest or with increased frequency is typical of unstable angina.

The nurse receives emergency laboratory results for a client with chest pain and immediately informs the physician. An increased myoglobin level suggests which of the following? a. Cancer b. Hypertension c. Liver disease d. Myocardial infarction

D Detection of myoglobin is one diagnostic tool to determine whether myocardial damage has occurred. Myoglobin is generally detected about one hour after a heart attack is experienced and peaks within 4 to 6 hours after infarction (Remember, less than 90 mg/L is normal).

When the nurse is monitoring a patient who is undergoing exercise (stress) testing on a treadmill, which assessment finding requires the most rapid action by the nurse? a. Patient complaint of feeling tired. b. Pulse change from 80 to 96 beats/minute. c. BP increase from 134/68 to 150/80 mm Hg. d. Electrocardiographic (ECG) changes indicating coronary ischemia.

D ECG changes associated with coronary ischemia (such as T-wave inversions and ST segment depression) indicate that the myocardium is not getting adequate oxygen delivery and that the exercise test should be terminated immediately. Increases in BP and heart rate (HR) are normal responses to aerobic exercise. Tiredness also is normal as the intensity of exercise increases during the stress testing.

A patient with a non-ST-segment-elevation myocardial infarction (NSTEMI) is receiving heparin. What is the purpose of the heparin? a. Platelet aggregation is enhanced by IV heparin infusion. b. Heparin will dissolve the clot that is blocking blood flow to the heart. c. Coronary artery plaque size and adherence are decreased with heparin. d. Heparin will prevent the development of new clots in the coronary arteries

D Heparin helps prevent the conversion of fibrinogen to fibrin and decreases coronary artery thrombosis. It does not change coronary artery plaque, dissolve already formed clots, or enhance platelet aggregation.

The adaptations of a client with complete heart block would most likely include: A. Nausea and vertigo B. Flushing and slurred speech C. Cephalalgia and blurred vision D. Syncope and slow ventricular rate

D In complete atrioventricular block, the ventricles take over the pacemaker function in the heart but at a much slower rate than that of the SA node. As a result there is decreased cerebral circulation, causing syncope.

To determine the effects of therapy for a patient who is being treated for heart failure, which laboratory result will the nurse plan to review? a. Myoglobin b. Homocysteine (Hcy) c. Low-density lipoprotein (LDL) d. B-type natriuretic peptide (BNP)

D Increased levels of BNP are a marker for heart failure. The other laboratory results would be used to assess for myocardial infarction (myoglobin) or risk for coronary artery disease (Hcy and LDL).

Which of the following classes of medications maximizes cardiac performance in clients with heat failure by increasing ventricular contractility? A. Beta-adrenergic blockers B. Calcium channel blockers C. Diuretics D. Inotropic agents

D Inotropic agents are administered to increase the force of the heart's contractions, thereby increasing ventricular contractility and ultimately increasing cardiac output. Beta-adrenergic blockers and calcium channel blockers decrease the heart rate and ultimately decrease the workload of the heart. Diuretics are administered to decrease the overall vascular volume, also decreasing the workload of the heart.

Which of these nursing interventions included in the plan of care for a patient who had an acute myocardial infarction (AMI) 3 days ago is most appropriate for the RN to delegate to an experienced LPN/LVN? a. Evaluating the patient's response to ambulation in the hallway b. Completing the documentation for a home health nurse referral c. Educating the patient about the pathophysiology of heart disease d. Reinforcing teaching about the purpose of prescribed medications

D LPN/LVN education and scope of practice include reinforcing education that has previously been done by the RN. Evaluating the patient response to exercise after an AMI requires more education and should be done by the RN. Teaching and discharge planning/documentation are higher level skills that require RN education and scope of practice.

Which of the following symptoms usually signifies rapid expansion and impending rupture of an abdominal aortic aneurysm? A. Abdominal pain B. Absent pedal pulses C. Angina D. Lower back pain

D Lower back pain results from expansion of the aneurysm. The expansion applies pressure in the abdominal cavity, and the pain is referred to the lower back. Abdominal pain is most common symptom resulting from impaired circulation. Absent pedal pulses are a sign of no circulation and would occur after a ruptured aneurysm or in peripheral vascular disease. Angina is associated with atherosclerosis of the coronary arteries.

What is the primary reason for administering morphine to a client with myocardial infarction? A. To sedate the client B. To decrease the client's pain C. To decrease the client's anxiety D. To decrease oxygen demand on the client's heart

D Morphine is administered because it decreases myocardial oxygen demand. Morphine will also decrease pain and anxiety while causing sedation, but isn't primarily given for those reasons.

The nurse will suspect that the patient with stable angina is experiencing a side effect of the prescribed metoprolol (Lopressor) if a. the patient is restless and agitated. b. the blood pressure is 190/110 mm Hg. c. the patient complains about feeling anxious. d. the cardiac monitor shows a heart rate of 45.

D Patients taking -blockers should be monitored for bradycardia. Because this category of medication inhibits the sympathetic nervous system, restlessness, agitation, hypertension, and anxiety will not be side effects.

Which of the following terms is used to describe the amount of stretch on the myocardium at the end of diastole? a. Afterload b. Cardiac index c. Cardiac output d. Preload

D Preload is the amount of stretch of the cardiac muscle fibers at the end of diastole. The volume of blood in the ventricle at the end of diastole determines the preload. Afterload is the force against which the ventricle must expel blood. Cardiac index is the individualized measurement of cardiac output, based on the client's body surface area. Cardiac output is the amount of blood the heart is expelling per minute.

Which of the following terms is used to describe the amount of stretch on the myocardium at the end of diastole? A. Afterload B. Cardiac index C. Cardiac output D. Preload

D Preload is the amount of stretch of the cardiac muscle fibers at the end of diastole. The volume of blood in the ventricle at the end of diastole determines the preload. Afterload is the force against which the ventricle must expel blood. Cardiac index is the individualized measurement of cardiac output, based on the client's body surface area. Cardiac output is the amount of blood the heart is expelling per minute.

Following an acute myocardial infarction, a previously healthy 67-year-old patient develops clinical manifestations of heart failure. The nurse anticipates discharge teaching will include information about a. digitalis preparations, such as digoxin (Lanoxin). b. calcium-channel blockers, such as diltiazem (Cardizem). c. -adrenergic agonists, such as dobutamine (Dobutrex). d. angiotensin-converting enzyme (ACE) inhibitors, such as captopril (Capoten).

D Rationale: ACE-inhibitor therapy is currently recommended to prevent the development of heart failure in patients who have had a myocardial infarction and as a first-line therapy for patients with chronic heart failure. Digoxin therapy for heart failure is no longer considered a first-line measure, and digoxin is added to the treatment protocol when therapy with other medications such as ACE-inhibitors, diuretics, and -adrenergic blockers is insufficient. Calcium-channel blockers are not generally used in the treatment of heart failure. The -adrenergic agonists such as dobutamine are administered through the IV route and are not used as initial therapy for heart failure.

The nurse is caring for a patient receiving IV furosemide (Lasix) 40 mg and enalapril (Vasotec) 5 mg PO bid for ADHF with severe orthopnea. When evaluating the patient response to the medications, the best indicator that the treatment has been effective is a. weight loss of 2 pounds overnight. b. improvement in hourly urinary output. c. reduction in systolic BP. d. decreased dyspnea with the head of the bed at 30 degrees.

D Rationale: Because the patient's major clinical manifestation of ADHF is orthopnea (caused by the presence of fluid in the alveoli), the best indicator that the medications are effective is a decrease in crackles. The other assessment data also may indicate that diuresis or improvement in cardiac output have occurred but are not as useful in evaluating this patient's response.

Which of the following diagnostic studies most likely would confirm a myocardial infarction? Serum myoglobin level Creatinine kinase (CK) White blood cell count (WBC) Troponin T levels

D Rationale: CK-MB elevates 4-6 hours after tissue necrosis. Troponin levels rise 6-8 hours after the infarct (tissue necrosis) but also can occur with other types of tissue damage. Myoglobin also elevates, but to a lesser degree. WBC levels elevate with an inflammatory response. Troponin levels are more elevated than are the other cardiac enzymes, are more specific to cardiac tissue, and rise 6-8 hours after the infarct (tissue necrosis).

A hospitalized patient with heart failure has a new order for captopril (Capoten) 12.5 mg PO. After administering the first dose and teaching the patient about captopril, which statement by the patient indicates that teaching has been effective? a. "I will need to include more high-potassium foods in my diet." b. "I will expect to feel more short of breath for the next few days." c. "I will be sure to take the medication after eating something." d. "I will call for help when I need to get up to the bathroom."

D Rationale: Captopril can cause hypotension, especially after the initial dose, so it is important that the patient not get up out of bed without assistance until the nurse has had a chance to evaluate the effect of the first dose. The ACE inhibitors are potassium sparring, and the nurse should not teach the patient to increase sources of dietary potassium. Increased shortness of breath is expected with initiation of -blocker therapy for heart failure, not for ACE-inhibitor therapy. ACE inhibitors are best absorbed when taken an hour before eating.

The nurse plans discharge teaching for a patient with chronic heart failure who has prescriptions for digoxin (Lanoxin), hydrochlorothiazide (HydroDIURIL), and a potassium supplement. Appropriate instructions for the patient include a. avoid dietary sources of potassium because too much can cause digitalis toxicity. b. take the pulse rate daily and never take digoxin if the pulse is below 60 beats/min. c. take the hydrochlorothiazide before bedtime to maximize activity level during the day. d. notify the health care provider immediately if nausea or difficulty breathing occurs.

D Rationale: Difficulty breathing is an indication of acute decompensated heart failure and suggests that the medications are not achieving the desired effect. Nausea is an indication of digoxin toxicity and should be reported so that the provider can assess the patient for toxicity and adjust the digoxin dose, if necessary. Digoxin toxicity is potentiated by hypokalemia, rather than hyperkalemia. Patients should be taught to check their pulse daily before taking the digoxin and, if the pulse is less than 60, to call their provider before taking the digoxin. Diuretics should be taken early in the day to avoid sleep disruption.

A 55-year-old patient with inoperable coronary artery disease and end-stage heart failure asks the nurse whether heart transplant is a possible therapy. The nurse's response to the patient will be based on the knowledge that a. heart transplants are experimental surgeries that are not covered by most insurance. b. the patient is too old to be placed on the transplant list. c. the diagnoses and symptoms indicate that the patient is not an appropriate candidate. d. candidacy for heart transplant depends on many factors.

D Rationale: Indications for a heart transplant include inoperable coronary artery disease and refractory end-stage heart failure, but other factors such as coping skills, family support, and patient motivation to follow the rigorous post-transplant regimen are also considered. Heart transplants are not considered experimental; rather, transplantation has become the treatment of choice for patients who meet the criteria. The patient is not too old for a transplant. The patient's diagnoses and symptoms indicate that the patient may be an appropriate candidate for a heart transplant.

When teaching the patient with heart failure about a 2000-mg sodium diet, the nurse explains that foods to be restricted include a. eggs and other high-cholesterol foods. b. canned and frozen fruits. c. fresh or frozen vegetables. d. milk, yogurt, and other milk products

D Rationale: Milk and yogurt naturally contain a significant amount of sodium, and intake of these should be limited for patients on a diet that limits sodium to 2000 mg daily. Other milk products, such as processed cheeses, have very high levels of sodium and are not appropriate for a 2000-mg sodium diet. The other foods listed have minimal levels of sodium and can be eaten without restriction.

Modification of lifestyle behaviors to help manage hypertension does not include which of the following? (Select all that apply.) Weight loss of even 10 pounds The DASH diet Fruits, vegetables, and whole grains Alcohol intake with meals

D Rationale: Weight loss of even 10 pounds; fruits, vegetables, and whole grains; the DASH diet; and a daily exercise regimen will help reduce high blood pressure. Alcohol intake with meals should be reduced to help manage high blood pressure.

Which of the following diagnostic tools is most commonly used to determine the location of myocardial damage? a. Cardiac catherization b. Cardiac enzymes c. Echocardiogram d. Electrocardiogram (ECG)

D The ECG is the quickest, most accurate, and most widely used tool to determine the location of myocardial infarction. Cardiac enzymes are used to diagnose MI but can't determine the location. An echocardiogram is used most widely to view myocardial wall function after an MI has been diagnosed. Cardiac catherization is an invasive study for determining coronary artery disease and may also indicate the location of myocardial damage, but the study may not be performed immediately.

Which of the following diagnostic tools is most commonly used to determine the location of myocardial damage? A. Cardiac catheterization B. Cardiac enzymes C. Echocardiogram D. Electrocardiogram

D The ECG is the quickest, most accurate, and most widely used tool to determine the location of myocardial infarction. Cardiac enzymes are used to diagnose MI but can't determine the location. An echocardiogram is used most widely to view myocardial wall function after an MI has been diagnosed. Cardiac catheterization is an invasive study for determining coronary artery disease and may also indicate the location of myocardial damage, but the study may not be performed immediately.

When assessing an ECG, the nurse knows that the P-R interval represents the time it takes for the: A. Impulse to begin atrial contraction B. Impulse to transverse the atria to the AV node C. SA node to discharge the impulse to begin atrial depolarization D. Impulse to travel to the ventricles

D The P-R interval is measured on the ECG strip from the beginning of the P wave to the beginning of the QRS complex. It is the time it takes for the impulse to travel to the ventricle.

Intravenous heparin therapy is ordered for a client. While implementing this order, a nurse ensures that which of the following medications is available on the nursing unit? a. Vitamin K b. Aminocaproic acid c. Potassium chloride d. Protamine sulfate

D The antidote to heparin is protamine sulfate and should be readily available for use if excessive bleeding or hemorrhage should occu

IV heparin therapy is ordered for a client. While implementing this order, a nurse ensures that which of the following medications is available on the nursing unit? Vitamin K Aminocaporic acid Potassium chloride Protamine sulfate

D The antidote to heparin is protamine sulfate and should be readily available for use if excessive bleeding or hemorrhage should occur. Vitamin K is an antidote for warfarin.

IV heparin therapy is ordered for a client. While implementing this order, a nurse ensures that which of the following medications is available on the nursing unit? A. Vitamin K B. Aminocaproic acid C. Potassium chloride D. Protamine sulfate

D The antidote to heparin is protamine sulfate and should be readily available for use if excessive bleeding or hemorrhage should occur. Vitamin K is an antidote for warfarin.

A patient with hyperlipidemia has a new order for colesevelam (Welchol). Which nursing action is appropriate when giving the medication? a. Administer the medication at the patient's bedtime. b. Have the patient take this medication with an aspirin. c. Encourage the patient to take the colesevelam with a sip of water. d. Give the patient's other medications 2 hours after the colesevelam.

D The bile acid sequestrants interfere with the absorption of other drugs, and giving other medications at the same time should be avoided. Taking an aspirin concurrently with the colesevelam may increase the incidence of gastrointestinal side effects such as heartburn. An increased fluid intake is encouraged for patients taking the bile acid sequestrants to reduce the risk for constipation. For maximum effect, colesevelam should be administered with meals.

The nurse obtains the following data when caring for a patient who experienced an acute myocardial infarction (AMI) 2 days previously. Which information is most important to report to the health care provider? a. The patient denies ever having a heart attack. b. The cardiac-specific troponin level is elevated. c. The patient has occasional premature atrial contractions (PACs). d. Crackles are auscultated bilaterally in the mid-lower lobes.

D The crackles indicate that the patient may be developing heart failure, a possible complication of myocardial infarction (MI). The health care provider may need to order medications such as diuretics or angiotensin-converting enzyme (ACE) inhibitors for the patient. Elevation in cardiac troponin level at this time is expected. PACs are not life-threatening dysrhythmias. Denial is a common response in the immediate period after the MI.

When administering IV nitroglycerin (Tridil) to a patient with a myocardial infarction (MI), which action will the nurse take to evaluate the effectiveness of the medication? a. Check blood pressure. b. Monitor apical pulse rate. c. Monitor for dysrhythmias. d. Ask about chest discomfort.

D The goal of IV nitroglycerin administration in MI is relief of chest pain by improving the balance between myocardial oxygen supply and demand. The nurse also will monitor heart rate and BP and observe for dysrhythmias, but these parameters will not indicate whether the medication is effective.

In which of the following disorders would the nurse expect to assess sacral edema in bedridden client? A. DM B. Pulmonary emboli C. Renal failure D. Right-sided heart failure

D The most accurate area on the body to assed dependent edema in a bedridden client is the sacral area. Sacral, or dependent, edema is secondary to right-sided heart failure. Diabetes mellitus, pulmonary emboli, and renal disease aren't directly linked to sacral edema.

Which of the following types of pain is most characteristic of angina? a. Knifelike b. Sharp c. Shooting d. Tightness

D The pain of angina usually ranges from a vague feeling of tightness to heavy, intense pain. Pain impulses originate in the most visceral muscles and may move to such areas as the chest, neck, and arms.

Which of the following types of pain is most characteristic of angina? A. Knifelike B. Sharp C. Shooting D. Tightness

D The pain of angina usually ranges from a vague feeling of tightness to heavy, intense pain. Pain impulses originate in the most visceral muscles and may move to such areas as the chest, neck, and arms.

Which electrocardiographic (ECG) change is most important for the nurse to communicate to the health care provider when caring for a patient with chest pain? a. Frequent premature atrial contractions (PACs) b. Inverted P wave c. Sinus tachycardia d. ST segment elevation

D The patient is likely to be experiencing an ST-segment-elevation myocardial infarction (STEMI) and immediate therapy with percutaneous coronary intervention (PCI) or fibrinolytic medications is indicated to minimize the amount of myocardial damage. The other ECG changes also may suggest a need for therapy, but not as rapidly.

After the nurse teaches a patient with chronic stable angina about how to use the prescribed short-acting and long-acting nitrates, which statement by the patient indicates that the teaching has been effective? a. "I will put on the nitroglycerin patch as soon as I develop any chest pain." b. "I will check the pulse rate in my wrist just before I take any nitroglycerin." c. "I will be sure to remove the nitroglycerin patch before using any sublingual nitroglycerin." d. "I will stop what I am doing and sit down before I put the nitroglycerin under my tongue."

D The patient should sit down before taking the nitroglycerin to decrease cardiac workload and prevent orthostatic hypotension. Transdermal nitrates are used prophylactically rather than to treat acute pain and can be used concurrently with sublingual nitroglycerin. Although the nurse should check blood pressure before giving nitroglycerin, patients do not need to check the pulse rate before taking nitrates.

When caring for a patient with acute coronary syndrome who has returned to the coronary care unit after having balloon angioplasty, the nurse obtains the following assessment data. Which data indicate the need for immediate intervention by the nurse? a. Pedal pulses 1+ b. Heart rate 100 beats/min c. Blood pressure 104/56 mm Hg d. Chest pain level 8 on a 10-point scale

D The patient's chest pain indicates that restenosis of the coronary artery may be occurring and requires immediate actions, such as administration of oxygen and nitroglycerin, by the nurse. The other information indicates a need for ongoing assessments by the nurse.

Dyspnea, cough, expectoration, weakness, and edema are classic signs and symptoms of which of the following conditions? A. Pericarditis B. Hypertension C. Obliterative D. Restrictive

D These are the classic symptoms of heart failure. Pericarditis is exhibited by a feeling of fullness in the chest and auscultation of a pericardial friction rub. Hypertension is usually exhibited by headaches, visual disturbances and a flushed face. Myocardial infarction causes heart failure but isn't related to these symptoms.

A client is scheduled for a cardiac catherization using a radiopaque dye. Which of the following assessments is most critical before the procedure? Intake and output Baseline peripheral pulse rates Height and weight Allergy to iodine or shellfish

D This procedure requires an informed consent because it involves injection of a radiopaque dye into the blood vessel. The risk of allergic reaction and possible anaphylaxis is serious and must be assessed before the procedure.

A client's physician orders nuclear cardiography and makes an appointment for a thallium scan. The purpose of injecting radioisotope into the bloodstream is to detect: a. Normal vs. abnormal tissue b. Damage in areas of the heart c. Ventricular function d. Myocardial scarring and perfusion

D This scan detects myocardial damage and perfusion, an acute or chronic MI. It is a more specific answer than (1) or (2). Specific ventricular function is tested by a gated cardiac blood pool scan.

Which of the following parameters is the major determinate of diastolic blood pressure? a. Baroreceptors b. Cardiac output c. Renal function d. Vascular resistance

D Vascular resistance is the impedance of blood flow by the arterioles that most predominantly affects the diastolic pressure. Cardiac output determines systolic blood pressure.

Which of the following conditions is most closely associated with weight gain, nausea, and a decrease in urine output? A. Angina pectoris B. Cardiomyopathy C. Left-sided heart failure D. Right-sided heart failure

D Weight gain, nausea, and a decrease in urine output are secondary effects of right-sided heart failure. Cardiomyopathy is usually identified as a symptom of left-sided heart failure. Left-sided heart failure causes primarily pulmonary symptoms rather than systemic ones. Angina pectoris doesn't cause weight gain, nausea, or a decrease in urine output.

The nurse expects that a client with mitral stenosis would demonstrate symptoms associated with congestion in the: A. Aorta B. Right atrium C. Superior vena cava D. Pulmonary circulation

D When mitral stenosis is present, the left atrium has difficulty emptying its contents into the left ventricle. Hence, because there is no valve to prevent backward flow into the pulmonary vein, the pulmonary circulation is under pressure.

Which of the following treatments is the definitive one for a ruptured aneurysm? A. Antihypertensive medication administration B. Aortogram C. Beta-adrenergic blocker administration D. Surgical intervention

D When the vessel ruptures, surgery is the only intervention that can repair it. Administration of antihypertensive medications and beta-adrenergic blockers can help control hypertension, reducing the risk of rupture. An aortogram is a diagnostic tool used to detect an aneurysm.

Left-sided heart failure is characterized by: A. Increased cardiac output B. Lowered cardiac pressures C. Decreased functioning of the left atrium D. Decreased functioning of the left ventricle

D. Decreased functioning of the left ventricle Left-sided heart failure is an abnormal condition characterized by decreased functioning of the left ventricle. If left ventricular failure is significant, the amount of blood ejected from the left ventricle drops greatly, which results in decreased cardiac output. Progress

A person who starts smoking in adolescence and continues to smoke into middle age: A. Has an increased risk for alcoholism B. Has an increased risk for obesity and diabetes C. Has an increased risk for stress-related illnesses D. Has an increased risk for cardiopulmonary disease and lung cancer

D. Has an increased risk for cardiopulmonary disease and lung cancer The risk of lung cancer is 10 times greater for a person who smokes than for a nonsmoker. Cigarette smoking worsens peripheral vascular and coronary artery disease. Inhaled nicotine causes vasoconstriction of peripheral and coronary blood vessels, increasing blood pressure and decreasing blood flow to peripheral vessels.

A patient with chronic HF and atrial fibrillation is treated with a digitalis glycoside and a loop diuretic. To prevent possible complications of this combination of drugs, what does the nurse need to do (select all that apply)? a. Monitor serum potassium levels b. teach the patient how to take a pulse rate. c. keep an accurate measure of intake and output d. Teach the patient about dietary restriction or potassium e. Withhold digitalis and notify health care provider if heart rate is irregular

a & b Rationale: Hypokalemia, which can be caused by the use of potassium-depleting diuretics (e.g., thiazides, loop diuretics), is one of the most common causes of digitalis toxicity. Low serum levels of potassium enhance the action of digitalis, causing a therapeutic dose to achieve toxic levels. Hypokalemia can also precipitate dysrhythmias. Monitoring the serum potassium levels of patients receiving digitalis preparations and potassium-depleting diuretics is essential. Patients taking digitalis preparations should be taught how to measure their pulse rate because bradycardia and atrioventricular blocks are late signs of digitalis toxicity. In addition, patients should know what pulse rate would necessitate a call to the health care provider.

A nurse is caring for a client with hypertension. The physician orders furosemide (lasix) 2 mg/kg to be given intravenously. The client weighs 24 kg. The medication comes in a single-use vial that contains 40 mg in 4 mL (10 mg/mL). How much will the nurse draw up for this client's dose? a) 4.8 mL b) 2.4 mL c) 0.24 mL d) 0.48 mL

a- 4.8 mL Explanation: The formula is as follows: 24Kg X 2 mg = 48 mg total dose 48 mg / 10mg/mL = 4.8 mL amount to be drawn up

The nurse is assisting the client to manage the cardiovascular risk factors of hyperlipidemia and hypertension. The client asks what kind of a diet would be best. The nurse's correct response is which of the following? a) A diet low in sodium, fat, cholesterol b) A diet with restricted fruits and fluids c) A diet with high sodium, fruits, vegetables d) A diet high in transfats and potassium

a- A diet low in sodium, fat, cholesterol Explanation: Diets that are restricted in sodium, fat, and cholesterol are commonly prescribed to manage the cardiovascular risk factors of hypertension and hyperlipidemia.

You are the clinic nurse doing assessments on your clients before they have outpatient diagnostic testing done. What would you document when assessing the client's pulse? a) Rate, quality, and rhythm b) Rate, rhythm, and volume c) Quality, volume, and rate d) Pressure, rate, and rhythm

a- Rate, quality, and rhythm Explanation: Assess apical and radial pulses, noting rate, quality, and rhythm. Pulse quality and volume are not assessed in this instance.

When caring for a client with essential hypertension what instruction should the nurse provide to the client to normalize blood pressure? a) Reduce sodium intake. b) Increase iodine intake. c) Increase intake of fluids. d) Avoid intake of low-fat diet.

a- Reduce sodium intake. Explanation: The nurse advises the client with essential hypertension to reduce sodium intake. The nurse also advises the client to reduce oral fluid to decrease circulating blood volume and systemic vascular resistance and adhere to a low-fat diet.

Within the physiology of the heart, each chamber has a particular role in maintaining cellular oxygenation. Which chamber of the heart is responsible for receiving deoxygenated blood from the venous system? a) Right atrium b) Left atrium c) Right ventricle d) Left ventricle

a- Right atrium Explanation: The right atrium receives deoxygenated blood from the venous system.

A client is at risk for excess fluid volume. Which nursing intervention ensures the most accurate monitoring of the client's fluid status? a) Weighing the client daily at the same time each day b) Measuring and recording fluid intake and output c) Assessing the client's vital signs every 4 hours d) Checking the client's lungs for crackles during every shift

a- Weighing the client daily at the same time each day Explanation: Increased fluid volume leads to rapid weight gain — 2.2 lb (1 kg) for each liter of fluid retained. Weighing the client daily at the same time and in similar clothing provides more objective data than measuring fluid intake and output, which may be inaccurate because of omitted measurements such as insensible losses. Changes in vital signs are less reliable than daily weight because these changes usually are subtle during early stages of fluid retention. Weight gain is an earlier sign of excess fluid volume than crackles, which represent pulmonary edema. The nurse should plan to detect fluid accumulation before pulmonary edema occurs

The nurse recognizes that primary manifestations of systolic failure include: a. decreased EF and increased PAWP b. decreased PAWP and increased EF. c. decreased pulmonary hypertension associated with normal EF d. decreased afterload and decreased left ventricular end-diastolic pressure

a. decreased EF and increased PAWP Rationale: Systolic heart failure results in systolic failure in the left ventricle (LV). The LV loses its ability to generate enough pressure to eject blood forward through the aorta. This results in increased pulmonary artery wedge pressure (PAWP). The hallmark of systolic failure is a decrease in the left ventricular ejection fraction (EF).

The clinic nurse caring for a client with a cardiovascular disorder is performing an assessment of the client's pulse. Which of the following steps is involved in determining the pulse deficit? a) Count the radial pulse for 20 to 25 seconds. b) Count the heart rate at the apex. c) Calculate the palpated volume. d) Calculate the pauses between pulsations.

b- Count the heart rate at the apex. Explanation: The nurse determines the pulse deficit by counting the heart rate through auscultation at the apex while a second nurse simultaneously palpates and counts the radial pulse for a full minute. The difference, if any, is the pulse deficit. The pulse quality refers to its palpated volume. Pulse rhythm is the pattern of the pulsations and the pauses between them.

The nurse prepares to auscultate heart sounds. Which nursing interventions would be most effective to assist with this procedure? a) Ask the client to take deep breaths through his mouth while the nurse auscultates heart sounds. b) Explain to the client that the nurse will be listening to different areas of the chest and may listen for a long time, but that does not mean that anything abnormal is heard. c) Ask the client to sit on the edge of the bed and hold his breath while the nurse listens. d) Insist that the family members leave the room if they must speak to each other while the nurse is auscultating heart sounds.

b- Explain to the client that the nurse will be listening to different areas of the chest and may listen for a long time, but that does not mean that anything abnormal is heard. Explanation: During auscultation the client remains supine and the room should be as quiet as possible while the nurse listens to heart sounds. The client should breathe quietly during the examination.

The ability of the cardiac muscle to shorten in response to an electrical impulse is termed a) diastole. b) contractility. c) repolarization. d) depolarization.

b-contractility. Explanation: Contractility is the ability of the cardiac muscle to shorten in response to an electrical impulse. Depolarization is the electrical activation of a cell caused by the influx of sodium into the cell while potassium exits the cell. Repolarization is the return of the cell to the resting state, caused by reentry of potassium into the cell while sodium exits the cell. Diastole is the period of ventricular relaxation resulting in ventricular filling.

A nurse is assessing a client with an abdominal aortic aneurysm. Which of the following assessment findings by the nurse is probably unrelated to the aneurysm? a. Pulsatile abdominal mass b. Hyperactive bowel sounds in that area c. Systolic bruit over the area of the mass d. Subjective sensation of "heart beating" in the abdomen.

b. Not all clients with abdominal aortic aneurysms exhibit symptoms. Those who do describe a feeling of the "heart beating" in the abdomen when supine or be able to feel the mass throbbing. A pulsatile mass may be palpated in the middle and upper abdomen. A systolic bruit may be auscultated over the mass. Hyperactive bowel sounds are not related specifically to an abdominal aortic aneurysm.

The healthcare provider is caring for a patient with a diagnosis of mitral stenosis. When auscultating the patient's chest, which type of heart sound should the healthcare provider expect to hear? Please choose from one of the following options. a. Systolic ejection murmur b. Midsystolic murmur c. Diastolic murmur d. Pansystolic murmur

c

You are caring for a patient with ADHF who is receiving IV dobutamine (Dobutrex). You know that this drug is ordered because it (select all that apply): a. incerases SVR b. produces diuresis c. improves contractility d. dilates renal blood vessels e. works on the B1-receptors in the heart.

c & e Rationale: Dobutamine (Dobutrex) has a positive chronotropic effect and increases heart rate and improves contractility. It is a selective β-adrenergic agonist and works primarily on the β1-adrenergic receptors in the heart. It is frequently used in the short-term management of acute decompensated heart failure (ADHF).

The nurse is assessing the client's cardiovascular system. The client asks the nurse why she presses on his toenails. Which is the best reply by the nurse? a) "I can tell a lot about your respiratory rate from pressing on your toes." b) "Pressing on your toenail gives me an idea about how well you have been eating." c) "I can see how quickly the blood returns when I press and release your toenail. This tells me how well your peripheral blood is flowing." d) "I can learn things about your blood coagulation by pressing on your toenail."

c- "I can see how quickly the blood returns when I press and release your toenail. This tells me how well your peripheral blood is flowing." Explanation: Decreased capillary refill time indicates a slower peripheral blood flow.

A nurse is discussing with a nursing student how to accurately measure blood pressure. Which of the following points does the nurse emphasize? a) A cuff that is too small will give a false low blood pressure. b) A cuff that is too large will give a false high blood pressure. c) A cuff that is too small will give a false high blood pressure. d) The size of the cuff does not matter as long as it fits snugly around the arm.

c- A cuff that is too small will give a false high blood pressure. Explanation: Using a cuff that is too small will give a false high blood pressure measurement, while using a cuff that is too large results in a false low blood pressure measurement.

The nurse is reviewing the morning laboratory test results for a client with cardiac problems. Which of the following would the nurse regard as a priority to report to the physician? a) Mg++ 2 mE/L b) Na+ 140 mEq/L c) K+ 3.1 mEq/L d) Ca++ 9 mg/dL

c- K+ 3.1 mEq/L Explanation: All are within normal limits except for the K+, which is low. A low K+ level can cause ventricular tachycardia or fibrillation.

A compensatory mechanism involved in HF that leads to inappropriate fluid retention and additional workload of the heart is: a. ventricular dilation b. ventricular hypertrophy c. neurohormonal response d. sympathetic nervous system activation

c. neurohormonal response Rationale: The following mechanisms in heart failure lead to inappropriate fluid retention and additional workload of the heart: activation of the renin-angiotensin-aldosterone system (RAAS) cascade and release of antidiuretic hormone from the posterior pituitary gland in response to low cerebral perfusion pressure that results from low cardiac output.

The client asks the nurse what his urine output has to do with his cardiac function. The best reply by the nurse is which of the following? a) "High urine output indicates poor cardiac function." b) "Poor urine output indicates a need for a blood transfusion." c) "Too much urine output indicates a need for a blood transfusion." d) "The urine output is an important indicator of cardiac function; poor urine output may indicate inadequate blood flow to the kidneys."

d- "The urine output is an important indicator of cardiac function; poor urine output may indicate inadequate blood flow to the kidneys." Explanation: Urine output is an important indicator of cardiac function. Reduced urine output may indicate inadequate renal perfusion.

To assess peripheral edema, which of the following are particular areas for examination? a) Lips, earlobes b) Uppper arms c) Under the sacrum d) Feet, ankles

d- Feet, ankles Explanation: When right-sided heart failure occurs, blood accumulates in the vessels and backs up in peripheral veins, and the extra fluid enters the tissues. Particular areas for examination are the dependent parts of the body, such as the feet and ankles. Other prominent areas prone to edema are the fingers, hands, and over the sacrum. Cyanosis can be detected by noting color changes in the lips and earlobes

The nurse auscultates the apex beat at which of the following anatomical locations? a) Midsternum b) 1 inch to the left of the xiphoid process c) 2 inch to the left of the lower end of the sternum d) Fifth intercostal space, midclavicular line

d- Fifth intercostal space, midclavicular line Explanation: The left ventricle is responsible for the apex beat or the point of maximum impulse, which is normally palpable in the left midclavicular line of the chest wall at the fifth intercostal space. The right ventricle lies anteriorly, just beneath the sternum. Use of inches to identify the location of the apex beat is inappropriate based upon variations in human anatomy. Auscultation below and to the left of the xiphoid process will detect gastrointestinal sounds, but not the apex beat of the heart.

Within the physiology of the heart, each chamber has a particular role in maintaining cellular oxygenation. Which chamber of the heart is responsible for pumping blood to all the cells and tissues of the body? a) Right ventricle b) Right atrium c) Left atrium d) Left ventricle

d- Left ventricle Explanation: The left ventricle pumps that blood to all the cells and tissues of the body.

What is considered the pacemaker of the heart? a) The AV node b) The bundle of HIS c) The Purkinje fibers d) The SA node

d- The SA node Explanation: The SA node is called the pacemaker of the heart, because it initiates the electrical impulses that cause the atria and ventricles to contract. Normally, it produces between 60 and 100 impulses per minute; the average is approximately 72 impulses per minute. Therefore, options A, B, and C are incorrect.

While auscultating the heart sounds of a client with heart failure, the nurse hears an extra heart sound immediately after the second heart sound (S2). The nurse should document this as: a) a first heart sound (S1). b) a murmur. c) a fourth heart sound (S4). d) a third heart sound (S3).

d- a third heart sound (S3). Explanation: An S3 is heard following an S2, which commonly occurs in clients experiencing heart failure and results from increased filling pressures. An S1 is a normal heart sound made by the closing of the mitral and tricuspid valves. An S4 is heard before an S1 and is caused by resistance to ventricular filling. A murmur is heard when there is turbulent blood flow across the valves.

When the nurse observes that the patient has increased difficulty breathing when lying flat, the nurse records that the patient is demonstrating a) dyspnea on exertion. b) orthopnea. c) paroxysmal nocturnal dyspnea. d) hyperpnea.

d- orthopnea. Explanation: Patients with orthopnea prefer not to lie flat and will need to maintain their beds in a semi- to high Fowler's position. Dyspnea on exertion refers to difficulty breathing with activity. Hyperpnea refers to increased rate and depth of respiration. Paroxysmal nocturnal dyspnea refers to orthopnea that occurs only at night


Kaugnay na mga set ng pag-aaral

Chapter 4 - Property description and appraisal math

View Set

Fundamentals- Quiz #8 (Exam 3 Material)

View Set

Real Estate Principles 2 - Financing Principles

View Set

Simultaneous equations: Solution by Elimination

View Set

Social Studies Ancient Greece Assessment

View Set

History 3.3 The Maritime Provinces

View Set

Strength And Conditioning Exam 2

View Set

A Beka: Vocabulary Spelling Poetry V Quiz 08B (English 11)

View Set